AANP FNP Certification Exam 2023 Questions and Answers (Verified Answers)

All diastolic murmurs are pathological. Murmurs Grades I-barely II-audible III- clearly audible. IV- first time thrill V-Steth edge VI-entire steth. EXAM
III first time audible, IV first time thrill

Fundal height 12 weeks
Fundal Height 12 weeks above symphysis pubis. EXAM

Fundus 16 weeks between symphysis pubis and umbilicus.
Fundus at 20 weeks is at umbilicus.
2 cm more of less from # of wk gestation is normal if more or less order US

3 month old infant with down syndrome, due to milk intolerance, mom started on goats milk; now has pale conjunctiva but otherwise healthy. Low HCT. What additional test would you order?
Iron, TIBC

3 months of synthroid, TSH increased, T4 normal, what do you do?
Increase Medication

3 ways to assess cognitive function in patient with signs/symptoms of memory loss
Mini mental exam

4 month old with strabismus, mom is worried……
tell her it is normal.

4 month old wont keep anything down, what is the main thing you look at?
Growth chart

6 month old closed anterior fontanel.
XRAY

Abnormal cells on PAP, what do you do next?
Refer for Colposcopy

CAGE ACRONYM
Cut down
Annoyed by criticism
Guilty about drinking
Eye opener drink

Causes of tachycardia
Fever
Anemia
Hypotension

Cranial nerves responsible for extraocular eye movements
CN 3,4,6

Definition of metabolic syndrome
cluster of conditions that increase risk of heart disease, stroke, diabetes.

diagnose trichomoniasis
wet prep

Elderly presents with atrophic vaginitis, small uterus, palpable 4×5 ovary, what do you do next?
Pelvic US

Epistaxis is most common in the area of the nose known as kiesselbachs triangle, where is this located?
Anterior septum

Definitive diagnosis of acute bacterial prostatitis
urinalysis and culture

GERD treatment
H2 is first line, give hs

Grade 3 cells on Pap, treatment?
LEEP
excision

Fingernail hematoma treatment?
drill hole and drain blood?

Increased risk of ectopic pregnancy
Salpingitis, or history of abortion, PID,

Koplick spots
Measles (rubeola). Grains of salt lesions inside mouth in Measles

Koplick spots
Measles (rubeola). Grains of salt lesions inside mouth in Measles

Legg-Calve-Perthes Disease
Avascular necrosis of the proximal femoral head

Lipid level of 1500, increased risk for?
Pancreatitis

Low HGB, Low HCT, High MCV indicates what?
Macrocytic anemia, B12 Def

Man with BPH, prostate feels on digital exam?
Enlarged, symmetrical, smooth

Man with HTN, CAD, present femoral pulses but absent pedal
Arterial Insufficiency

McMurray’s Sign
(+) palpable or audible click while extending with varus stress
Meniscus tears

Lachman’s Test
pivot shift test (ACL tear)

Newborn with foot turned in, what do you do?
refer to orthopedist

Osgood-Schlatter disease
Knee pain.
inflammation or irritation of the tibia at its point of attachment with the patellar tendon

Patient forgot to start Thanksgiving dinner and husband states she has trouble remembering tasks and trouble with organization. What is this indicative of?
Alzheimer’s

Pt has Barretts Esophagus, insurance no longer covers GI who was treating condition. Pt at FNP office wanting refill prescriptions. What do you do?
Refer to oncologist

Pt presents with rash on shoulder, erythematous maculopapular rash with center clearing and scaling?
Tinea Corporis

Pt presents with “bag of worms:, indicates?
Varicocele

Pt with atopic dermatitis, look for what other diseases?
Asthma

Pt with bleeding after menopause
endometrial biopsy, need to screen for cancer

Pt with hx of PID, increased rick for?
Infertility

Pt with HIV took high potency anti viral treatments and CD4 is >400, what does this indicate?
This is good. Want higher than 350

Pt with hx of htn and stroke, now having memory loss. What does this indicate?
Vascular dementia

Pregnant teacher with exposure to 5ths disease (SLAP CHEEK), what risk is there to the fetus?
slap cheek, PVB19, rash hands / feet Fetal death and birth defects

Quick assessment of patients fall risk? Timed Get up and Go
Timed Get up and Go

Red beefy tongue?
pernicious anemia

Rotator cuff injury presentation
disturbs sleep, arm weakness, dull ache

Shingles near eye
immediate referral to ophthalmology

Signs and symptoms of Roseola (6ths disease) ?
Viral infection
Can result in a maculopapular rash, but up to 70% of cases proceed without the rash stage
-Usually accompanied by a high fever (41°C or 105°F) that comes on quickly and lasts up to 3 days followed by rash
-Seizures may occur during this period.
-On the fourth day, the fever disappears and the rash appears, first on chest and trunk, then less prominently on the face and limbs.
-High fever, pink flat or raised rash

Treatment for chronic alcoholism:
12 step program

Treatment for Gonorrhea?
Rocephin 250mg IMx1 plus
Azithromycin 1 gm orally x1 to cover chlamydia. , or doxy 100 mg BID x7d. Green colored vaginal discharge, friable cervix. EXAM
Report to health department

Young female want birth control, forgets to take pills, does not want to get pregnant for at least 5 years:
IUD

Basal cell cancer
Waxy, pearly, telangiectasia, ulcer center lesion
-most common type of skin cancer caused by UV exposure. Metastatic is rare

Actinic Keratosis
Scaly red to yellow located in sun exposed area
-a precancerous skin growth that occurs on sun-damaged skin

Actinic Keratosis
Numerous round dry pink to red areas. Scaly red to yellow located in sun exposed area
-a pre squamous cell carcinoma occurs on sun-damaged skin
biopsy
cryo or 5fu cream

subungual hematoma tx
Make a hole and drain the blood

Moderate acne treatment
Without inflammation: Topical retinoid

Moderate inflammation: Topical Retinoid or benzoyl peroxide or Azaleic acic ( very expensive and hard to get covered)

PLUS ADD

Oral antibiotics: doxycycline or tetracycline or minocycline- (tetracyclines has been proven most affective for inflammatory acne)

Minocycline- long term use has been linked with pseudotumor cerebri

Oral contraceptives for hormone related

Spironolactone

heart murmur with holosystolic or pan systolic

Heart mumur with mid systolic
MR – radiate axilla, 5th ICS MCL, apex,

AS – radiate neck, 2ICS right sternal border
MR ASSH

Coarctation of Aorta
COA: bounding radial and weak femoral pulse
increase blood pressure in arms, and lower pressure in lower legs.

-congenital cardiac condition characterized by a narrowing of the aorta

Murmur Grade III – VI
Loud murmur easily heard

JVD caused by
-tension pneumothorax,
-Rt. sided heart failure,
-cardiac tamponade,
-traumatic axphysia from
Increase in portal pressure(LIVER) in venous side or cor pulmonale

pt. with gradual onset of fever, hemorrhages on nail beds, painful raised red nodules, rash on palms
endocarditis , painful red spots on fingers olser’s nodes, janeway legions rash on palms and soles.

Know the difference between Peripheral Arterial Disease and Chronic Venous Insufficiency or PVD. There was question about PAD.
PAD: Absence of pulse, decrease blow flowing down, PAIN, Dx doppler or ABI<0.9, TX exercise by walking or antiplatelet,

PVD: Volume, edema, discoloration, decrease blood going up,

Chronic Bronchitis Treatment
Smoking cessation
Pulmonary rehabilitation
Pharmacologic therapy
Supplemental oxygen

TB… PPD is positive if area of induration is:

5 mm in an immunocompromised patient or close contact
10 mm in immigrant, health care workers, drug user
15 mm in a patient who lives in an area where TB is very rare.

what condition would make you order Lateral X-ray of the neck. Options include: Drooling, Unable to do ROM of the neck / stiff neck.
Croup/Epiglottitis

AV nicking (Arterioles pressing on vein of the eye)
HTN retinopathy

intraocular pressure (IOP)
Fluid pressure inside the eye; measured with tonometry

Rovsing sign
pain in the RLQ when the LLQ is palpated (indicative of appendicitis)

Pencil-like stools occur in an obstruction of what
Sigmoid descending colon
It’s a thin narrow stool and possible causes include colon cancer, diarrhea, IBS. Refer for GI colonoscopy

headache after trauma
SDH

migraine headache
-paroxysmal (sudden, periodic) attacks of mostly unilateral headache, often accompanied by disordered vision, nausea, or vomiting, lasting hours or days and caused by dilation of arteries.

4-12 hours,

abortive triptans

prophylaxis propranolol, TCA amitriptyline, anticonvulsants topiramate

Vitamin B12
Folate shares a close relationship with this other B Vitamin.

Vitamin B12
Cognitive deficits, glossitis, pernicious anemia,
Folate shares a close relationship with this other B Vitamin.

88/yr. old patient in for follow up secondary. She’s been treated with Tylenol for Joint arthritis. Her SED rate was checked after 6 weeks of treatment and it was 28. Normal range is from something to 25. How would you treat the pt.
be changed to NSAID, SED rate is a sign of inflammation

Which medication causes low sperm count for a patient
SSRI

grandiosity
Exaggerated belief in or claims about one’s importance or identity.
Bipolor

ADHD
A behavioral problem characterized by short attention span, restless movement, and impaired learning capacity.

pt. pap’s smear noted with Low Grade Squamous Intraepithelial Lesions and High Grade Squamous Epithelia Lesion noted on the report, what should NP do?
HPV test if not done. Refer for colposcopy

Chlamydia trachomatis
Doxycycline (+ ceftriaxone for gonorrhea coinfection)

Genital warts treatment
Cryotherapy
OR
Podophyllotoxin cream –
OR
Imiquimod (Aldara cream)

HIV pt. with antiviral and CD4 count still less than 200. What should NP tell the pt.
tell the pt. that he is qualified to be diagnosed with AIDS according to CDC

Most common cause of death in children
motor vehicle

Tanner 2
Tanner 2- female breast bud areola develops Male testes scrotum start to enlarge, scrotum gets darker

Tanner 4
Tanner 4- female nipples and areola become elevated from breast, secondary mound Male penis grows wider grows in length, darker scrotum

pt. expericieng memory loss, and increase in confusion and she has a history of stroke, HTN, What type of Alzheimer.
vascular dementia

presbycusis
a gradual loss of sensorineural hearing that occurs as the body ages

a pregnant female at slightly above symphysis pubic and Fundal height is 32cm (above the umbilical). What should be done
Ultrasound

Romberg test
cerebellar
-ask client to stand with feet at comfortable distance apart, arms at sides, and eyes closed
-expected finding: client should be able to stand with minimal swaying for at least 5 seconds

Direct Coombs test
r/o bilirubin

preeclampsia treatment
bed rest, laying on her side

Molluscum contagiosum treatment
Currettage, liquid nitrogen

pyloric stenosis
non bilious vomiting, olive like firm mass palpated on right upper quadrant

horizontal nystagmus that stops when eye is close to midline in a college student
Ménière’s disease ?

Nystagmus test
the involuntary jerking of the eyes as a person gazes to the side

eating, painful lump noted on the jaw that comes and go.
sialolithiasis).

Etopic Pregnancy: Risk Factors
Previous ectopic pregnancy
Prior fallopian tube surgery
Previous pelvic or abdominal surgery
Certain sexually transmitted infections (STIs)
Pelvic inflammatory disease
Endometriosis

cigarette smoking
age older than 35 years
history of infertility
use of assisted reproductive technology, such as in vitro fertilization (IVF)

Which among the list can cause increase in respiration
Options include (low oxygen, high oxygen, hypercapnia, hypocapnia)?

Which among the list can cause increase in respiration
hypercapnia

Osteoporosis Risk Factors (ACCESS)
A-lcohol Use
C-orticosteroid Use
C-alcium low
E-strogen low
S-moking
S-edentary lifestyle/s

ACCESS leads to OSTEOPOROSIS

to prevent fracture in a pt. with low vitamin d hydroxyl, high TSH and low Hct
VITAMIN D 600-800, CALCIUM 1000-1200.

OSTEOPOROSIS BONE ABSORPTION EXCEED BONE FORMATION.
LOW TSH= Hyperthyroidism- BONE DEMINERALIZATION.
NEED CALCIUM for BONE GROWTH

Carotid bruit
abnormal flow of blood through the carotid artery

Carotid bruit
abnormal flow of blood through the carotid artery due to atherosclerotic disease

Common causes of GERD
risk factors- alcohol, anticholinergic, CCB, chocolate peppermint, fatty, spicy, citrus foods, hormones, obesity, pregnancy, smoking, theophylline, exacerbated by CCB verapamil

Common causes of GERD
risk factors-
alcohol,
anticholinergic,
CCB, BB
chocolate peppermint, fatty, spicy, citrus foods,
hormones,
obesity,
pregnancy,
smoking,
theophylline

Zeprexa. What lab and intervention to put in place
CAUSES ELEVATED LIPDS, GLUCOSE, WEIGHT
monitor CBC for low WBC
weight- BMI q 3m
b/p, mental status, lips, prolactin, glucose

Weber test
Sensorineural loss Weber test(top of head) no laterization, normal finding, does not lateralize to either ear, bilateral hearing loss, if hear better in left ear, right sensorineural loss

Weber test
Sensorineural loss Weber test (top of head) no laterization- normal, does not lateralize to either ear- bilateral hearing loss, if hear better in left ear, right sensorineural loss.
SUN- sensorial lateralize unaffected ear
CAFFE- Conductive lateralize to affected ear

Assessment on patient with ascites
Dullness

Assessment on patient with ascites
Dullness to percussion

Varus Stress Test
application of a lateral force to the medial aspect of a joint in an attempt to create a gap in the lateral joint line, thereby testing the stability of the lateral aspect of the joint
LCL

Varus Stress Test
TEST LCL (lateral-vaRus)
McMurrays- Meniscus CLICK
application of a lateral force to the medial aspect of a joint in an attempt to create a gap in the lateral joint line, thereby testing the stability of the lateral aspect of the joint
LCL

German Measles (Rubella)
Pink, papular rash (similar to measles but paler) first appears on face, then spreads. Distinguished from measles by presence of neck lymphadenopathy and absence of Koplik spots.

patient with IOP of 32mmHg, what do you expect during fundoscopic exam

patient with IOP of 32mmHg, what do you expect during fundoscopic exam
increase cup-to-disc ratio
retinal hemorrhage
optic nerve asymmetry and pallor
measured w tonometry

BPH and urge incontinence
anticholinergics/oxybutynin, impamine/tricyclic/antidepressant

Anticholinergic- can’t think or blink, can’t see (Increase eye pressure) or pee, can’t spit or shit, SADCCUB sedation, anorexia, dry mouth confusion, constipation, urinary retention, BPH

BPH and urge incontinence
TX anticholinergics/oxybutynin,
impamine/tricyclic antidepressant

seasonal affective disorder (SAD)
a mood disorder caused by the body’s reaction to low levels of sunlight in the winter months

intussusception
telescoping obstruction of the intestines, cuts off blood supply, fatal, sudden loud crying, comes/goes, vomiting, blood/mucus mixed with stool, SAUSAGE LIKE
MASS

intussusception
telescoping obstruction of the intestines, cuts off blood supply, fatal, sudden loud crying, comes/goes, vomiting, blood/mucus mixed with stool, SAUSAGE LIKE
MASS CURRENT JELLY STOOL

IBS (irritable bowel syndrome)
An intestinal disorder causing pain in the belly, gas, diarrhea, and constipation.
due to Small intestinal bacterial overgrowth, or SIBO

IBS (irritable bowel syndrome)
SS pain in the belly, gas, diarrhea, and constipation. Pencil like stool.
Caused by: Small intestinal bacterial overgrowth, or SIBO
TX fiber, avoid gas foods, antispasmodics, decrease life stress

Osteoporosis treatment
TX first line is bisphosphonates alendronate, Fosamax, calcium500 mg, vitamin d thru food and supplementation, testosterone, wt bearing exercise

Osteopenia- increase calcium dark green vegetables, salmon, sardines, soy and OJ

Hormone (estrogen) replacement therapy (HRT) slows bone loss
Natural progesterone cream prompts new bone growth
Statins increase bone mineral density

Osteoporosis treatment
TX BIOPHOSPHATES alendronate, Fosamax, CALCIUM 500 mg, vitamin D food and supplementation, testosterone, WT BEARING EXERCISE

CAUCASIAN and ASIAN affected most
RISK PPI, STATIN, STEROIDS, THYROID,

Osteopenia- increase calcium dark green vegetables, salmon, sardines, soy and OJ

Hormone (estrogen) replacement therapy (HRT) slows bone loss
Natural progesterone cream prompts new bone growth
BONE DENSITY >2.5

Retinoblastoma
white reflection in child’s pupil

for staph aureus infection (skin) with pus
MRSA- TX Bactrim or tetracyclines?

hyperparathyroidism
high calcium

Hyperthyroidism treatment
methimazole, PTU-propylthiouracil (preferred in pregnancy)
Radioactive iodine, Beta blockers

Mammography Screening
-Age 45 – 54 yearly mammogram
-55 and older every 2 years

Fifth’s Disease (Erythema Infectiosum)
B19: lytic infection, respiratory transmission
Sx: flushed rash/fever in kids
Px: fever, get better in a week

pt has AOM but has hives on Amoxicillin and N/V with erythromycin, what meds to give
TREATMENT: Amoxicillin (first line), then Augmentin, Omnicef, Ceftin, Levaquin. If your patient is only PCN allergic do azithromycin or clarithromycin.

chlamydia in pregnancy
Azithromycin 1 gm PO x1 or Amoxicillin 500 mg PO TID x7d. Test of cure 3 weeks after completion of treatment (PREGO). EXAM

papilledema
optic disc swollen w/ blurred edges due to increased ICP EXAM

actinic keratoses
Precursor to squamous cell carcinoma. “numerous dry round and pink to red lesions” with a rough and scaly texture. Does not heal. Slow growing in sun exposed areas. Diagnosis: BIOPSY Golden Standard. Treatment: Sm. (cryotherapy), Lrg. (5-FU cream)- which causes ur skin to ooze, crust, scab, redness EXAM

Allergic Conjunctivitis
“stringy; increased tearing” PO antihistamines. Type I sensitivity. Typically bilateral. Rhinitis and allergic shiner.

COPD
COPD- Gold 1-2- SABA or SAMA ON EXAM.BASCIALLY ANTICHOLINERGIC FIRST LINE FOR COPD ON EXAM

Gold 1-2 that are poor controlled- LAMA or LABA. May use SABA for rescue.
Gold 3-4 LAMA first line. If poor use LAMA plus LABA. Alternative is LABA + ICS.
Gold 3-4- refer
SABA- Albuterol, levoalbuterol (terol)
LABA- Formeterol, salmeterol (Terol)
SAMA- Atrovent Ipatropium (tropium)
LAMA- Spiriva Tiotroium (tropium)
COPD long term is OXYGEN

CN IX Glossopharyngeal

  • Shoulder shrug/ ROMBERG test EXAM

CN V Trigeminal
Herpes. CORNEAL ABRASION. EXAM

CN VIII Vestibulocochlear
ears 8 EXAM

CN VII Facial
BELLS EXAM

ACEI contraindicated
pregnancy

Safe to give varicella/MMR
Do not give <12 mo. EXAM QUESTION

Acne Vulgaris
common acne. Retin-A, acne worsens 4-6 weeks if no improvement in 8-12 weeks increase dose or add erythromycin, benzoyl peroxide.

Acne Rosacea

  • chronic small acne like papules/pustules around nose mouth chin. TREATMENT- Metrogel, Azelex. Low dose tetracycline. Clindamycin. EXAM

Cataracts
is on EXAM in elderly night vision issues. Opaque

Kawasaki disease

  • acute high fever, enlarged lymph. BRIGHT RED RASH, conjunctivitis, dry cracked lips, strawberry tongue, Swollen hands, feet, AFTER the fever resides the rash PEELS on hands/feet. Treated with high dose aspirin and gamma globulin. This is TOXIC and VASCULAR, think blood clots, heart problems etc. Treat: high dose aspirin. EXAM

Erythema migrans
Erythema Migraines- (stage 1 Lyme) Target bulls-eye, usually appears in 7-14 days POST bitten tick. Rash is hot to touch with rough texture, flu like symptoms. DX: B. Burgdorferi via ELISA, confirm with western blot. Increased ESR. TREATMENT: Less than 7 Amoxicillin or cefuroxime axetil. Older than 7 Doxycycline. EXAM

Rocky Mountain Spotted Fever
Inc. fever, chills, N/v, photophobia, myalgia, arthralgias THEN 2-5 days later you develop a petechial rash on forearms, ankles, wrists, that spreads towards trunk and becomes generalized. Think rocky NC/OK/AK/TN/MO. DX: PCR essay with Rickessetti Antigen TREATMENT- doxycycline. EXAM

Addison’s
Addison’s- deficient in cortisol (think low sodium, blood sugar, but Increase K. You must give cortisol. (Diagnosis Plasma Cortisol <5 mcg/dl @ 0800.) EXAM

rheumatoid arthritis
Early morning stiffness, sausage joints. Symmetrical involvement. Longer stiffness than OA. Joint space narrowing. Pain, warm, tender, swollen, things. TREAT: NSAIDS, steroids, DMARDS, TNF. Only has BOUCHARDS, SWAN NECK IS DESCRIPTION ON EXAM

Osteoarthritis
Large weight bearing joints. Early morning stiffness with inactivity. Has both nodes. FIRST LINE Acetaminophen. EXERCISE: Isometric exercises for knee OA. Non-weight bearing, like biking, swimming, stationary bike. EXAM

Osteoporosis
OSTEOPOROSIS = WEIGHT BEARING- walking, lifting weights etc. bones are forced against gravity. EXAM

Fibromyalgia polymyalgia
Fibro- 11/18 points. Widespread pain for at least three months. EXAM. polymyalgia Tx prednisone 12 months. Must taper off. RISK for Temporal arteritis, DX BX, elevated ESR

Serotonin syndrome
Acute Serotonin Syndrome- Dilated pupils, high fever, muscular rigidity, mental status changes, hyperreflexes, clonus, uncontrolled shivery. You get this from SSRI, MAOIs, TCA. Could be potentially life threatening. EXAM

Fundal Height
Fundal Height 12 weeks above symphysis pubis. EXAM TOPICS
Fundus 16 weeks between symphysis pubis and umbilicus.
Fundus at 20 weeks is at umbilicus.
2 cm more of less from # of wk gestation is normal if more or less order US.

Psoriasis
Psoriasis- Inherited. Pruritic erythematous plaques, fine silvery-white scales with pitted fingernails. Scalp, elbows, knees, sacrum, intergluteal folds.
(Koebner phenomenon- new psoriatic plaques form over skin trauma)
(Auspitz sign- pinpoint bleeding when plaques are removed).
TREATMENT: Topical steroids, Tar preps (mild). For (severe) do anti-TNF, or immunologic.
D

Dacrocystitis
Darcryotosis lacrimal sac, rub down towards mouth. If think secondary infection abx. EXAM how it presents.

Acne Rosacea
Acne Rosacea- chronic small acne like papules/pustules around nose mouth chin. TREATMENT- Metrogel, Azelex. Low dose tetracycline. EXAM

Parathyroid hormone
PTH is responsible for calcium loss or gain from bones, kidneys, and GI tract. EXAM

Diabetic Retinopathy
Diabetic Retinopathy-Cotton wool spots (moderate retinopathy), micro-aneurysms. ALSO RETINAL HEMORRHAGES ON CENTER OF EYE APPEAR ORANGE RED

HTN Retinopathy
Hypertensive Retinopathy- Copper/silver wire arterioles. AV nicking(mild retinopathy). Retinal Hemorrhages. EXAM

Wilms Tumor
Wilms tumor (Nephroblastoma)- Not painful. Asymptomatic abd mass does NOT cross the midline. 2-3 y. o.d. do not palpate. Do ABD US. PUNT. Think Nephro doesn’t cross. Stays where kidney is. EXAM

Primary Amenorrhea
Primary amenorrhea: NO menarche by 15 y. with or w/o secondary sex characteristics.

Aphthous stomatitis
Cancer sores. Aphthous stomatitis: painful shallow ulcers heal 7-10 days. Magic mouthwash.

Temporal arteritis
Temporal arteritis- one temple indurated cord like gold stand. Biopsy. Abrupt visual changes blindness, inc. ESR. CPR. Most have POLYMYALGIA RHEUMATICA. Treat high dose steroids.

Atopic Dermatitis (eczema)
Inherited. Extremely itchy. On flexural folds, neck, hands. Inc. IgE. “small vesicles that rupture leaving painful, bright-red, weepy lesions” they become lichenified from itching. First line: Topical steroids. Avoid hot water/soaps. PO antihistamines. EXAM

Tinea Corporis
ring like itchy rash, slowly enlarge central clearing”-Treatment: most respond to topical antifungals, if severe do oral Lamisil. EXAM AZOLE ending

Cellulitis
Deep dermis poor demarcated low legs. EXAM/ MULTIPLE QUESTIONS. DVT RISK, DM WITH CELLULITIS WATCH FOR OSTEOMYLITIS.

Erysipelas
Group A strep, painful, Upper dermis, clear demarcated, cheeks, shins.
TREATMENT- Dicloxacillin QID x10d. Cephalexin, Clinda. PCN ALLERGY? Do Azithro x5d.
MRSA TREATMENT: Bactrim, doxy, mino, clinda. If sulfa allergy do not use Bactrim.

Varicella Zoster
“contagious 48 h. before, until all lesions crusted over” low grade fever, generalized lymphadenopathy, intense itching, erythematous macules, papules develop over macules, then vesicles erupt. “initially on trunk, then scalp and face” TREATMENT supportive, antihistamines, acyclovir 20mg/kg 5xd. If given first 24 hours works best. EXAM

Impetigo
Impetigo-Gram positive. Itchy pink-red lesions, evolve into vesiculopustules that rupture. If bullous-large blisters. Severe- Keflex, dicloxacillin. PCN Allergic-Azithro, clinda. If NO BULLAE- Bactroban. EXAM

Scarlet fever- scarlantina
“sandpaper textured-pink rash with sore throat” strawberry tongue, rash starts on head and neck, spreads to trunk. The skin THEN desquamates. EXAM

Lichen planus
LICHEN PLANUS: SMALL FLAT TOPPED, RED TO PURPLE BUMPS THAT MAY HAVE WHITE SCALES/FLAKES.. WHISPY GREY WHITE STREAKS CALLED WICHHAMS STRIAE. INNER WRISTS FOREARMS, AND ANKLES. IF ON SCALP CAUSE HAIR LOSS. Causes hep C, medications, contact with chemicals. EXAM

Spider bite
fever chills, n/v, located arms, upper legs, or the trunk. Biten area becomes swollen, red, and tender, and blisters appear within 24-48 hours. Necrotic in center, which kills the tissue. Ice packs to wound and cold inactivates the toxin, tx like cellulitis of the skin, abx ointment at first, watch etc. Exam

Pityoris rosea
Pityoris rosea itchy, herald patch, xmas tree pattern, rash hands soles/feet think to test for secondary syphilis RPR then VDRL are screening, then dx FTA-ABS. EXAM

Corneal abrasion
Corneal Abrasions- Round/Irregular. Was on EXAM.

Acute Angle Closure Glaucoma
acute/severe halos, cupping optic nerve, cloudy cornea, mid-dilated oval pupil. ER STAT. EXAM

Conductive
Conductive: Lateralization to bad ear. Rinne- BC > AC.
Rinne (1st mastoid, 2 front of ear, time each area).
Weber: Tunning fork midline. CN 8 (acoustic). EXAM

Koplik spots
Koplik Spots- “clusters sm. Size red papules w/ white centers in the buccal mucosa by lower molars”. Rubeolla. Fever, conjunctivitis, coryza, cough (3c). Morbiliform rash. EXAM

Sensorineural
Sensorineural: Lateralization to good ear. Rinne- AC > BC.

OME
Ear pressure, popping, muffled hearing, chronic allergic rhinitis, sterile serious fluid is trapped in the middle ear. TM should NOT BED RED. TM may bulge or retract. TREATMENT: Oral decongestants, steroid nasal spray, treat like allergies. Usually Painless. Weber- Lateralization to affected ear. Rhinne- BC > AC. PRECEDES OR USUALLY FOLLOWS AOM. SUPPORTIVE CARE AND WAIT 3 MOS SOMEX. EXAM

Presbycusis
sensorineural loss without lateralization. Involves the inner ear. Symmetrical progressive. Human speech lost first. AGING ADULT EXAM

OE
Otitis Externa (swimmers ear)- Pseudomonas aeruginosa. (other- S. aureus). External ear pain- d/c itching, hearing loss, tragus, green d/c. TREATMENT: Corticosporin, Cipro EXAM

Sinusitis
TX AMOXICILLIN OR AUGMENTIN ALLERGY MACROLIDE

Meiniers disease
VERTIGO TINNITUS, HEARING LOSS. nystagmas

Mono
test heterophile antibody test. ON EXAM

MR. ASS
(Systolic Murmur) Only systolic murmurs will radiate to a location on the exam.

Mitral Regurg
(Holo/pansystolic)- radiates to axilla. Think Mitral area 5th ics MCL.
Aortic Stenosis (mid systolic ejection) radiates to neck. Think 2ics rsb.

All diastolic murmurs are pathological. Grades Murmurs
I-barely II-audible III- clearly audible. IV- first time thrill V-Steth edge VI-entire steth. EXAM

MVP
MVP- S2 click, followed by systolic murmur. Asymptomatic. MVP with palpitations is treated with BB. LATE SYSTOLIC.

S3- HF,
S4-LVH stiffening,
S3- HF, Kentucky, early diastole. Abn >35. Bell EXAM
S4-LVH stiffening, Tennesse, late diastole. “Atrial kick/gallop” EXAM

Isolated Systolic HTN
CCB

PAD/ PVD
PAD/ PVD (same)- Nocturnal pain relieved by lowering legs, poor pulses, dependent rubor, intermittent claudication, atrophy, shiny, hairless, cold feet. Initial do a pulse check, ABI 0.9 or less is PAD. Ateriography is the most DEFINITIVE test. Try to develop collateral circulation. Otherwise- Trental, Pletal. EXAM

CVI
CVI- Impaired venous return. Achy legs relieved by elevation, edema after prolonged standing, night cramps, brownish discoloration, cold, ulcers. Etc. do support stockings. EXAM

blood pressure
BP – ST 1 (140-159/ 90-99), if you know this you will get the rest!! Normal is <120/80. ELERGLY OVER 60 150/90 IS OK. ISH WILL INCREASE SYSTOLIC NOT DIASTOLIC. ON EXAM.

Thiazide diuretics
no sulfa allergies, hyperuricemia, hypokalemia, hypomagnesia, hyponatremia, hyperglycemia, hypertriglycerides. ON EXAM

Statin
Must check LFT before starting Statin. Know when to start statins and what to check for to decide mod-high dose statins. ON EXAM

Pulses paradoxus
Pulsus paradox Apical pulse can still be heard even though the radial pulse is no longer palpable. Certain issues cause impairment with diastolic filling, 10 or greater drop in the SYSTOLIC pressure. I think her patient had asthma and their pressure dropped by 10 etc. ON EXAM

Emphysema
Emphysema Lungs- Percussion-HYPERENNOSANCE tactile frem + egophony- dec. CXR- flattened diaphragms with hyperinflation. Inc. AP diameter, accessory muscles, pursed-lip breathing, weight loss. ON EXAM

Acute Bacterial Pneumonia- CXR
middle lobe. ON EXAM

OSA
does not include Microglossia which is an absent tongue congenital. EXAM

TB
fatigue, fever, cough. Never do fewer than 3-4 drugs initially if positive, then u can narrow it down. Latent TB usually treated with INH. If u suspect ACTIVE TB order, NAAT, C&S, AFB. The AFB is not diagnostic. SPUTUM FOR C & S if gold standard. Deep morning cough collected for three “consecutive days”. TB is usually upper lobes.

TPO
TPO- this lab is off MEANING ELEVATED in BOTH hyper/hypo thyroidism. TPO is GOLD stand for diagnosis in Hashimotos. But you always want to order a TSH first, THEN ur thyroid panel do not get ahead of yourself. Check ur TSH lab on both in 6-8 weeks but never sooner than 6 weeks that is how long these meds take to work. TOPIC ON EXAM

hyperthyroid
Hyperthyroid- Low TSH, high “FREE” T4/T3. ALWAYS DO FREEs. Graves disease-autoimmune. Lid lag, exophthalmos, everything is hyper (body wise). Treatment: PTU/Tapazole. PTU PREFER IN PREGNANCY
RAIU-no w/ prego. Destroys thyroid, lifelong treatment for hypo then.

A1C > 9
If you are already on TWO oral drugs for diabetes and A1c is 9 or higher, start BASAL insulin. If you cannot tolerate metformin and your A1c is 9 or higher start BASAL insulin. ON EXAM

Parathyroid
For parathyroid- dx blood test. You will have elevated calcium because your parathyroid is releasing too much from bones and this will just cause it to float around and not help ur bones. TX: BIPHOSPHANATES FOR SECONDARY HYPERPARATHY. EXAM

cushings
Central obesity, moon face, purple striae, hairy, hypertension, elevated plasma CORTISOL in AM. “INC BS, SODIUM” Dec K. You must draw cortisol levels in the morning.

Fructosamine test
checks sugar for past 2-4 weeks.

triglycerides
causes pancreatitis >500. If >500 treat with Niacin or Fibrate or Niaspan. If your patient is already on NIACIN you can add a fibrate like (LOPID/TRICOR). Apparently an insulin infusion works also. ON EXAM

Pancreatitis
diagnosed with amylase / lipase draw. Amylase beings 2-12 h. Lipase 4-8 hours. Lipase however is MORE specific and sensitive to alcoholic pancreatitis. ACUTE: Grey Turner/ Cullen sign. Abd pain that rates to midback “boring” epigastric pain. Fever, n/v. EXAM TOPICS

Metformin
Metformin – monitor BUN, Creatinine.
contraindicated renal and liver disease
ARF creatinine up and GFR down.
affects liver enzymes, weight loss, avoid in alcohol drinkers, lactic acidosis, diarrhea flatulence
CT: hold 24 hr before and 48hr after

Polycythemia vera
slow growing blood cancer. blood too thick, clots. bone marrow to many RBC. risk bleed , anemia, CBC

CCB
BLE edema- walk around

Kava Kava
anxiety and insomnia, don’t mix w sedating benzos.

Bipolar med
lithium- monitor TSH, toxicity bind to TH cause hypothyroidism.

Depression med
acute Anxiety/ panic med
GAD med
depression SSRI
panic benzo, Xanax, ativan
GAD SSRI
SNRI- Buspar, Effexor, Cymbalta- Taper OFF

SNRI/ Benzo Discontinuation
Buspar, Effexor, Cymbalta and Benzo Taper OFF

Thiazide GLUT-
Glyceridemia
Lipidemia
Uricacidemia
Triglyceridemia
HYPO-Kalemia

ACE/ARB contraindications
pregnancy
Renal failure
Renal Stenosis

METABOLIC SYNDROME
METABOLIC SYNDROME
NO THIAZIDES,CCB (HF)
NO TZD ACTOS (Pioglitazone)

GIVE Metformin, ACE or ARB- kidney protective, BB- causes hypoglycemia,

Triglyceride < 300 what do we do first?
Triglyceride in 300 plus risk for _ what do we do?
Lifestyle modification
Pancreatitis, Niacin then add, fenofibrates

Serotonin syndrome ss
hyper rigidity, fever, myoclonus, dilated pupils, AMS, hyper reflexes, from SSRI or mix w MAOI and TCA

NSAID contraindicated in
HF and ARF
increase BP impair renal prostaglandin and sodium retention

strabismus
misalignment of eyes, abnormal after 6 months

macular degeneration
loss of central vision

retinal detachment
floaters, curtain, flashes

Xanthelasmas
sharply demarcated yellowish deposit of cholesterol underneath the eye

Red reflex absent
retinoblastoma (leukorrhea) , cataracts, glaucoma.
Will have white reflex

Mono, return to play and Dx
4-6 weeks when spleenolmegaly resolves
Mono spot/ Heterophile

Viral stomatitis
ulcer on cheek
(Aphthous)- viral canker sore

cholesteatoma
chronic sinusitis or OM. cauliflower, foul-smell, hearing loss.
erodes bones in face affects facial CN 7.
Benign- risk hearing loss refer SURGERY

Acute Closed Angle Glaucoma
Vs
Open Angle
Acute: Sudden pain, halos, cupping, dilated, cloudy , IOP, HA, refer ED

Open (primary) : CN 2, gradual loss peripheral vision first

Papilledema
Swollen, optic disc, increase cup to disc ratio, HA, ICP, HTN,

Acyclovir (cheapest)
200mg 5 x day

Pt on PPI, has osteoporosis has a cough,
refer for EGD r/o Barrett’s.

allergic rhinitis
blue pale turbinate clear drainage. Tx inhaled corticosteroids

hordeolum Vs
chalazion
blepharitis
hordeolum: painful swollen red warm abscess TREAT hot compress erythromycin, dicloxacillin.
chalazion does not hurt
Blepharitis always unilateral, Tx baby shampoo warm compress

Sialolithiaisis
painful lump, calculi or salivary stones.
sub mandibular gland whartons; duct.

Roseola infantum- Sixth disease
viral, young children, high Fever 3-4 days followed by maculopapular rash

Herpetic keratitis
ocular herpes – blurr vision, inflammation of eye; gritty feeling, conjunctivitis, sharp pain, and photophobia- AVOID SUN
REFER OPTHO Tx acyclovir

Cranial nerves responsible for extraoculomotor movements?
a. 2, 3, 6?
b. 3, 4, 6?

Patient with hx of hypertension and stroke, now having memory loss and confusion – indicates what?
a. Alzheimer’s
b. Vascular Dementia?

CAGE acronym
?

Treatment for chronic alcoholism?
?

Frail elderly mammogram breast tissue?
?

Patient has Barrett’s esophagus, insurance no longer covers gastroenterologist who was treating condition, patient at the FNP office, wanting a prescription for medication… What should the FNP do?
a. Refer to oncologist?
b. Refill prescription?

GERD treatment?
a. PPI? Per American College of Gastroenterology, treatment should begin with a PPI.
b. H2? An H2 is inferior to PPI’s.

3 months on Symmetrel, TSH increased, T4 normal, what do you do?
a. Increase medication?
b. Decrease medication?

Lipid level of 1500, increased risk for?
a. pancreatitis?

Frail elder, increased creatinine, indication?
?

Fingernail hematoma treatment?
a. drill hole and drain blood?

Abnormal cells on PAP, what do you do next?
a. Refer for colposcopy?

Red beefy tongue?
a. Pernicious Anemia?
b. Iron Deficiency?
c. Folate Deficiency?

Low Hemoglobin, Low Hematocrit, High MCV – Indicates what?
?

Koplick Spots?
a. Measles?

Signs and Symptoms of Roseola?

Pregnant teacher, with exposure to Fifths’ Disease – what risk is there for the fetus?

6 Month Old with Closed Anterior Fontanel?

3 Month Old infant with Down Syndrome, due to milk intolerance, mom started infant on Goat’s Milk; now has pale conjunctiva but otherwise healthy; with low Hematocrit, what other test would you order?
a. CBC
b. Iron, TIBC

4 Month Old with Strabismus, mom worried, what do you tell her?
a. Normal?

4 Month Old “won’t keep anything down”, what is the main thing you look for?
a. Dehydration?

Man with HTN, CAD, present femoral pulses, but absent pedal pulses?
a. DVT?
b. Venous Insufficiency?
c. Arterial Insufficiency?

Diabetic patient with foot laceration, at risk for what?
a. foot ulcer?
b. acute osteomyelitis

Definition of metabolic syndrome?

Treatment for Gonorrhea?
a. Rocephin IM and Zithromax po
b. Doxycycline po

Diagnose trich?
a. Wet Prep?
b. KOH?

Patient with history of PID, increase risk for?
a. Infertility?

Increased risk for ectopic pregnancy, history of…?
a. Salpingitis

Newborn with foot turned in (“toeing in”), what do you do?
a. Refer immediately to orthopedist?
b. Routine followup?

Osgood Schlatter Disease pain location?

Growth plate fracture (Salter-Harris fx) location and pain?

Rotator Cuff injury presentation?

Causes of tachycardia?
a. Fever?
b. Anemia?
c. Hypotension?

Definitive diagnosis of acute bacterial proctatitis?
a. Urinalysis and Culture
b. Vigorous Massage of Prostate to Release Fluid for Culture?
c. Urethral Culture?

Man with high BPH, perform digital rectal exam – how does prostate feel?
a. Asymmetrical, Nodular, Firm
b. Symmetrical, Boggy
?

Patient present with “bag of worms”, what does this indicate?
a. Varicocele

Legg-Calve-Perthes disease?
Avascular necrosis of the proximal femoral head…

McMurray’s Sign

Epitaxis is most common in the area of the nose known as Kiesselbach’s Triangle, where is this located?
a. anterior septum?
b. middle turbinate?
c. sinus turbinate?

Elderly presents with atrophic vaginitis, small uterus, and palpable 4×5 ovary, what do you do?
a. Pelvic ultrasound?

Grade 3 cells on Pap, treatment?
?

Patient with bleeding after menopause?
a. Endometrial biopsy?

Young adult female, wants birth control, forgets to take pills, doesn’t want to get pregnant for at least 5 years, what do you suggest?
a. Intrauterine Device?

Shingles near eye, patient wants cream and analgesic, what do you do?
a. Order Acyclovir cream and analgesic?
b. Immediate referral to opthamologist?

Patient presents with rash on shoulder, erythematous maculopapular rash with center clearing, and scaling?
a. Tinea Pedis?
b. Tinea Corporis?
c. Psoriasis?

Peripheral vision loss = ?
Central vision loss = ?

Patient with Atopic Dermatitis, look for what other diseases?
a. Asthma?

Patient forgot to start Thanksgiving dinner, and husband states that she has trouble remembering tasks and has trouble with organization… What is this indicative of?
a. Alzheimer’s?
b. Delerium?

Quick assessment of patients fall risk?
a. Timed Get up and Go?

Mitral Regurgitation = sound and heard best at?
Mitral Stenosis = ” ” ” ” ” “?
Mitral Prolapse = ” ” ” ” ” “?
Mitral Sclerosis = ” ” ” ” ” “?
Aortic Stenois = ” ” ” ” ” “?
Aortic Regurgitation = ” ” ” ” ” “?
Aortic Sclerosis = ” ” ” ” ” “?

3 ways to assess cognitive function in patient with signs/symptoms of memory loss – all but which of the following?
a. Mini Mental
b. Depression Screen
c.
d.

Patient voices aching when ambulating? This question did not specify if it was bilateral or unilateral Mentioned something about “rubor” but I can’t remember what it said exactly…
a. DVT?
b. Venous Insufficiency?
c. Arterial Insufficiency?

Patient with HIV took high-potency anti-viral treatments and CD4 is >400, what does this indicate?
a. Patient has full-blown AIDS as defined by the CDC?
?

Patient has following labs: listed serology for Hepatitis, and then had to know if they had an active infection or immune, etc…

how long will a cough last for acute bronchitis?
up to three weeks is completely normal

how do you treat acute bronchitis?
95% cases are viral– tessalon pearls to help with cough. * prednisone is never the answer **

explain the following views of chest xrays: AP, PA, PA and lateral
which one do you want for pneumonia patient?
AP- shows the heart predominantly because its from front to back
PA- shows the lungs predominantly because its back to front
lateral shows fluid line

what is the gold standard for diagnosing community acquired pneumonia
chest xray- PA/lateral, repeat post treatment in 6 weeks (repeat not really done anymore)

treatment guidelines for CAP- healthy adult no comorbidities
MAD LUNG
amoxicillin 1 gram TID (best choice)
doxycycline 100 mg twice daily
macrolide (mycin) in areas with low macrolide resistance so not the best choice

treatment guidelines for CAP- adult with comorbidities
monotherapy: respiratory quinolone (levofloxacin 750 mg daily)
combination therapy: augmentin or cephalosporin PLUS macrolide or doxy

what are common respiratory fluroquinolones?
Think FLOXACIN– moxifloxacin, gemifloxacin, levofloxacin

what are common macrolides for CAP
think “MYCIN” — azithromycin, clarithromycin

what are common tetracyclines for CAP
doxycycline

what are common beta lactams for CAP
amoxicillin, augmentin (amox with clav), cefpodoxime, cefuroxime

how would you treat a pregnant 29 year old with pneumonia?
Amoxicillin 1 g TID– cant give levaquine (flouroquinolones no in pregnancy due to fetal ligament destruction, doxycycline no in pregnancy stains tooth bones in fetus)

how long do you give antibiotics to patient with pneumonia?
5-10 days, you always want to continue antibiotics for 3 more days after clinically stable (no fever)

When do you get a chest xray for pneumonia?
it is the gold standard for diagnosis, you do not need to do resolution chest xray unless things are lingering or you suspect something else.

When do you give pneumococcal vaccine?
adults > 65 years old you give PPSV23 or you can give both PCV 13 and PPSV23 but must be 1 year apart

adults > 65 with immunocompromising condition give both pcv 13 and ppsv23

adults 19-64 at increased risk of pneumococcal disease (asthma, copd, smokers, cv dz) – give PPSV23 only

adults 19-64 with asplenia, cochlear implants, csf leak– give pCV13 NOW then PPSV23 in 8 weeks , then PPSV23 in 5 years.

What is the diagnostic criterion for COPD
FEV1/FVC ratio of < 0.70

What are characteristics of COPD
midlife onset, symptoms slowly progressive, exposure to lung irritant, DOE is progressive, chronic cough and chronic sputum production

what are characteristics of asthma
onset early in life, symptoms vary widely from day to day, symptoms worse at nighttime/early AM, allergic rhinitis, eczema, family history, obesity

what are characteristics of heart failure (when evaluating diff dx of COPD)
chest xray with dilated heart, pulmonary edema

what are characteristics of tuberculosis when ruling out for COPD patient
onset all ages, chest xray with lung infiltrate, microbiologic confirmation

what are the characteristics of bronchiectasis
LOTS OF PURULENT SPUTUM, chest xray shows bronchial wall thickening and bronchial dilation

What intervention has the greatest influence on slowing progression of a COPD patient?
smoking cessation!!

what are first line smoking cessation drugs? (3)
varenicline (chantix), nicotine patch, buproprion extended release (zyban)

what is second line smoking cessation drug?
sertraline (zoloft)

what smoking cessation drug would you give to someone with a history of a suicide attempt?
nicotine patch– NOT CHANTIX OR ZYBAN

What are the short acting beta agonists
albuterol, levalbuterol

what are the long acting beta agonists
“terols”
Salmeterol
Formoterol

How do inhaled anticholinergic drugs work?
used in COPD, prevent bronchoconstriction

how do beta agonists work?
stimulate beta 1 (tachycardia) and beta 2 (bronchodilation)

what is the suffix for inhaled anti-cholinergic drugs?
“tropium” think ipratropium (atrovent) SHORT ACTING, tiotropium (spiriva) long acting

What is a SAMA?
short acting antimuscarinic (anticholinergic) – Ipratropium

what is a LAMA?
long acting muscarinic antagonist (anticholinergic) Tiotropium (spiriva)

what are the 4 steps of GOLD guidelines
1) SABA or SAMA prn
2) LABA or LAMA PLUS rescue med
3) ICS + LABA/LAMA plus rescue med
4) ICS+ LABA AND LAMA plus rescue med
THEN REFER

How to manage COPD exacerbation?
figure out the cause
mild: sabas with spacers / nebulizer
moderate: SABAs plus AB and or steroid
Severe: hospitalization

right supraventricular nodes signal what malignancies?
lungs, mediastinum, esophagus

left supraventricular nodes signal what malignancies?
abnormal (stomach, GB, liver, pancreas, ovaries, prostate)

us preventive services task force routine lung cancer screening recommends what annual screening for current smokers aged 55-80 with 30 pack year history or have quit within the last 15 years?
annual low dose CT (LDCT)

how do you diagnose asthma > 5 years of age:
presence of asthma features
reversible airway obstruction on spirometry

12% improvement from baseline or FEV1 > 200 ml after SABA

How do you treat asthma?
saba always as rescue inhaler
Step wise approach (GINA)
Low dose ICS
then ICS + LABA
increase dosage strength

acute bronchitis classic case symptoms
cough that keeps awake at night
dry cough but may be productive
low grade fever or chest pain with cough
wheezing and rhonchi
median duration of cough is 18 days up to 3 weeks
history of a cold before onset of symptoms

objective findings in acute bronchitis
lungs: clear to severe wheezing, rhonchi
percussion: resonant
CXR: normal
afebrile to low grade fever

treatment plan for bronchitis
symptomatic treatment- increas fluids and rest
tessalon perles, expectorant / mucolytic (guaifenesin)
ventolin (albuterol) for wheezing
for severe wheezing consider short term oral steroid

complications of acute bronchitis
exacerbation of asthma
pneumonia from secondary bacterial infection

pertussis “whooping cough”
caused by bordetella pertussis (gram negative)
coughing illness at least 14 days
paroxysmal coughing, inspiratory whooping
neonates / infants at highest risk for death

three stages of pertussis
catarrhal 1-2 weeks, if treated at this stage can shorten disease course
paroxysmal: lasts 2-4 weeks, treatment has little influence but is useful to decrease spread
convalescent: treatment goal is to eradicate carriage state / disease spread

how to diagnose pertussis
nasopharyngeal swab for culture – collect at 0-2 weeks following cough
PCR may provide accurate results up to 4 weeks
Pertussis antibodies by ELISA
CBC with elevated WBCs and marked lymphocytosis
cxr should be negative

how to treat pertussis
administer a course of antibiotics to close contacts iwthin 3 weeks of exposure
first line: macrolides – azithromycin z pack x 5 days
alternative bactrim x 14 days
antitussives, mucolytics, rest, hydration, frequent small meals

complications of pertussis
sinusitis, otitis media, pneumonia, fainting, rib fractures from coughing

signs of pulmonary embolism
new onset dyspnea, hemoptysis, pleuritic chest pain, vital signs with tachycardia, tachypnea, may have signs of DVT

Hegar’s sign
Softening of lower uterine segment

Chadwick’s sign
Bluish color of cervix and vagina at 6-8 weeks

Goodell’s sign
Softening of cervix at 4+ weeks

Lipids in chronic inactivity
Low HDL

Lipids in under- or untreated hypothyroidism
Elevated total cholesterol, TG, and LDL

Lipids in chronic renal insufficiency
Elevated total cholesterol and TG

Lipids in alcohol abuse
Elevated TG, HDL, and LDL

Vaccines with live attenuated viruses
MMR, varicella, flu-mist(intranasal), Zostavax

Drugs of choice for abdominal infection
Beta lactams + metronidazole (most common); fluoroquinolones

Drugs of choice for urinary tract infection
Fluoroquinolones (except moxi), TMP-SMX, fluconazole, b-lactams, nitrofurantoin, fosfomycin if pt has many allergies

Drugs of choice for pulmonary infections
Macrolides, resp fluoroquinolones, b-lactams, doxycycline (MRSA, atypical coverage)

Drugs of choice for skin and soft tissue infections
MRSA: TMP-SMZ, doxycycline, clindamycin; b-lactams

Drugs of choice for MRSA
Vancomycin, daptomycin , linezolid, TMP-SMZ, doxycycline, ceftaroline

Macrolide drugs
Erythromycin, clarithromycin, azithromycin

Macrolide AEs
GI, QT prolongation

Tetracycline uses
MRSA, rocky mtn spotted fever, atypicals (mycoplasma pneumonia, chlamydia), spirochetes (Lyme), h.pylori

Tetracycline AEs
GI, teeth, hepatic dysfunction, photosensitivity

Fluoroquinolone drugs
Ciprofloxacin, levofloxacin, moxifloxacin

Fluoroquinolone uses
UTI (cipro or levo), atypicals (mycoplasma, legionella)

Fluoroquinolones and dairy or vitamins
Take 2 hrs before or 4 hours after (decrease concentrations)

Metronidazole (Flagyl) uses
C. Diff, vaginitis

Metronidazole AEs
GI, peripheral neuropathy, disulfiram reaction

Nitrofurantoin uses
UTI -not pyelonephritis due to no systemic absorption

Nitrofurantoin AEs, contraindications
Pulmonary fibrosis, contraindicated in CrCl <60 (excludes many elderly)

Aminoglycoside drugs
Gentamycin, tobramycin

Aminoglycoside uses
Atypical coverage, resistant infections, tobramycin for CF only

Aminoglycoside AEs
Ototoxicity (irreversible), nephrotoxicity (reversible)

Aminoglycoside monitoring
Renal dose adjust, hearing test, drug level monitoring

Antifungals (azoles)
Fluconazole, itraconazole

Itraconazole interactions
Acid suppressive therapies

Fluconazole dose adjust
Renal

Antifungal AEs
GI, QTc prolongation (fluconazole)

Antifungal drug interactions
QT prolongation meds, warfarin

Anti-infectives not for pregnant
Tetracyclines, TMP-SMZ

TMP-SMZ coverage
G+, G-, MRSA, Protozoa (toxoplasma gondii), fungus (pneumocystis jirovecii), poor anaerobic activity

TMP-SMZ AEs
Sulfa allergy, photosensitivity, hematologic (anemia, leukopenia, thrombocytopenia)

TMP-SMZ on renal function
Dose adjust in renal impairment, false elevation in serum Cr

TMP-SMZ drug interactions
Warfarin

Gram + only coverage
Clindamycin, linezolid, vancomycin, daptomycin

Gram – only
Monobactams (aztreonam), penicillin VK

Gram +/-
PCN (aminopcn, ext. spectrum), carbapenams, aminoglycosides, TMP-SMZ, fluoroquinolones, fosfomycin

Head growth in first year of life
Total 12cm

Head growth 0-3mo
6cm

Head growth 4-6mo
3cm

Head growth 6-12mo
3cm

Head growth 2-7yo
0.5cm/yr

Head growth 8-12yo
0.3cm/yr

Tumor grading: T0
No evidence of primary tumor

Tumor grading: T1
2 cm or less in greatest dimension

Tumor grading: T2

2-5cm

Tumor grading: T3

5cm

Chvostek sign
Hypocalcemia, spasm when tap facial nerve

Trousseau sign
Hypocalcemia, spasm when compress brachial artery w/ BP cuff

Myocardial ischemia EKG changes
Inverted T wave, T wave depression

Myocardial injury EKG changes,
ST segment elevation, tall peaked t wave

Myocardial infarction EKG changes
Q wave

1st gen antihistamines
Diphenhydramine, chlorpheniramine

2nd gen antihistamines
Loratadine, desloratadine, cetirizine, fexofenadine, levocetirazine

Ishihara chart
Test for color blindness

Scarlet fever presentation
Sandpaper-like rash, exudative pharyngitis, localized anterior cervical lymphadenopathy, rash 2 days after pharyngitis then peels later

Roseola (HHV-6) presentation
Rash lasts hours to 3 days, follows 3-7 day high fever, sometimes febrile seizures, tx supportive

Rubella (German measles, rubella virus)
Mild sx, posterior and postauricular cervical lymphadenopathy, arthralgia 5-10 days before rash, teratogenic, contagious 1 week before and 2 weeks after rash

Measles (rubeola)
Acute presentation, generalized lymphadenopathy, photophobia, koplik spots, pharyngitis mild w/o exudate, rash 3-4 days after sx, contagious 1 wk before and 2-3 weeks after rash, permanent neuro impairment and/or death possible, tx supportive

Hand foot mouth dz (coxsackievirus a16)
Fever, malaise, sore mouth, anorexia, lesions, fecal-oral or droplet, highly contagious 2-6 wks, tx supportive

Fifth disease (human parvovirus b19)
Rash starts on face (slapped cheek) spreads to trunk, extremities, contagious before rash but not during or after. Droplet. Leukopenia. Hydrops fetalis in pregnant. Tx supportive.

Infectious mononucleosis (Epstein-Barr)
Diffuse lymphadenopathy, hepatic and splenic tenderness/ enlargement, incubation 20-50 days, systemic corticosteroids if resp distress from swollen airway, monospot/leukopenia w/ atypical lymphocytes diagnostic. Avoid contact sports at least one month.

Kawasaki disease
Acute phase: fever >= 104 >= 5 days, polymorphous exanthem on trunk/flexor region/perineum, erythema of oral cavity (strawberry tongue), chapped lips, bilateral conjunctivitis w/o discharge, edema/erythema of hands and feet w peeling skin. Children 1-8yr. Tx IV immunoglobulin, pop aspirin, consult

Lyme stage 1 (early localized disease)
Mild flu-like illness, singular annular lesion (erythema migrans), sx resolve 3-4 wks w/o tx

Lyme stage 2 (early disseminated)
Months after initial infx, classic rash may reappear w/ multiple lesions. Arthralgias, myalgia, HA, fatigue. Less common: heart block, neuro findings (Bell’s palsy)

Lyme stage 3 (late persistent)
1 year after initial infection, msk s/s persist w/ joint pain, frank arthritis, joint damage. Neuropsychiatric findings – memory problems, depression, neuropathy.

Lyme organism
B. burgdorferri (tick-transmitted spirochete)

Lyme testing
ELISA confirmed with western blot

Lyme tx
Doxycycline (children >8yo), amoxicillin, cefuroxime – adults and children

Chancroid tx
Azithromycin or ceftriaxone, alt. ciprofloxacin or erythromycin

Genital herpes (HSV 2) tx
Acyclovir

Lymphogranuloma vereneum (c. trachomatis) tx
Doxycycline, or erythromycin

Nongonococcal urethritis/cervicitis (c. Trachomatis) tx
Azithromycin or doxycycline

Gonococcal urethritis/cervicitis tx
Cefixime or ceftriaxone, add azithromycin or doxy if chlamydia not ruled out

Bacterial vaginosis tx
Metronidazole

Pelvic inflammatory disease tx
Ceftriaxone+doxycycline +/- metronidazole

Trichomoniasis tx
Metronidazole

ADA screening for DM in Children
-symptomatic children (polyuria, polydipsia, polyphagia, blurred vision) regardless of risk factors
-asymptomatic children after puberty or 10 years of age or older if overweight or obese (>85th percentile). Plus 1 of the following:
*T2DM in 1st or 2nd degree relative
*high risk racial/ethnic group
*signs of insulin resistance (HTN, dyslipidemia, acanthosis nigricans, PCOS, SGA)
*maternal hx of DM or GDM during the child’s gestation

Statistics

  • leading causes of death: Heart disease, cancer, lung disease
  • leading cause of cancer death: lung
  • leading cause of death in adolescents: accidents
  • most common cancer: skin.
  • in males: prostate. in females: breast

suicide: males more successful, women more attempts. highest rate is older white males.

Osgood-Schlatter:
knee pain in young adults, overuse. Repetitive stress pain, tenderness, swelling at the tendon’s insertion site. The tibial tuberosity. Rule out avulsion fracture if there is an acute onset and order a lateral xray. RICE. Usually stops when the growth stops.

If patient has right sided weakness, etc. the CVA occurred where
left side

initial evaluation of symptoms of acute prostatitis
Urinalysis and urine culture

A 65-year-old woman presents for a follow-up examination after a new patient visit. She has not seen a healthcare provider for several years. She is a smoker and her hypertension is now adequately controlled with medication. Her mother died at age 40 from a heart attack. The fasting lipid profile shows cholesterol = 240 mg/dL, HDL = 30, and LDL = 200. In addition to starting Therapeutic Lifestyle Changes, the nurse practitioner should start the patient on:

  1. bile acid sequestrant.
  2. a statin drug.
  3. a cholesterol absorption inhibitor.
  4. low-dose aspirin.
    A statin drug

Ortolani’s Click
a click is heard or felt as dislocation is reduced (developmental dysplasia of hip) (good until one year)

Which of the following laboratory tests should a nurse practitioner order when the suspected diagnosis is temporal arteritis?
Erythrocyte sedimentation rate (ESR)

What are narrow therapeutic index drugs?

  1. Warfarin sodium (Coumadin): monitor INR
  2. Digoxin (lanoxin): monitor digoxin level, EKG, electrolytes(potassium, magnesium, calcium)
  3. Theophylline: monitor blood levels
  4. Carbamazepime (Tegretol) and Phenytoin (Dilantin): Monitor blood levels
  5. Levothyroxine: Monitor TSH
  6. Lithium: Monitor blood levels, TSH (risk of hypothyroidism)

Otitis Externa tx
Fluoroquinolone & Polymyxin B cortisporin drops

An elderly male patient complains of a new-onset, left-sided temporal headache accompanied by scalp tenderness and indurated temporal artery. The NP suspects temporal arteritis. What screening test would you order to assist with diagnosis?
sedimentation rate (expect to be very elevated)

Basal Cell Carcinoma
Pearly domed nodule with overlaying telangiectatic vessels. Could be plaque, papule, possible central ulceration and crusting. Dx: Biopsy Tx:

Normal, healthy woman of reproductive age
white, clear, flocculent(physiologic leukorrhea), no complaints, pH 3.8-4.2 (toward acidic), no odor, microscopic shows lactobacilli (gram+bacteria)

Multiple infections from bacteria and fungus?
Screen for HIV

Screening Tests

  • sensitivity: detect those WITH the disease. higher the sensitivity is higher the false positives
  • Specificty: detect those who DONT have the disease.

erythromycin for chlamydia eye infection in infants

to assess pts ability to think abstractly a nurse pract could ask the patient
the meaning of a common proverb

The most commonly prescribed medication for mild systemic lupus erythematosus (SLE) is:

1.
azathioprine (AZA).
2.
belimumab (Benlysta).
3.
ibuprofen (Advil).
4.
cyclophosphamide (Cytoxan).
ibuprofen (advil)

A 17-year-old female is suspected of having polycystic ovary syndrome. In addition to testosterone, the most appropriate diagnostic tests to order would be:
follicle-stimulating hormone (FSH), luteinizing hormone (LH), prolactin, and thyroid-stimulating hormone (TSH).

Barlow’s Maneuver
Feeling of a slip as the femoral head slips away from the acetabulum (toward the butt) (good until 6 mo)

Candida vulvovaginitis
etiology: candida albican (80-90%)
white, curdy, “cottage-cheese” like, sometimes increased, itching/burning discharge, pH <4.5, odor is usually absent, microscopic shows mycelia, budding yeast, pseudohyphae w/KOH prep. Treatment: oral diflucan or vaginal miconazole or terconazole

PSEUDOHYPHAE, CLOTRIMAZOLE CREAM

Proton Inhibitors
Increased risk of fractures(postmenopausal women),
Pneumonia, Clostridium difficile infection, hypomagnesemia, B12 and iron malabsorption, atrophic gastritis, and kidney disease

Bacterial Conjunctivitis tx
Eye drops or ointment: Polytrim, trimethoprim, polymyxin, macrolide

A patient with an elevated WBC (>11k) accompanied by neutrophilia (>70%) and the presence of bands is what kind of shift and prognosis?
-Shift to the left
-Serious bacterial infection

Actinic Keratosis
Rough flat, dry crusty, erythematous papules or plaques. Scaly patch of red brown skin caused by years of SUN exposure. Precursor to squamous cell carcinoma. Dx. Biopsy. Tx: topical 5 fluroracil 5-FU, cryotherapy.

A patient with macular degeneration has deficit vision in?
Central vision

Dacyrocystitis
Typical symptoms of acute tear duct infection include:
Pain, redness and swelling of the lower eyelid at the inner corner of the eye
Excessive tearing
Pus or discharge from the eye
Fever
Tx:
lacrimal sac massage- rub down towards mouth.
oral clindamycin (topical tobramycin or moxifloxacin if mild-purulent drainage, no redness)

Pre-DM in children values
A1C: 5.7% to 6.4%
Fasting: 100-125
2 hour GTT: 140-199

Primary Prevention

  • individual actions: eating nutritious diet, exercise, seatbelts, gun safety. IMMUNIZATIONS

squamous epithelial cells with stippling appearance, no lactobacilli and many WBCs is
wet mount that shows BV

trigeminal neuralgia manifests
electric shock facial pain

The most common sign of cervical cancer is:

1.
postcoital bleeding.
2.
strong odor from vaginal discharge.
3.
itching in the vaginal area.
4.
molluscum contagiosum.
postcoital bleeding

To assess a patient’s ability to think abstractly, a nurse practitioner could ask the patient:
the meaning of a common proverb.

Auspitz sign
droplets of blood when scales removed = psoriasis

Bacterial vaginosis
etiology: unclear, likely polymicrobial, associated with G. vaginalis, M. hominid, others.
thin, homogeneous, white, gray, adherent, often increased, discharge is foul odor(fishy), itching is occasionally present, pH 5-7(alkaline- no active bacilli in vagina), “fishy” smell, microscopic > 20 clue cells/HPF, few or no WBCs. Treatment: metronidazole topical, oral Flagyl, clindamycin vaginal cream, oral tinidazole (Tindamax)

CLUE CELLS, METRONIDAZOLE GEL OR ORAL, CLINDAMYCIN CREAM

Omeprazole (Prilosec)
interacts with with Warfarin (Coumadin),
diazepam (Valium),
Carbamazepine (Tegretol),
Pheytoin (Dilantin),
ketocanazole (Nizoral)

Viral Conjunctivitis tx
Antihistamine, decongestant drops (Trigluridine in herpes conjunctivitis)

What is the common presentation of a navicular fracture?
Tenderness at the “N spot,” which is defined as the proximal dorsal portion of the navicular (see the image below). This is the most important physical finding.

Melanoma
ABCDE: asymmetry, border irregularity, color variation, diameter greater than .6 mm, elevation above skin level.

Type 2 DM in children values
A1C: >6.5%
Fasting: >126
2 hour GTT: >200
Random: >200

Central clearing lesion after camping trip flu like symptoms with muscle aches for several days is? and treated by?
Lyme disease; doxycycline

Wilms tumor
(Nephroblastoma)- Not painful. Asymptomatic abd mass does NOT cross the midline. 2-3 y. o.d. do not palpate. Do ABD US. PUNT. Think Nephro doesn’t cross. Stays where kidney is.

the headache of an intracranial tumor
focal neurological signs and pain worse in supine position

socioeconomic status
not important during employment physical with 21 yo with bruising on breasts

The nurse practitioner prescribes amitriptyline (Elavil) for a patient with neuropathic pain secondary to diabetes mellitus. On follow-up, the patient complains of urine retention and dry mouth. The practitioner would:

1.
discontinue amitriptyline and begin ibuprofen (Motrin).
2.
refer to physical therapy.
3.
start methocarbamol (Robaxin).
4.
discontinue amitriptyline and begin gabapentin (Neurontin).
discontinue amitriptyline and begin gabapentin (neurontin)

To assess spinal function at the S1 level, which deep tendon reflex should be tested?
Achilles

Herald patch + christmas tree
pityriasis rosea

Secondary Prevention

  • screening tests (pap, mammogram, CBC).

Strep Pharyngitis tx
PCN, amoxicillin, macrolide, cephalosporin

Vitamin K Agonist Warfarin (Coumadin)
Interactw with “G” hers:
Garlic
Ginger
Gingko
Ginseng

Other herbs/supplemts:
Feverfew
green tea
fish oil

**Discontinue 7 days before surgery

What is the gold standard test for sickle cell anemia, glucose-6-phosphage dehydrogenase (G6PD) anemia, and alpha or beta thalassemia??
Hemoglobin electrophoresis

Postherpetic neuralgia
Tx: Prophylaxsis TCA-Elavil

Atrophic vaginitis (genitourinary syndrome of menopause GSM)
etiology: estrogen deficiency (after menopause). D/C scant, white-clear dryness as well sometimes urinary incontinence, itching/burning, discharge but often w/o symptoms, pH >5 (little to no lactic bacilli), odor is absent, microscopic few or absent lactobacilli.
Treatment: topical and/or vaginal estrogen if symptomatic and/or recurrent UTI. (Oral estrogen as solo intervention likely inadequate)

Diverticulitis: Treatment s/s:
Antibiotics and clear liquids and increased fiber (some say no nuts or seeds). (7 to 10 days ABT)
Ciprofloxacin (500 mg PO twice daily) plus metronidazole (500 mg PO three times daily). Amoxicillin-clavulanate (875/125 mg twice daily) is an acceptable alternative.

The criteria for patients with acute uncomplicated diverticulitis to be treated in the outpatient setting include:

●Reliability to return for medical reevaluation if condition worsens

●Compliance with outpatient treatment plan

●Abdominal pain is not severe

●No higher than a low-grade fever

●Can tolerate oral intake

●No or minimal comorbid illnesses

●Available support system

Approximately six weeks following the resolution of symptoms of acute diverticulitis, patients who have not had a recent colonoscopy should undergo one to exclude other possible diagnoses (such as colonic neoplasia) and to evaluate the extent of the diverticulosis.
Recomendations for surgery: Patients in whom elective surgery has been recommended following a single attack of diverticulitis include younger patients (variously defined in the literature as less than 40 or 50 years of age) and those who are immunosuppressed.

Neuroblastoma-
painful abd mass fixed first irregular, crosses midline. Most common side is adrenal glands. Weight loss fever. HORNERS syndrome. RACOON eyes, bone pain, HYPERTENSION. 1-4 year olds. Dx ultrasound PUNT to nephro. NEURO think brain in middle crosses midline. **Urine catecholamines and anemia

ADHD
hyperactivity, impulsive and/or inattention.

DSM-5 Criteria:
*sx present before 12 years of age
*sx for at least 6 months
*sx in 2 different settings

Medications considered 1st line if over 6 years of age

TZD
Pioglitazone (Actos)
Black Box warning: cause or exacerbate congestive heart failure in some patients; do not. use if New York health association Class III or IV Heart failure

STOP if causes dyspnea, weight gain, cough (heart failure)

in order to improve longevity of patient with COPD, tx of choice is
oxygen

current social relationships,
history of present or past traumas
mental health status
all important on the pe of 21 yo, for employment with bruising on breasts

A 17-year-old male with rheumatoid arthritis is being treated with an NSAID and omeprazole (Prilosec). The patient complains of headache, abdominal pain, and gas. These symptoms are most likely:

1.
associated with the omeprazole.
2.
related to the underlying condition.
3.
the result of the NSAID.
4.
caused by viral gastroenteritis.
associated with the omeprazole

Koplick Spots
white with red ring inside cheek from rubeola or mumps

Tertiary Prevention

  • AA, support groups, education for those with the disease, rehab, exercise programs for obese

Which of the following signs/symptoms are often associated with headaches due to an intracranial tumor?
Pain worse in supine position; focal neurological signs

Acute rhinosinusitis tx
Wait 10 days then Amoxicillin or Augmentin (If allergy, use fluouroquinolones/tetracyclines)

MRSA
Tx:

Autism
Impairment in social communication and social interaction. Restricted, repetitive patterns of behavior, interests and activities.

Red flags: delayed language/communication milestones, regression in social and language skills, sibling with autism

screen at 9, 18, 24 and 30 months or when concerns are raised by parents

Genital Herpes
causative organism: human herpes virus 2
clinical findings: painful, ulcerated lesions, marked lymphadenopathy with initial lesions. Women=thin vaginal discharge if lesion at vagina or intoitus
with recurrence symptoms vary- asymptomatic transmission common
treatment: oral acyclovir (Zovirax), famciclovir (Famvir), valacyclovir(Valtrex), dose and length of treatment depends on the medication choice and the clinical presentation.

Diverticulosis: Treatment s/s:
Diverticu-lO-sis has nO inflamation: Both diverticulitis and *osis are similar to umbilical hernia in that a vein running through the muscle of the bowel muscle weakens the area and pressure causes a hernia “out pouching”.

RETINOBLASTOMA
leukocoria: Hallmark sign white spots in eye. Cancer. Red light reflex negative

What are first and second line antibiotics for acute otitis media?

  1. Amoxicillin
  2. Amoxicillin-Clavulanate (Augmentin)

Screening Tests:

  • Breast Cancer: 50yrs q2years until 75yo. (40-49 is individualized)
  • Cervical Cancer: 21yrs q3yrs until 65y. no screening after hysterectomy
  • Colorectal Cancer: 50yrs until 75. FOBT x3 yearly. Flex sig q5yrs OR colonoscopy q 10yrs
  • Lipids: men >35y and women >45y. unless increased risk for heart disease
  • Prostate: don’t screen
  • Skin Cancer counseling: 10-24yo with fair skin

which of the following criteria differentiates a TIA from a CVA
absence of residual symptoms – TIA sx occur rapidly and then resolve, CVA sx are residual – TIA is temporary block and usually no residual sx

CD4 count and viral load
most widely accepted indicators of HIV infection

The medication of choice for the initial treatment of juvenile rheumatoid arthritis is:

1.
acetaminophen.
2.
prednisone.
3.
aspirin.
4.
ibuprofen.
ibuprofen

Which of the following laboratory tests are most widely accepted as indicators of the progression of HIV infection?
CD4 count and viral load

slapped cheek and lacy exanthem
Erythema Infectiosum or Fifths disease

Intertrigo tx
Nystatin

Atypical antipsychotics:
Risperidone (Risperdal)
Olanzapine (Zyprexa)
Quetiapine (Seroquel)
High risk for weight gain
metabolic syndrome
Type 2 DM

** high mortality in elderly patients

Monitor: weight and blood sugar
TSH, L lipids, weight body mass index

If a patient is allergic to penicillin and has a gram+ infection what are the alternative antibiotic choices?

  1. macrolides (azithromycin, clarithromycin)
  2. clindamycin
  3. quinolones with gram+ activity (levofloxacin or moxifloxacin)

Xerosis
Dry skin. Use petroleum based product.

Nongonococcal urethritis and cervicitis
causative organism: chlamydia trachoma’s, ureaplasma urealyticum, mycoplasma genitalium (obligate intracellular pathogen)
clinical findings: Friable cervix=bleeds easily, may have no symptoms, irritative voiding symptoms, occasional mucopurulent discharge, microscopic shows large number of WBCs. Treatment: azithromycin 1 g PO 1 time dose

ABUNDANT WBCs

Bacterial vaginosis: s/s and treatment
Clue cells such as: stipling of squamous epithelial cells with indistinct borders, no lactobacillus rods, many white blood cells

Nonpregnant women
Drugs — Metronidazole or clindamycin administered either orally or intravaginally results in a high rate of clinical cure (70 to 80 percent at four weeks of follow-up) (table 4) [88-91]. Oral medication is more convenient, but associated with a higher rate of systemic side effects than vaginal administration.

Side effects of metronidazole (oral or vaginal) include a metallic taste, nausea (in 10 percent of patients), transient neutropenia (7.5 percent), a disulfiram-like effect with alcohol, prolongation of International Normalized Ratio in patients taking vitamin K antagonists (eg, warfarin), and peripheral neuropathy.

Erythema infectiosum
(5th disease)- “slapped cheeks” 5-14 y.o. LACY, spreads to upper arms lgs trunks dorsum of hands and feet. Rash can last up to 40 days. Fever, rash, runny nose, headache. EXAM humanparovirus19, no labs for it.

Most common bacterial pathogen causing pneumonia
strep pneumoniae, but most pathogens 6 months to 5 years are viral

Biphosphonates:

Alendronate (Fosamax)
Risedronate (Actonel)
Erosive esophagitis, abdominal pain,
Stop immediately if symptoms:
esophagitis (chest pain, difficulty swallowing, burning (mid back)
jaw pain (osteonecrosis)

Contraindicated: Active GI disease (GERD, PUD), CKD, esophageal stricture/varices

prednisone
management of polymalgia rheumatica

A 12-year-old with sickle cell anemia has recently experienced a sickle cell crisis and presents for a follow-up examination after a recent hospitalization. It is most important to continue monitoring growth, development, and:

1.
white blood cell levels.
2.
fecal occult blood test.
3.
hemoglobin levels.
4.
urine dipsticks.
hemoglobin levels

A 3-year-old patient presents at an inner-city clinic with fever, cough, malaise, and loss of appetite. The patient lives with several relatives, including a grandmother who also has a cough. Which of the following diagnostic tests would be most appropriate for the patient?
Sputum culture

Thumb sign
swelling of the epiglottis, which may be visible on a lateral radiograph in patients with Epiglottitis

Lung Cancer Screening

  • 55-74yo with >30y pack smoking history and who quit <15y ago. LDCT annually

med of choice for polymyalgia rheumatica
prednisone

Urticaria tx
Benadryl or Zyrtec

Shingles
Prodrome: itching burning photophobia fever headache malaise. Acute phase: Dermatomal rash 3-4 days, unilateral, macupapular rash progresses to vesicles then pustules 3-4 days. Convalescent phase: 2-3 week rash resolves. Dx PCR. Tx: acyclovir, zostrix, gaba, amitriptyline. (one dermatone)

Gonococcal urethritis and vaginitis
1 million cases daily WW- abx resistance prevalent
causative organism: neisseria gonorrhoeae (gram – bacteria)
clinical findings: irritative voiding symptoms, occasional purulent discharge, often w/o symptoms in either gender
Microscopic exam: large number of WBCs
STI most likely to give penile DC also called the “drip”
Treatment: ceftriaxone 250 mg IM as a one time dose plus azithromycin 1 g PO x 1 dose

If you find Gonorrhea ALWAYS tx for chlamydia

CEFTRIAXONE

10 month old child with runny nose, rash, cough, with tiny white papules with red areola in mouth what does this suggest?
Measles

ADHD
hyperactive, impulsive, inattentive.
Present prior to 12 years.
Symptoms last > 6 months,
should be evident in at least 2 different settings. Treated with schedule II – Ritalin, Adderal, vyvanse, streterra etc

most sensitive sign of pneumonia in children
increased respirations. fever is inconsistent.

A patient with COPD is prescribed ipratropium bromide (Atrovent) for dyspnea. If no relief at follow up visit what is the next step?

  1. Albuterol inhaler (Ventolin)
  2. OR a combination inhaler

Statins
Atrovastatin (Lipitor)
Lovastatin (Mevacor)
Rosuvastatin (Crestor)
Simvastain (Zocor)
Do not mix with grapefruit juice
Drug induced hepatitis
Rhabdomylsis higher if mixed with azole antifungals

High dose zocor (80mg) has highest risk of rhabdomyolis (muscle pain/tenderness)

Chenese Descnet: higher risk myopathy or rhabdomyoliss when taking simvastatin 40mg/day with niacin
Creatine Kinase level goes up

polymyalgia rheumatica
Risk factors include female gender and age >50
Pain stiffness of the shoulder/pelvic girdle typically seen in >50 year olds with the “hallmark” difficulty combing/brushing hair responding well to low dose steroids

A 90-year-old female is brought to the clinic by her neighbor. She states that everything is fine, but the nurse practitioner notes that she has poor hygiene and bruises on her trunk. The neighbor is concerned that the patient often has no money to buy food, despite income from social security and a coal miner’s pension. The nurse practitioner suspects abuse. Which of the following is the nurse practitioner obligated to do next?
1.
Report the case to the proper authorities.
2.
Tell the neighbor to check on the woman daily and report back.
3.
Document the data and report the information to risk management.
4.
Call the patient’s family and inquire about the concerns.
report the case to proper authorities

Which of the following best describes psoriatic lesions in an elderly patient?
Red, sharply defined plaques with silvery scales

Steeple Sign
tapering of the upper trachea on a frontal chest radiograph reminiscent of a church steeple suggestive of Croup

Active Immunity

  • via immunization or a person who was exposed to agent

helical CT pulmonary angiography
test for PE

Actinic Keratosis tx
Topical 5 fluoracil 5-FU cryotherapy

What is the classic triad of symptoms for mononucleosis?

  1. sore throat
  2. prolonged fatigue
  3. enlarged cervical nodes
    (usually a teen)

Dog bite
Treat analgesia (tyenol, Nsaids, Demerol) Ab: Augmentin/doxycycline/Bactrim, wound cleaning with soap and water, betadine, lidocaine. Wound debridement, facial bites closed with sutures. Tetanus

Trichomoniasis
causative organism: trichomonas vaginalis (protozoan pathogen)
clinical findings: dysuria, itching, vulvovaginal irritation, yellow-green vaginal discharge, occasionally frothy (30%), cervical petechial hemorrhages (“strawberry spots”) in about 30%, often w/o symptoms in either gender, microscopic exam: motile organisms and large number of WBCs, pH is alkaline
Treatment: metronidazole 2 g (No alcohol) PO or tinidazole 2 g PO as a 1 time dose (parasitic infection) no etoh -GI upset

ORAL METRONIDAZOLE

Trunk like rash:
Roseola: measles

MASTITIS
red firm tender area fever chills, flu like symptoms. Dicloxacillin, or Keflex. If you suspect MRSA, do Bactrim or clinda.

treatment of bacterial pneumonia in children:
High dose amoxicillin, augmentin or 3rd generation cephalosporin (cefdinir).

if type 1 reaction to PCN use clindamycin or azithromycin

electric shock unilateral facial pain
trigeminal neuralgia

serum ferritin level
distinguish iron deficiency anemia from other anemias

In most cases, the first manifestation of Alzheimer’s disease is:

1.
impaired judgment.
2.
decrease in short-term memory.
3.
disorientation in time and place.
4.
decrease in long-term memory.
decrease in short-term memory

A 25-year-old presents with the chief complaint of decreased mobility and pain of the right shoulder exacerbated by movement. The patient reports that he participated in extensive house painting 24 hours prior to the onset of pain. He denies any trauma. Passive ROM is intact. No redness or ecchymosis is present. What is the next step that should be taken in order to make a diagnosis?
Palpate structures around the shoulder.

Galeazzi’s Sign (aka Allis sign)
uneven knee heights – Developmental Displasia of the Hip

Passive Immunity

  • immunoglobulin or through breast feeding/from mother

Atopic Dermatitis (eczema) tx
Topical steroids, emollients

Lincosamides
Clindamycin (cleocin)
Higher risk of CDAD
Metronidazole (flagyl) PO TID x 10-14 days
Probiotics daily-BID x few weeks

Alpha thalassemia is more common with what ethnicity?
Southeast Asians (Filipinos)

Lupus
Multisystem autoimmune disease, ch by remission and exacerbations. Butterfly rash, avoid sun exposure, Tx: Refer rheumatology, topical steroids and oral steroids.

Syphilis
causative agent: treponema pallidum (spirochete bacterium)
clinical findings:
Primary stage: chancre, firm, round, painless genital and/or anal ulcers with clean bas and indurated margins, localized lymphadenopathy, aprox 3 weeks duration, resolves w/o treatment. Secondary stage: nonpruritic skin rash, *palms and soles, as well as mucous membrane lesions, fever, lymphadenopathy, sore throat, patchy hair loss, headaches, weight loss, muscle aches, and fatigue. Resolution w/o treatment is possible.
Latent stage: presentation variable w/ decrease in cognitive function, occurs when primary and secondary symptoms have resolved
Treatment: antimicrobial therapy, with dosage and length of therapy usually dictated by disease stage. Options include injectable PCN or PO doxycycline
IM Bi-cillin- used as well

INJECTABLE PENICILLIN OR IF SEVERE PENICILLIN THEN GIVE DOXYCYCLINE

Other Childlike rashes: Key characteristics:
INSERT HERE

UTI in pregnancy
10 (3) wbc is considered positive in prego with symptoms. Normal people its 10 (5). MEDS: Macrobid (not for 3 trimester) Augmentin, Amoxicillin, Cephalexin, Fosfomycin.

bronchiolitis
Lower resp. tract illness that occurs when an infecting agent causes inflammation and obstruction of the small airways (bronchioles).
Common under 2 years of age.
Management: supportive. No bronchodilators or steroids

Anaphylaxis

  • Pruritus/uticaria or angioedema AND EITHER respiratory compromise OR hypotension/end organ dysfucnction
  • give Epi pen then send to ED
  • refer to allergy if cause is unknown

sudden vision loss in which person feels like a curtain came down over his eye
retinal detachment

tx with clear liquids and oral antibiotics
pt with diverticulosis, temp 100, localized LLQ discomfort, palpable mass, leukocytosis, and CAN TOLERATE FLUIDS

The optimal treatment for latent tuberculosis is:

1.
rifampin (Rifadin) for 5 months.
2.
isoniazid (Nydrazid) for 9 months.
3.
pyrazinamide for 6 months.
4.
ethambutol for 6 months.
osioniazid (nydrazid) for 9 months

A patient who is 28 weeks pregnant reports a single episode of vaginal bleeding. History indicates normal prenatal progress to date, and the patient denies pain, vaginal itching, or discharge. Which of the following is the most appropriate intervention to aid in the diagnosis of this case?
Ultrasound

Gower’s Maneuver
patient that has to use their hands and arms to “walk” up their own body from a squatting position due to lack of hip and thigh muscle strength = muscular dystrophy

Anthrax
Tx: doxycycline/fluoroquinolones

Seborrheic Dermatitis tx
Rotation of prescription/non-prescription antifungal shampoo (Ketoconazole/metronidazole), Capitrol shampoo, selenium sulfide, selsun blue (adults / children) ciclopirox shampoo, topical stroid gel (hydrocortizone face, ears hydrocortizone cream), eyelids – baby shampoo

cystic fibrosis
*Effects lungs, GI and sweat glands.
*Autosomal recessive
*Routine screening in all states
*Presents with recurrent lower respiratory infections and persistent productive cough
*Weight loss and greasy BM common
*Mucous blocks ducts of the pancreas

Genital warts (Condyloma acuminata)
Causative organisms: HPV (commonly HPV-6 & 11)
Clinical findings: verruca-form lesions can be subclinical or unrecognized
Treatment: prevent w/immunization, topical podofilox, liquid nitrogen, cryoprobe, trichloroacetic acid, bichloracetic acid, surgical removal, or topical imiquimod (only indicated for external warts treatment).
trichloroacetic acid use acceptable in pregnancy

DO NOT USE IF PREGNANT: podofilox, podophyllin, sinecatechins and imiquimod

IMIQUIMOD

Erythema toxicum
A rash of small yellow or white bumps surrounded by red skin. Can appear anywhere on the body. Disappears on its own in about two weeks. Common in newborns, usually showing up two to five days after birth.

Alpha fetal protein test
AT 16 WEEKS TEST FOR AFP
Low- Downs
High-Neural tube deficits

Beta thalassemia is more common with what ethnicity?
Mediterranean people

Live Vaccines

  • MMR: 1 yr and 4-6y (if out of country give 1 dose)
  • Varicella: 1 yr and 4-6yr
  • FluMist
  • Zoster
  • Rotavirus

*give same day or separate doses by 1 mo.

decreased gastric production with aging

tests for polycystic ovarian syndrome
testosterone, follicle stimulating hormone, luteinizing hormone, prolactin, thyroid stimulating hormone

Unilateral spontaneous serous or serosanguineous discharge from a single duct of a breast is most often caused by:

1.
intraductal papilloma.
2.
mucinous breast lesions.
3.
Paget’s disease.
4.
ductal carcinoma in situ.
intraductal papilloma

A 29-year-old male with noncomplicated Chlamydia infection may exhibit:
no remarkable clinical symptoms.

Fat pad/Sail sign
Elbow fracture (in kids, a posterior fat pad sign suggests a condylar fracture of the humerus. In adults it suggests a radial head fracture)

Moderate acne tx
oral abx + topical retinoid +/- benzoyl peroxide (tetracycline + tazarotene +/- Benz Pero ….
Retin topical, oral tetracycline then Accutane (isotretinoin)

What do pica and spoon shaped nails indicate?
Iron deficiency anemia

Subungual hematoma
collection of blood underneath a toenail or finger nail. Tx: Trephination, drill hole and drain the blood.

Which HPV viruses most likely to cause malignancies
16, 18, 31, 33, 45, 52,& 58. Gardasil effective against all of these plus 6 & 11

Fifth disease
Starts with a slight fever, achiness, and cold symptoms, followed a few days later by bright red cheeks and a lacy, red, sometimes itchy rash on the trunk and feet. Also called slapped cheeks disease or erythema infectiosum. Most common in preschool and school-age children.

Condyloma Acuminata
genital warts types 6/11 HPV. Treatment: trichloracetic acid….Condylox, aldara, veregne. In pregnancy: TCA or ablation

Lead toxicity
*Sx: fatigue. stomach ache, irritability.
*IDA
*Cognitive and behavioral changes cannot be reversed

Hep B Vaccine

  • 3 doses.
  • don’t ever restart series
  • test pregnant women for HBsAg
  • if received series and then exposed to Hep B if no HbsAb test done give another dose to be safe

MED FOR MYCOPLASMA PNEUMO
AZITHROMYCIN

primary amennorhea presentation
17 yo Tanner Stage III- never had menses, normal weight and height

A young child with asthma presents for follow-up evaluation. After numerous changes in medications and doses, the parents report that the child continues to have difficulty with coughing, especially during the night. Which of the following conditions would be the most likely cause of the continued asthma symptoms?

1.
Vocal cord dysfunction
2.
Cystic fibrosis
3.
Gastroesophageal reflux
4.
Allergic rhinitis
gastroesophageal reflux

An 88-year-old patient presents with right-sided weakness after being unable to rise unassisted following a fall to the bathroom floor. History includes aphasia and noncompliance with a hypertension medication regimen. What is the most likely diagnosis?
Left-sided stroke

Adam’s sign
Forward bend test for scoliosis

Leukemia
*Most common in children ALL
*Pancytopenia
*Long bone pain
*Hepatosplenomegaly
*Lymphoblasts

Hidradentitis suppurativa tx
Doxycycline, topical atb

What is a common presentation for systemic lupus erythematosus (SLE)?
malar rash (butterfly rash) and should minimize sunlight exposure

Peripheral arterial disease
Impedance of arterial blood flow in lower extremity ankle brachial index <0.9, plaque develops in vessel d/t atherosclerosis, pain with exercise, relief w rest, lack of hair growth on LE, gangrene toes Tx: check pedal pulses, ABI test, exercise by walking, lifestyle Modifications – smoking cessation, antiplatelet

Pelvic inflammatory disease (PID)
causative organisms: N. gonorrhoeae C. trachomatis, bactericides, enterobacteriaceae, streptococci, gram – anarobes
clinical findings: irritative voiding symptoms, fever, abdominal pain, cervical motion tenderness, vaginal discharge. *Increased risk of ectopic pregnancy and/or infertility with each outbreak.
treatment- Ceftriaxone 250mg IM as a single dose plus doxy 100mg po bid x14days with or without metronidazole 500mg PO BID 14days

Folliculitis
Pimples or pustules form around hair follicles and may crust over. Typically occurs on the neck and in the underarm or groin area. Uncommon before age 2.

Syphillis
Syphillis- PAINLESS GENITAL CHANCRE- Condyloma lata. First test do RPR, VDRL SCREENING if reactive then confirm with FTA ABS.

Td/ TdaP

  • q10yrs. or if dirty wound give booster if last dose >5yrs ago.
  • Tdap is only once (older than 7yo), q10is Td. and during EVERY PREGNANCY
  • C/I if Guillain- Barre syndrome

-Dtap 3 doses in children or unimmunized (adults get 2 Dtap and 1 tdap)

SSRIs can cause delayed ejaculation

secondary amennorhea
started but now stopped. Causes are: pregnancy, weight loss, obesity, manipulation. Treatment is hormone replacement, ovulation stimulation (CLOMID) periodic progesteronal

A pregnant woman with known HIV infection can reduce the risk of perinatal transmission through zidovudine (Retrovir) therapy. Based on current research, optimal therapy is to start daily dosing:

1.
post amniocentesis.
2.
after 14 weeks of gestation.
3.
if premature rupture of membranes occurs.
4.
if maternal viral loads are greater than 10,000.
after 14 weeks gestation

A patient with macular degeneration has difficulty seeing objects:
in the center of the visual field.

Kernig’s Sign
Flex hips 90 degrees pain with extension of leg = meningitis or subarachnoid hemorrhage

Headache red flags
*awakens child
*thunderclap
*neuro findings – n/v, AMS
*<3 years old
*absent family hx of migraines

Postherpetic Neuralgia PHN Tx
Prophylaxis: TCA (Elavil)

What is the presentation of polymyalgia rheumatica (PMR) and first-line treatment?

  1. Pain, severe stiffness in shoulders and hip girdle (risk for temporal arteritis)
  2. Long-term steroids

chronic venous insufficiency
Varicose veins results from venous incompetence secondary to valvular dysfunction. Symptoms LE edema, skin discoloration, ulceration, DVT/PE are complication. WARM TO TOUCH. Tx: light exercise, stockings, wt loss, elevate legs

Vulvovaginitis or STI?

  1. Clue cells with alkaline pH
  2. Pseudohyphae
  3. Abundant WBCs
  4. Clue cells with alkaline pH = Bacterial vaginosis
  5. Pseudohyphae= Candida vulvovaginitis
  6. Abundant WBCs= Nongonococcal cervicitis/vaginitis

Hand, foot, and mouth disease
Fever, loss of appetite, and a sore throat, followed by painful, blisterlike sores in the mouth. Rash on the palms of the hands, the soles of the feet, and sometimes the buttocks. The rash starts as small flat red dots that may turn into bumps or blisters. Most common in preschoolers but can occur at any age.

chanCROID
PAINFUL, purulent lesion (NOT chancre, which is painless and associated with syphillis) oral dose (1 gram) of azithromycin, a single IM dose of 250 mg ceftriaxone, oral 500 mg of erythromycin q.i.d for seven days, or 500 mg of Ciprofloxacin b.i.d for three days.

best laboratory test to distinguish IDA anemia from other anemias is
serum ferritin

whether the pt has reversible airway obstruction
office spirometry using an albuterol nebulizer can confirm asthma, because it can indicate

T-wave inversion with a normal ST segment on a 12-lead EKG may represent:

1.
acute coronary ischemia.
2.
right ventricular hypertrophy.
3.
atrial hypertrophy.
4.
hyperkalemia.
acute coronary ischemia

Which of the following is the leading cause of cancer-related deaths in the majority of women?
Lung Cancer

Brudzinski’s Sign
Involuntary flexion of legs when neck is passively flexed = meningitis

Flu vaccine

  • 6mo or older. allergy to egg is no longer a contraindication
  • flu mist not to pregnant women
  • children 6mo-8yo need 2 doses for first flu season

Cellulitis tx
Pcn, Macrolide

What is the gold standard exam for temporal arteritis?

  1. biopsy of the temporal artery
  2. Refer to ophthalmology

Bacterial endocarditis
Fever, chills, anorexia, weight loss. malaise, headache, myalgia, arthralgia, night sweats, abdominal pain, dyspnea, cough, pleuritic pain. Symptoms janeway lesions, red spots on the soles of the feet or palms. Osler’s nodes red, tender spots under the skin, whites of your eyes, or inside your mouth.

Splinter hemorrhages on nails, Janeway lesion (red macules palms/soles not painful), Osler’s nodes (painful violaceous nodes found mostly on pads of the fingers and toes).

Intervention in vulvovaginitis

  1. Clotrimazole cream
  2. Metronidazole gel
  3. Oral metronidazole
  4. Clindamycin cream
    A. Candida vulvoginitis
    B. Trichomoniasis
    C. Bacterial vaginosis
  5. Clotrimazole cream. A. Candida vulvoginitis
  6. Metronidazole gel. C. Bacterial vaginosis
  7. Oral metronidazole.
    C. Bacterial vaginosis
    B. Trichomoniasis
    .
  8. Clindamycin cream. C. Bacterial vaginosis

Impetigo
Small red bumps that may be itchy. Often develops around the nose and mouth but can easily spread to other parts of the body. Bumps become pus-filled blisters that may burst and develop a soft yellowish-brown crust. Child may have a fever and swollen lymph glands in the neck. Most common in children between 2 and 6 years old.

PID
cervical motion tenderness indicates PID.
results in infertility
Treat symptomatic PID even if chlamydia and gonorrhea are negative. plus doxycycline (100 mg orally twice daily for 14 days)

ceftriaxone 250 mg IM x1 + Doxy 100 mg Q12 x 14d Follow up with bimanual exam in 2-3 days

child headaches
*c/o abdominal pain, n/v
*auras common

bump on testicle can indicate cancer of testicle

CBC and hemoglobin electrophoresis
12 yo, family hx of thalassemia and anemia, with Tanner Stage II, HCT 35 should order this

The 16-year-old mother of a 2-month-old presents the infant, reporting that the child is very irritable and does not feed well. During physical examination, the child’s head drops back and the child exhibits sudden flexing of the extremities. As the flexing stops, the child cries uncontrollably. Funduscopic examination reveals retinal hemorrhages. Which of the following diagnostic tests should be ordered?

1.
Skull X-rays
2.
MRI
3.
CT scan
4.
Pet scan
CT scan

Which of the following would be most appropriate to perform in the initial evaluation of a patient with symptoms of acute prostatitis?
Urinalysis and urine culture

Café au lait spots
neurofibromatosis (tumors and seizures)

Varicella

  • LIVE
  • 2 doses one month apart.
  • give if no history of chicken pox

Erysipelas tx
PCN or macrolide

What does a positive Finkelstein test indicate?
de Quervain’s tenosynovitis

Treatment for which of the following STIs?

  1. Ceftriaxone
  2. Injectable pcn
  3. Imiquimod
  4. Trichloroacetic acid (TCA)
    A. External genital warts in a 25yo man
    B. Gonococcal urethritis in a 22 yo man
    C. Syphilis in 45 yo man
    D. External genital warts in a 28 yo pregnant woman
  5. Ceftriaxone-B. Gonococcal urethritis in a 22 yo man
  6. Injectable pcn-C. Syphilis in 45 yo man
  7. Imiquimod-A. External genital warts in a 25yo man
  8. Trichloroacetic acid (TCA)-A. External genital warts in a 25yo man. *D. External genital warts in a 28 yo pregnant woman

Measles
Starts with fever; a runny nose; red, watery eyes; and a cough. Red bumps with tiny white dots appear a few days later on the inside of the cheeks. Next, a rash appears on the face and progresses down the back and trunk to the arms and hands and finally to the legs and feet. The rash starts as flat, red patches but eventually develops bumps and may be itchy. It lasts about five days, then fades to a brownish color, leaving skin dry and flaky. Rare due to vaccinations; most likely to occur in unvaccinated children.

Gonorrhea
Rocephin 250 mg IM and Azithromycin 1 gm po x1, or doxy 100 mg BID x7d. Green colored vaginal discharge, friable cervix

hyperbilirubinemia
*Infants >35 weeks w/ total bilirubin >95th percentile.
*Total bilirubin >25 to 32 = risk for neurologic dysfunction.
*Physiologic – 2 to 3 days after birth (full-term). Peaks day 5 to 7 in preterm infants.
*Pathologic – 1st 24 hours after birth.

PPD >5mm
Induration of 5 or more millimeters HIV-infected persons
-A recent contact of a person with TB disease
-Persons with fibrotic changes on chest radiograph consistent with prior TB
-Patients with organ transplants
-Persons who are immunosuppressed for other reasons

Shingles/Zoster

  • 60yo
  • regardless of history of shingles

for diabetic patients taking beta blockers, best indication of hypoglycemic episode will be
sweating (WILL NOT SEE TACHY AND PALPITATIONS DUE TO BETA BLOCKER USE)

absence of residual symptoms
differentiates TIA from stroke

The most important diagnostic factor in evaluating angina pectoris is the patient’s:

1.
King of Heart’s monitor.
2.
physical examination.
3.
history.
4.
echocardiogram.
history

Which of the following gastrointestinal changes is associated with normal aging?
Decreased production of gastric acid

Howell-Jolly Bodies and target cells
Sickle Cell Anemia

MRSA tx
Bactrim or tetracyclines

What does a positive Anterior drawer or Lachman maneuver indicate?
Positive indicated anterior cruciate ligament (ACL) is damaged

PPD: 10mm positive for
An induration of 10 or more millimeters
-Recent immigrants (< 5 years) from high-prevalence countries
-Injection drug users
-Residents and employees of high-risk congregate settings
-Mycobacteriology laboratory personnel
-Persons with clinical conditions that place them at high risk
-Children < 4 years of age

  • Infants, children, and adolescents exposed to adults in high-risk categories

Genital Candida albicans infection in men typically presents
balanitis, groin-fold involvement, and scrotal excoriation (no penile discharge. Jock itch does not affect the scrotum.)

Milia
Tiny white or yellow pearly bumps on the nose, chin, and cheeks. Common in newborns. They go away without treatment in a few weeks.

mammogram frequency
50-74 q2years unless family history or other concerns (Women with a parent, sibling, or child with breast cancer are at higher risk for breast cancer and thus may benefit more than average-risk women from beginning screening in their 40s.)

Pyloric Stenosis
*Narrowed pyloric sphincter due to hypertrophy
*Sx mostly likely to occur at 3-6 weeks
*Projectile, non-bilious vomit after eating.
*Olive like mass
*Dx with US
*Refer for surgery

Meningococcal MCV4
1-2 doses: 11-12yo and booster at 16yo

lung ca
leading cause of cancer deaths in men and women.

There are three reasons for latex allergies. Which of the following does NOT cause the problem?

1.
Immediate hypersensitivity
2.
Irritant contact of dermatitis
3.
Cytotoxic hypersensitivity
4.
Immune complex reaction
cytotoxic hypersensitivity

Which of the following is NOT an indication of preeclampsia?
Gluosuria

Burtonian Lines
Thin, blue-black discoloration of gingival border = lead poisoning

S1 spinal function is assessed with what DTR
achilles

Xerosis tx
Petroleum based product (not lotions)

PPD: 15mm
An induration of 15 or more millimeters is considered positive in any person, including persons with no known risk factors for TB.

Consideration in caring for a 68 yo m w/a BMI of 38, otherwise healthy, who is now presenting with genital candidiasis includes which of the two most helpful measures?
A. Order a medium potency topical steroid to the affected region to help with symptom control
B. prescribe topical miconazole
C. obtain an in office blood glucose
D. advice on the use of antimicrobial soap to the region
B. prescribe topical miconazole
C. obtain an in office blood glucose

Molloscum contagiosum
Flesh-colored, dome-shaped lesions that can be pearly in appearance and have a dimpled center. May be itchy. Uncommon before the first birthday.

Postmenopausal bleeding f/u testing

  • ENDOMETRIAL BX

GER
*Frequent regurgitation
*GERD = pathology
*Immature LES until 9-12 months old
*Red Flags: choking with eating, coughing with eating, forceful vomiting, bilious vomit, blood in stool, poor weight gain, refusal to eat, constipation or diarrhea, abdominal tenderness, fever.
*Avoid smoke, small frequent feedings (thickened), trial of PPI.

What can a positive McMurray’s sign indicate?
Meniscus injury of the knee.

Pneumonia Pneumovax- PPSV-23

  • one dose 65yo in well adults
  • if vaccinated before 65y need booster in 5 yrs and then again at >65yo

Paxil, SSRI
delayed ejaculation

A patient on warfarin (Coumadin) therapy for recurrent deep vein thrombosis (DVT) is about to have lumbar spinal fusion surgery. The patient’s warfarin is put on hold starting 5 days prior to the surgery and subcutaneous Lovenox has been ordered for DVT prophylaxis until the resumption of the warfarin. The nurse practitioner knows that the patient’s postoperative warfarin dose should be restarted based on the:

1.
value of her morning Prothrombin time.
2.
loading dose of 10 mg, plus the previous warfarin dose.
3.
baseline PT and INR values.
4.
target INR of 2.
caseline PT and INR values

Assessment findings that would support a diagnosis of mitral regurgitation include:
loud, high-pitched pansystolic murmur.

Goodwell’s Sign
Softening of the cervix (pregnancy indication)

children over 8 years old can use doxy

Psoriasis tx
Topical Steroids

Lateral X-ray of the neck
Epiglottitis

Acute, uncomplicated UTI in non pregnant women
Pathogen: E. coli (gram-, most common), Klebsielle spp. (gram-), S. saprophyticus (gram+)
Treatment: E. coli: TMP/SMX-DS PO BID x 3 days (always order double strength). If E. coli resistant or sulfa allergy then nitrofurantoin (Macrobid) 100 mg BID x 5 days or fosfomycin (Monurol) 3 g PO x 1 dose. Add phenazopyridine (Pyridium) PO to assist with symptom control.

Roseola
Usually starts with a sudden relatively high fever, often over 103 degrees Fahrenheit, that typically lasts three to five days. A pink rash on torso and neck follows and may spread to the arms, legs, and face. Child may be irritable and may have diarrhea or vomiting. Most common in children between 6 months and 3 years old.

Atrophic vaginitis cause and tx
lack a estrogen, apply topical estrogens

Intussusception
*Intestinal obstruction
*Sudden onset. Intermittent. Crampy abdominal pain.
*Cries and pulls legs up to chest
*Currant jelly BM
*Dx: US
*Non-operative: enema under fluoroscopy or surgery

On eye exam, what does neovascularization, cotton wool spots, and micro-aneurysms indicate?
diabetic retinopathy

PCV13

  • children receive
  • give first
  • only ever need 1 dose

in doing initial eval of patient with suspected acute prostatitis, what test to do
urinalysis and culture

Somogyi effect
Type 1 diabetes with increased early am levels with correct insulin dose

An 87-year-old patient presents with round, pruritic plaques and small vesicles on the lower legs. The most likely diagnosis is:

1.
allergic contact dermatitis.
2.
plaque psoriasis.
3.
cutaneous T-cell lymphoma.
4.
nummular eczema
nummular eczema

An 18-year-old college student presents for an athletic physical. When asked about current medications, she mentions that she takes “some herb” she bought at a health food store for migraines and menstrual cramps. Which of the following herbal remedies has been commonly used for these conditions?
Feverfew

Chadwick’s Sign
bluish discoloration of the cervix, vagina, and labia resulting from increased blood flow observed 6-8 weeks after conception

On eye exam, what can atrioventricular nicking, silver and/or copper wire arterioles indicate?
Hypertensive retinopathy

Shingles Dx & Tx
Dx: Viral Culture, polymerase chain reaction (PCR)
Tx: Acyclovir, Zostrix cream, Gabapentin, amitriptyline

Croup (laryngotracheobronchitis) Parainfluenza (*)
cold symptoms, low fever, stridor, barking cough, and hoarseness. No drooling, nasal flaring, sore throat, resp distress, ab breathing. Steeple sign frontal radiograph of neck. Mild: Outpt supportive care Mod: Hosp resp support Racemic epi, short course corticosteroids

Which of the following is inconsistent with BPH?
A. obliterated median sulcus
B. size >= 2.5 cm x 3 cm
C. symptoms improved w/use of an alpha-1 receptor blockade such as tamsulosin (Flomax)
D. surgical intervention should be offered early in the disease process
D. surgical intervention should be offered early in the disease process

Ringworm
Rash of one or several red rings, ranging from dime- to quarter-size. Rings are usually crusty or scaly on the outside and smooth in the center and may get larger over time. May also appear as dandruff or bald spots on scalp. Most common in children 2 and older.

WET PREP for which gu infections
BV, YEAST, TRICH

Encopresis
*Involuntary soiling in child 4 years or older
*Underlying issue: constipation
*Loss of urge to defecate
*Management: PEG, Miralax. Behavior change, fiber, fluids, reward system.

Hegar’s Sign
Softening of the cervicouterine junction

Koplik spots
measles

dawn phenomenon
Characterized by hyperglycemia that is present on awakening in the morning due to the release of counterregulatory hormones in the predawn hours.

A 3-year-old presents with a 2-day history of acute diarrhea with a total of 8 watery stools without blood or mucus, and 2 episodes of vomiting in the past 48 hours. Assessment reveals no current antibiotic therapy, dehydration <5%, soft abdomen with hyperactive bowel sounds, no masses or organomegaly; other physical findings are normal. Which of the following is the most appropriate management plan?

1.
Stool culture; Immodium AD liquid.
2.
No laboratory workup; instruct parents on signs and symptoms of dehydration; diet of clear liquids, advancing to bananas, rice cereal, Jell-O, and soup.
3.
Start diet of clear liquids, advancing to bananas, rice cereal, Jell-O, and soup over next 72 hours; Imodium AD liquid, 1 mg/sml
4.
Stool for ova and parasites; advise parents of likely sources of Giardia; prescribe furazolidone (Furoxone)
No laboratory workup; instruct parents on signs and symptoms of dehydration; diet of clear liquids, advancing to bananas, rice cereal, Jell-O, and soup.

An 88-year-old patient has had a gradual onset of hearing loss in the left ear. Examination shows a large accumulation of cerumen in the external auditory canal. Assuming there is no neural loss, the nurse practitioner would expect the Weber test to:
lateralize to the left

BCG

  • LIVE for TB
  • not in US.

Spider bite tx
Atb on wound, cold packs, NSAIDS

When checking deep tendon reflexes (DTRs) in a patient with severe sciatica or diabetic peripheral neuropathy, what can you expect the Achilles reflex to show?
The achilles reflex may be absent or hypoactive (0-1+)

Epiglottis (*)
Hemophilus influenza, Streptococci, Pneumococci. (6-10) Rapid onset, acute bacterial infection, high fever, stridor, choling sensation, resp distress, drooling, head tilted back to breath (hyperextension). Thumb sign on radiograph. ER!! Cephalosporin

Epididymoorchitis (upper reproductive e tract infection with inflammation of epididymis/testis)
Age ⬇️35
Causative organisms: N. gonorrhoeae, C. trachomatis
Presentation: irritative voiding symptoms, fever, painful swelling of epididymis and scrotum. Infertility is possible post infection due to scarring of the vas deferins
Treatment: Ceftriaxone 250 mg IM x 1 dose plus doxycycline 100 mg po BID x 10 day.
Scrotal elevation to help with symptom relief. (Prehn’s sign=relief of discomfort with scrotal elevation)

Rubella (German measles)
Pink-red rash that appears first on the face, then spreads over the body and lasts two to three days. Child may have a mild fever, swollen lymph nodes behind the ears, a runny or stuffy nose, a headache, and a sore throat. Rare due to vaccinations; most likely to occur in unvaccinated children.

Trichomonas s/s, dx, tx
Dysuria, itchy, strawberry cervix, +/- discharge: grayish bubbly, wet prep: mobile unicellular organisms with flagella tx Flagyl 2g x 1 OR 500 mg q12h x 7d

Viral Gastroenteritis
*Fecal oral route common
*Measure for dehydration: weight loss, capillary refill, loss of skin turgor, increased and deep RR.
*Oral rehydration for mild to moderate dehydration over 3-4 hours
*50 mL/kg (mild) to 100 mL/kg (moderate) over 4 hours

Dog bite tx
Analgesics: Tylenol, nsaids, Demerol
ATB: Augmentin/doxycycline/Bactrim,

macrolide antibiotic
smoker with acute bronchitis with cough with thick, yellow-green foul smelling sputum

A mother presents her 12-month-old child with concern because the child does not yet say “mama” or “dada.” The mother reports that in the first months of life, the child loved listening to music and being talked to. The past medical history is negative for ear infections. The nurse practitioner should:

1.
evaluate for hearing loss.
2.
refer to speech therapy.
3.
re-evaluate in 3 months.
4.
check for cerumen impaction.
evaluate for hearing loss

During an employment physical examination of a 21-year-old female, bruising around the areolae on the breasts is noted. An appropriate health history for these findings should include all of the following EXCEPT:
socioeconomic status

Chvostek sign
tap the facial nerve in front of the tragus and the ipsilateral muscles of face contract – indicative of hypocalcemia

pernicious anemia is B12 deficiency

True or false, a benign S4 heart sound may be auscultated in some elderly patients
True

Dacrocystitis
inflammation of the lacrimal sac. Symptoms includes thick eye discharge, pain, redness/swelling/warmth of lower eyelid. Tx includes lacrimal sac massage downward toward mouth 2-3 times daily and systemic antibiotics if needed.

Epididymoorchitis usually e-coli
Age ⬆️ 35 or insertive partner in anal intercourse
Causative organisms: Enterobacteriaceae (coliforms)
Presentation: irritative voiding symptoms, fever, painful swelling of epididymis and scrotum. Infertility is possible post infection.
Treatment: Levofloxacin 500 mg PO QD or ofloxacin 300 mg PO BID x 10 days

Scabies
Severely itchy rash of scattered red bumps, usually between the fingers, around the wrists, in the armpits and diaper area, and around the elbows. May also show up on the kneecaps, palms, soles, scalp, or face. May leave curvy white or thin red lines or little blisters on nearby skin. Itching is most intense after a hot bath or at night and may keep the child awake. May occur at any age.

VVC
Candidia Vaginitis- wet smear= pseudohyphae / spores w/ lrg wbc.
Cheese curd like pruritis, itching, swelling, redness. TREAT: Diflucan 100 mgx1. Or OTC= Monistat, clotrimazole

Hydrocele
Common in newborns and is a collection of fluid in the scrotum. Usually resolves before 12 months of age.

most appropriate dx tests for PCOS are
testosterone, LH, prolactin, TSH

The most common side effect of the oral ribavirin used in the treatment of hepatitis C is:

1.
hemolytic anemia.
2.
weight loss.
3.
depression.
4.
hypothyroidism.
hemolytic anemia

Which of the following findings is typically a sign of acute appendicitis?
Rebound tenderness at McBurney’s point

Dix-Hallpike or Nylen-Barany maneuver

  • = Vertigo & nystagmus when quickly move pt to supine position with head 30 degrees lower than table and to turned to the side at 45 degree

What is a rare but life threatening reaction to ace-inhibitors? What is a common side effect?
Angioedema
-cough

Lyme Disease tx
Doxycycline, amoxicillin, or azithromycin

Erysipeals
Skin infection involving upper dermis and superficial lympahtics. Strep A and has clear demarcated boarder, tx includes PCN such as dicloxacillin qid x 10 days, Cephalexin, clindamycin for PCN allergic do Azithromycin.

Acute bacterial prostatitis
Age ⬇️ 35
Causative organisms: N. gonorrhoeae, C. trachomatis
“it hurts when my bottom hits the chair.”
Presentation: irritative voiding symptoms, suprapubic, perineal pain, fever, a tender, boggy prostate, leukocytosis increased white count
Treatment: ceftriaxone 250 mg IM as 1 x dose w/doxycycline 100 mg PO BID x 10 days

Scarlet fever
The rash begins as a mass of tiny red bumps in the armpits, neck, chest, and groin and rapidly spreads over the entire body. The bumps feel like fine sandpaper and may itch. Child may have a fever and a red sore throat. Early in the infection, the tongue may have a white or yellowish coating that later turns red. The bumps on the tongue may appear larger than normal, a condition called strawberry tongue. Tonsils may be swollen and red. As the rash fades, the skin may peel, especially on the hands and feet and in the groin. Uncommon before age 2.

BV : s/s, microscopy, test, tx,
fishy, smelly vaginal discharge, wet smear reveals epithelial cells with bacterial coating, KOH to cotton swab – whiff test, flagyl cream x 7 days, clindamycin cream, flagyl PO

UTI
*Causes: Bowel and bladder dysfunction (withholding, incontinence, constipation), anatomic abnormalities, vesicoureteral reflux.
*Treatment: 2nd or 3rd generation cephalosporin (cefixime, cefdinir, ceftibuten) for 3-5 days if afebrile, 10 days if febrile.
*Renal and bladder US for all infants 2 to 24 months with 1st febrile UTI.
*Voiding cystourethrogram for vesicoureteral reflux.
*Get US in any child with recurrent febrile UTIs and UTI with family hx of renal or urologic disease, poor growth or HTN

Lupus tx
Topical and oral steroids

weight loss, shoulder and pelvic pain, pain with ROM, no weakness, elevated ESR
polymyalgia rheumatica!!!!

An 86-year-old patient presents with pinguecula, a yellow triangular degenerative tissue thickening of the bulbar conjunctiva. This condition is:

1.
cured by eye drops.
2.
a genetic process.
3.
a malignant growth.
4.
a solar-induced lesion
a solar-induced lesion

The best laboratory test to distinguish iron deficiency anemia from other anemias is:
serum ferritin level

McBurney’s Point
RLQ of abdomen that is one-third of the distance from the anterior superior iliac spine to the umbilicus (location of appendix)

Acute bacterial prostatitis (uncomplicated disease in men with low risk for STI)
Causative organisms: Enterobacteriaceae (coliforms)
Presentation: irritative voiding symptoms, suprapubic, perineal pain, fever, a tender, boggy prostate, leukocytosis
Treatment: ciprofloxacin 500 mg PO BID or ofloxacin 200 mg PO daily x 14 days

What class of drug is Ipratropium (Atrovent)
Anticholinergic

Normal HR at birth
120-160

Warts
A small grainy bump or cluster of bumps, usually on a hand but can occur anywhere on the body. Warts are usually skin-toned but may be lighter or darker in color and can contain black dots. Flat warts, which are smaller and smoother, can also appear anywhere on the body, but in children they most often show up on the face. Plantar warts show up on the soles of the feet. Most warts disappear on their own in a few months to a few years. Uncommon before age 2.

LSIL in 21-24 y/o. 25-29 year old. >30 year old
21-24y.o.: Repeat PAP in 12 mo
25-29 yo: refer for colpososcopy/biopsy.

30 if HPV (-) repeat PAP in 12 mo. If HPV (+) then colp / biopsy

Cellulitis
Skin infection involving deep dermis. Diffuse, acute infection of the skin marked by local heat, redness, pain, and swelling. Tx:. MRSA tx: Bactrim, Clinda, or Doxy. Non MRSA infection: Keflex

Anthrax tx
Doxycycline/Fluoroquinolones (Cipro)

pansystolic murmur
mitral regurgitation

During a well-child examination of an 18-month-old, premature tooth decay and inflamed gums, particularly of the maxillary incisors, are noted. The approach to this problem includes:

1.
referring the child to a dentist, assessing for bottle feeding and fluoride in local water.
2.
teaching the mother to brush the child’s teeth, encouraging vitamin supplements.
3.
recommending fluoride mouthwash and toothpaste, suggesting celery sticks for snacks.
4.
providing fluoride treatment, teaching dental hygiene, and eating foods high in calcium.
referring the child to a dentist, assessing for bottle feeding and fluoride in local water

A patient is referred with a diagnosis of diabetes mellitus, hypertension, and coronary artery disease. The patient is on both insulin and a beta blocker. Assuming that the patient will continue the beta blocker, it will be important to educate the patient on the recognition of hypoglycemia. Which symptom would be most indicative of hypoglycemia in this patient?
sweating

McMurray’s Test
audible/palpable click when knee is raised with tibia externally rotated and then straightened- one hand on knee other on foot

What drug class are erythomycin, azithromycin (Z-pack), clarithromycin (Biaxin)?
Macrolides

Cutaneous anthrax

Normal prostate
firm, smooth, nontender
about as firm as pressing in on the tip of your nose

What antibiotic is used for corneal abrasion:
Gentimicin ophthalmic solution

Young woman with breast mass. Preferred imaging
Ultrasound in young (less dense tissue, better image). Can tell if mass is fluid filled cyst or solid

HR of 3 year old
80-120

Raynaud’s Phenomenon tx
CCB

tx corneal abrasion with
gentamicin

Which of the following is typically observed in the GI system upon stimulation of the parasympathetic nervous system?

1.
No overall effect
2.
Increased sphincter tone
3.
Increased peristalsis
4.
Decreased secretions
increased peristalsis

An 88-year-old male presents with concerns about memory loss. He feels good, takes an aspirin daily, and has no chronic diseases. He lives alone, drives his own car, and manages his financial affairs. To evaluate his memory, which of the following tests should the nurse practitioner choose?
Folstein Mini-Mental State Examination

Lachman’s Test
drawer test for acl or pcl test knee at 20-30 degrees one hand on thigh other on proximal tibia, pull tibia forward to assess anterior motion of tibia to femur (intact ACL should prevent anterior movement)

HR of 6 year old
70-110

name first generation cephalosporins
Keflex

Contraindication for beta blockers?
Asthma, COPD, chronic bronchitis, Emphysema, bradycardia and AV block. Inhibits of bronchodilator response to beta agonists

Acute prostate
tender, boggy, indurated
about as firm as pressing in over your puffed out cheek

S/s of CHF:
s3, edema, dyspnea, JVD, x-ray reveals cardiomegaly, long standing hypertension, systolic murmur, tachycardia INSERT MORE HERE

Risk factors/findings for ovarian ca
Family history.
Should not ever be able to palpate an ovary, r/o US ovarian CA.
Risks: >50, early menarche, late menopause, obesity, family history, 1st prego after 35, or not ever prego.

Chronic Venous Insufficiency tx
Light exercise, stockings, Wt loss, elevate legs

feverfew is used for
migraines and menstrual cramps

Type 2 diabetes mellitus is characterized by which of the following?

1.
Decreased production of exogenous glucagon-like peptide
2.
Autoimmune beta-cell destruction
3.
Relative deficiency of dipeptidyl peptidase-4
4.
Decreased reabsorption of sodium-glucose transporters
decreased reabsorption of sodium-glucose transporters

Which of the following wet-mount results confirms a preliminary diagnosis of bacterial vaginosis?
Squamous epithelial cells with stippling appearance and indistinct borders, no lactobacillus rods, and many white blood cells

Apley’s Grind test
flex knee 90 degrees with patient prone put pressure on heel hands and rotates leg laterally and medially, repeat while pulling up on leg (+ = limited ROM or pain)

Prostate cancer
nodular, firm, nontender
usually malignant lesions not palpable until disease is advanced

Name second and third generation cephalosporins
Second- Cefaclor, Ceftin, Cefzil
Third- Rocephin, Suprax, Omnicef

When to start screening BP
3 years old – age, gender and height

Levothyroxin may worsen?
Osteoporosis

Natural estrogen-
Isoflavones

Adverse effects of beta blockers
Bronchospasm, bradycardia, depression, fatigue, ED, HF, hypoglycemia

Infective Endocarditis tx
Amoxicillin or macrolide

most common 2 pathogens for older adults residing in community
strep pneumo, H. flu

In pulmonary function testing, forced vital capacity represents the:

1.
maximum volume of air that can be forcefully exhaled after maximum inspiration.
2.
total volume of air that is exhaled after normal inspiration.
3.
total volume of air that the lungs can hold, minus the expiratory reserve volume.
4.
volume of air that can be forcefully exhaled in 1 second.
maximum volume of air that can be forcefully exhaled after maximum inspiration

Office spirometry performed with an albuterol nebulizer treatment can confirm a diagnosis of asthma because it indicates:
whether a patient has reversible airway obstruction.

Straight leg raise
lie supine, leg elevated off table produces radiating or sciatic pain – herniated disc

Name a quinolone antibiotic
Ciprofloxacin, ofloxacin

TB
Gold std Culture and stain

Which of the following is least likely to be noted in a 55 yo M who presents with bladder cancer?
A. textile worker for 25 years
B. 60 pk yr smoking history
C. Report of intermittent painless gross hematuria
D. report of recent-onset intermittent acute urinary retention
D. report of recent-onset intermittent acute urinary retention

Testicular torsion s/s?
Absent cremaster reflex, testicular pain, difficulty walking, nausea, MORE?

Ectopic pregnancy s/s, physical exam findings, increased risk for
Light to scant bleeding in 6-7 weeks/lower abd pain/pelvic pain. Intermittent cramping, if radiating to right shoulder think rupture.

Pain is worsen with SUPINE or with JARRING. Previous ectopic pregnancy, tubal ligation, PID. Anything that is causing scaring

what age should dyslipidemia risk assessment begin?
2 years old. 2, 4, 6, 8, 10 then annually

Ventricular Septal Defect tx
Lasix & Lisinopril

what class of anti-HTN meds is absolutely CONTRAINDICATED in bilateral renal artery stenosis and has been associated with acute renal failure
ACEs – With stenosed renal arteries, afferent flow cannot be increased, angiotensin II can actually become the only mechanism by which the kidney can increase filtration, and with severe stenosis, efferent arteriole constriction is crucial for maintaining some degree of filtration. ACE inhibitors would prevent conversion of Ang. I (which is inactive) to Ang. II, and therefore make everything worse by removing the kidney’s only remaining regulatory mechanism as well as cutting the perfusion pressure and eliminating what little renal function remains. With tubular cells that are already less effective due to damage from chronic ischemia, the resulting decrease in GFR would effectively accelerate kidney failure .

An adult male presents as first patient of the day after awaking about 5:00 AM with indigestion and chest pressure. On checking his pulse, he found no change from previous measurements. He states that, while waiting for the nurse practitioner to arrive, he began to feel a little clammy. A 12-lead EKG records the following patterns. On the basis of these EKG readings, the diagnosis is:

1.
anterior-septal acute myocardial infarction.
2.
lateral acute myocardial infarction.
3.
inferior acute myocardial infarction.
4.
anterior acute myocardial infarction.
inferior acute myocardial infarction

A 32-year-old patient reports a 6-month history of intermittent symmetrical swelling of the wrists and daily morning stiffness lasting 1 hour or more in and around other joints. What is the most likely diagnosis?
Rheumatoid arthritis

Pelvic Rock test
Screens for sacroiliac joint dysfunction. Place hands on bilateral anterior superior illac spines and attempt to rotate SI joint, or side lying apply pressure to ilium (+ = pain at SI joint)

CHD is the most common what in newborns?
congenital anomaly

Name a quinolone with Gram + coverage
Levaquin, moxifloxacin, gatifloxacin

ectopic pregnancy
Light scant bleeding in 6-7 weeks, lower abd pain, intermittent cramping, pain radiating to right shoulder and pain is worsen with supine position

Which of the following is a worrisome finding noted during pelvic exam on a 62 yo F?
A. flattening of the vaginal rugae
B. vaginal pH=5.6
C. ovary palpable on bimanual exam
D. scant white vaginal discharge
C. ovary palpable on bimanual exam

Alternate ABT for erythromycin for mycoplasma pneumonia?
macrolide such as azithromycin.

BPH physical findings, tx, when to take meds, evaluating PSA post tx,
BPH- Symmetrical rubbery and enlarged. All of BPH meds, take at bedtime. Hytrin is good for ppl with HTN and BP; If on Proscar must times PSA X2.

HCTZ should not be taken with digoxin
may increase risk of digoxin toxicity

During the past 24 hours, a 62-year-old has experienced abdominal pain that radiates to the back. The patient also reports several episodes of nausea and vomiting, a low-grade temperature, and a history of excessive drinking. Physical examination reveals a distended abdomen. Laboratory serum values indicate elevated alkaline phosphatase, amylase, and serum lipase. The most likely diagnosis is:

1.
alcoholic liver disease.
2.
acute mesenteric ischemia.
3.
viral hepatitis.
4.
acute pancreatitis.
acute pancreatitis

A 47-year-old patient presents with complaints of upper abdominal discomfort with nausea and burning after eating. The patient does not currently take any medications. The most likely differential diagnoses would include:
gastritis and peptic ulcer disease.

Prehn’s sign
lifting the scrotum relieves pain of epididymitis

Cholesterol <200 HDL >40
LDL <100
Triglycerides <150
Lifestyle modification first line
Niacin, fenofibrate

Name two sulfa drugs

  1. Trimethoprim-sulfamethoxazole (Bactrim, Septra)
  2. Nitrofurantoin (macrobid)

Herniated disc and sciatica
better with standing and worse with sitting

Alternate ABT for common conditions such as Pneumonia, Atypical pneumonia, MRSA, Impetigo, Bladder infection, Otitis media etc
INSERT HERE

Acute bacterial prostatitis: S/s, physical exam findings, labs, tx based on age
High fever, chills, suprapubic, perineal pain, radiates to back or rectum, s/sx of uti.
Prostate is warm and boggy.
UA/ Culture is definitive. CBC shift to left (band cells) UA, pyuria, hematuria.
Older than 35= cipro, levaquin 4-6 wk. other Bactrim.
<35-Rocephin 250mg IM and doxy 100 mg BIDx10d.

Innocent murmur clues
*Grade <2
*Softer intensity when sitting compared to supine
*Not holosystolic
*Minimal radiation
*Musical or vibratory quality

Urge incontinence
reports of strong sensation of needing to void that cannot be supressed, often has involuntary loss of urine
most common in older adults
Treatment: behavioral therapy, pharmacotherapy (antimuscarinics such as: Detrol,Ditropan, VESIcare, Enablex, Toviaz. Adverse effects = dry mouth, sedation, mental status change, particularly in higher doses.

Addison’s Disease tx
Steroid Cortisol

Erythema multiforme
a milder form of Stevens-Johnson syndrome that produces pink-to-red targetlike lesions, wheals, and blisters, with no mucosal involvement. The clinician should look for a history of antibiotic and other drug treatment such as sulfa drugs, penicillins, and other drugs. Hypersensitivity reaction from meds, allergy, or infection.

An otherwise healthy adult female presents complaining of vaginal burning and itching during urination. She has a social history of two sexual partners. She denies fever or vaginal discharge, but has noted a lesion on one labium. The nurse practitioner should order all of the following EXCEPT:

1.
wet mount and potassium hydroxide.
2.
dark-field microscopy of fluid from the lesion.
3.
nucleic acid amplification testing for gonorrhea.
4.
serology for Haemophilus ducreyi.
dark-field microscopy of fluid from lesion

A 2-month-old infant is presented for examination and immunizations. History includes an uncomplicated full-term delivery and hepatitis B virus immunization shortly after birth. Examination is unremarkable except for a diffusely erythematous (non-beet-red) macular rash in the diaper area, sparing the inguinal folds. No satellite lesions are noted. The infant’s diaper rash is most likely caused by:
contact dermatitis.

Romberg Test
Evals proprioception and cerebellar fxn; stand with feet together, arms at side, close eyes. + = loss of balance

Name two tetracycline drugs

  1. doxycycline
  2. minocycline (Minocin)

Lumbar stenosis
is aggravated by long periods of standing and walking, relieved by sitting and rest.

Stress incontinence
associated with lifting, coughing, sneezing, exercise, anything that causes intraabdominal pressure
more common in F, rare in M
Treatment: vaginal tampon, urethral stents, periurethral bulking agent injections, pessary use. Kegel exercise in younger, premenopausal patients. Pelvic floor rehab w/biofeedback, electrical stimulation and bladder training.

Side effects and uses of ortho-novum and other antibiotics:
INSERT HERE: spotting? amenorrhea etc

Generalized anxiety disorder
SSRI, SNRI, Wellbutrin. May do benzo for short time (for acute anxiety attack)

Pathologic murmur clues
*Grade >3
*Holosystolic
*Max intensity at LUSB
*Harsh or blowing quality
*Systolic clicks
*Diastolic murmurs
*Increased intensity in upright position
*Gallop
*friction rub

Meniere’s Disease tx
Meclizine (Antivert), diuretics, diazepam

how is MS dx
lumbar puncture

A 3-year-old female has been diagnosed with bacterial meningitis. She attends preschool daily at a local church day care program. The day before her diagnosis, she ate lunch with her mother at a local restaurant. For which contact(s) would chemoprophylaxis with rifampin be recommended?

1.
Preschool contacts in the past 7 days
2.
A friend of the patient’s sister who visited after school yesterday
3.
The checkout employee at the grocery store yesterday
4.
Diners at the restaurant during lunch the day the patient ate there
preschool contacts in past 7 days

A nurse practitioner is evaluating a 40-year-old patient suspected of having a pulmonary embolus. The patient complains of anxiety and cough. A stat chest X-ray is normal. Which of the following tests should the nurse practitioner perform next?
Helical CT pulmonary angiography

Finger to Nose test
Tests cerebellar function – patient alternately points from their nose to examiner’s finger

Barlow-Ortolani maneuver

  • To detect hip dislocation in newborn

What category of drugs are allowed for pregnant or lactating women? (A, B, C, D)
Category B
ex: pick acetaminophen over NSAID. Avoid nitrofurantoin and sulfa drugs during third trimester (hyperbilirubinemia)

sentinel nodes (virchow’s nodes)
Left superaclavicular area. They are the first lymph nodes that a cancer lesion will drain into. Therefore, when cancer is diagnosed, these nodes are biopsied to see whether the cancer has spread into the lymph system.

Functional incontinence
often occurs I presence of mobility problems, associated w/the inability to get to the toilet or the lack of knowing that there is a need to void.
Usually found in those with limited mobility or altered cognition. Worsens when there is limited availability of an assistant for toileting.
Treatment: Ameliorated by having an assistant who is aware of voiding cue available to help with toileting activities.

Orthonovum (estrogen based contraception) causes spotting? T/F
True

PTSD, major and minor depression, first choice meds?
SSRI Paxil (sex dysfunc causes). Zoloft, Celexa (Good for older few drug interactions) Lexapro. Gradually wean paxil.

Hep B tx
Peg-IFN, Interferon, ETV, TDF

diabetes can contribute to pregnancy induced HTN

Congenital dysplasia of the hip:

1.
more commonly affects both hips.
2.
occurs more commonly in females.
3.
is correctable at any age.
4.
typically self-corrects by 12 weeks of age.
more commonly affects both hips

A 65-year-old male presents to a clinic complaining of increasing fatigue, dyspnea on exertion, and ankle edema during the day. He has a history of mild hypertension, for which he saw his physician years ago. The physician advised the patient to decrease his salt intake. On physical examination, the patient is tachycardic, positive for jugular venous distention, and positive for S3, with a systolic murmur. Chest X-ray reveals cardiomegaly with vascular redistribution. The nurse practitioner’s diagnosis should be:
heat failure

Heel to shin test
Tests cerebellar function – runs heel of one foot along the shin of the opposite leg fairly quickly

What HTN medication can cause acute renal failure and is contraindicated in bilateral renal stenosis?
ACE inhibitors. How to monitor? Test renal function.

What is the preferred treatment for cutaneous anthrax and how do you get it?

  1. Ciprofloxacin 500mg BID for 60 days (8 weeks)
  2. Doxycycline 100mg BID if cipro allergy
  3. Comes from touching fur or animal skins that are contaminated with anthrax spores (not contagious)

Transient incontinence
occurs during an acute illness such as UTI, delirium, medication use, restricted activity
Treatment: discontinuation of offending medication.

Where should the examiners 2nd-5th fingers be placed to perform Ortolani’s
the greater trochanter

Acute serotonin syndrome, s/s , causes
s/s Dilated pupils, high fever, muscular rigidity, mental status changes, hyperreflexia/clonus, shivery.
causes: SSRI, MAOIs, TCA. Could be potentially life threatening

syncope
Blood glucose. Checking the blood glucose is indicated for patients with syncopal and near-syncopal episodes. The nurse practitioner should also perform a thorough history of the incident. Possible causes of syncope are cardiac arrhythmia, vasovagal, hypoglycemia, orthostatic hypotension, seizure, accidental fall, and others.

Hep C tx
Antivirals and PEG-IFN

Anticholinergic agents such as ipratropium (Atrovent) and tiotropium (Spiriva) are used in COPD primarily to:

1.
induce bronchodilation.
2.
decrease airway inflammation.
3.
expand the lung fields.
4.
treat hypoxemia.
induce bronchodilation

Trigeminal neuralgia manifests itself primarily with:
electric-shock-like unilateral facial pain.

Cullen’s sign
bruising or bluish discoloration and/or edema around the umbilicus = pancreatitis, ruptured ectopic pregnancy, aortic rupture, abdominal trauma

hold assistive device (like cane) on opposite side

Optic disk
normal examination has sharp margins, a yellowish orange to a creamy pink color, and round or oval shape. The ratio of veins to arteries is 3:2

Treatable causes of urinary incontinence
(DIAPPERS)
Delirium
Infection (urinary)
Atrophic urethritis and vaginitis
Pharmaceuticals (diuretics, others)
Psychologic disorders (depression)
Excessive urine output (heart failure, hyperglycemia due to undetected or poorly controlled DM)
Restricted mobility
Stool impaction

Acute bacterial sinusitis is treated with?
amoxicillin

Lumbar stenosis
Aggravated by long periods of standing and walking. Relieved by forward flexion, sitting and rest.

Club foot AKA
talipes equinovarus

sickle cell, thalassemia and leukemia can all cause
hepatosplenomegaly – but IDA wil not

The clinical presentation of placenta previa that develops during the third trimester includes:

1.
intermittent dark-red spotting.
2.
occult bleeding with abdominal or back pain.
3.
sudden onset of painless and profuse bleeding.
4.
painless occult bleeding that becomes visible upon onset of labor.
sudden onset of painless and profuse bleeding

Which of the following criteria differentiates a TIA from a stroke?
Absence of residual symptoms

Grey Turner’s Sign
bruising or bluish discoloration of the flanks indicative of retroperitoneal hemorrhage = pancreatitis

arcus senilis
gray-white arc or circle around the limbus of the iris that is common with aging. caused by lipid deposits deep in the edge of the cornea and is quite commonly present in elderly. However, it can also appear early in life as a results of hypercholesterolemia

Idiopathic Thrombocytopenia purpura tx
Initial: Glucocorticosteroids (prednisone)

Overactive bladder medication with highest anticholinergic effect and lowest
Highest is oxybutynin (Ditropan)
Lowest is mirabegron (Myrbetriq)
Not used to treat OAB: tamulosin (Flomax) or finsasteride (Proscar)

S/s of acute bacterial sinusitis versus viral?
Insert here. How long before treat? Alternative to amox allergy?

sciatica
sciatica is a form of radiculopathy and one of the most common causes is a herniated disc. So both are aggravated by long periods of sitting.

metatarus adductus
“c” shaped foot or “kidney bean shaped” foot without ankle deviation. in-toeing

Myerson’s Sign / Glabellar tap sign
tapping between eyebrows, above bridge of nose (glabella) produces a sustained blink response = Parkinson’s Disease

VSD sx
cyanosis and poor feeding – murmur is heard in tricuspid area

A 27-year-old male patient, who works as a janitor, presents with a 6-month history of an intermittent rash on his hands. History reveals itching and occasional burning. Examination reveals irregularly-distributed scaly maculopapular erythematous patches extending from the dorsum of the hand several inches up the forearms, and dry palms with no nail involvement. The most likely diagnosis is:

1.
contact dermatitis.
2.
scabies infestation.
3.
psoriasis.
4.
eczema.
contact dermatitis

A 20-year-old male presents with a 1-month history of a “bump on my testicle.” He reports being sexually active, with frequent episodes of unprotected intercourse with multiple partners. Which condition should be suspected?
Cancer of the testicles

Ishihara chart
A chart used to test for color blindness. (7 yr old)

Contraindications for Metformin
Significant renal & hepatic disease, Sepsis, Alcoholism, Hypoxia causing conditions (cardiac/pulmonary), dehydration, & advanced age.

Healthy 32 yo F who is taking combined oral contraceptives, is here to review the results of her recent liquid-based Pap screening that revealed atypical squamous cells of unknown significance(ASCUS) and high risk HPV +. No h/o previous abnormal cervial cytology, with her last screening aprox 2 yr ago. After discussing the significance of these findings, the most appropriate next step is to:
A. Advise that she return in 6-12 months for a repeat Pap w/HPV cotesting
B. obtain screening tests for N. gonorrhoeae and C. trachomatis infection.
C. referral for colposcopy
D. counsel that the usual cervical cancer screening guidelines​ should be followed
C. referral for colposcopy

Delayed ejaculation is caused by a common medication:
Paxil- SSRI

Treatment to prevent fracture in patient with low vitamin D high TSH low HCT

  • VITAMIN D 600-800, CALCIUM 1000-1200.

Nursemaid’s elbow
Annular ligament displacement over the radial head

Macrocytic Folate Deficiency tx
Folate, leafy green veggies, beans, liver, pasta, cereals

Anorexia nervosa occurs most commonly in which of the following?
1.
Painters
2.
Only children
3.
High-level athletes
4.
Individuals from large families
high level athletes

A 65-year-old female presents with shoulder and pelvic girdle pain for the past 6 months. She reports recent unintentional weight loss. On physical examination, there is pain on ROM, with no weakness noted. Laboratory studies show a low hemoglobin and an elevated sedimentation rate. Which of the following is the most likely diagnosis?
Polymyalgia rheumatica

Murphy’s sign
Deep pain on inspiration while fingers are placed under right rib cage = cholecystitis

Down’s syndrome patients can have
congenital heart defects, seizures, eye issues, hearing loss, obesity – NOT failure to thrive

Straight leg raise
This test is performed by laying on your back with both legs straight and the physician raises one of your legs upward, keeping the knee straight. This is done to help find the reason for low back and leg pain. (sciatic nerve/herniated disk)

Which of the following is demonstrated to provide the most symptom relief in treating vasomotor symptoms? (hot flashes)
A. clonidine
B. paroxetine
C. conjugated estrogen
D. venlafaxine
C. conjugated estrogen

Uncomplicated chlymydia: may present with?
No clinical s/s:
Difference between complicated and uncomplicated chlamydia?

Morton’s neuroma
Morton’s Neuroma- do mulder test. “pebble, burning, numbness” ¾ metatarsals. PUNT TO pod

Mulder sign or foot squeeze test – Sharp pain b/w 3rd & 4th toe when foot is squeezed in pt w/ MORTON NEUROMA

Scoliosis

10 degree curve. Use Adam’s forward bend test

hyperinflation of lungs can be seen on chest X-ray in patients with asthma

A frail elderly patient presents with constipation. Which of the following normal physiologic changes seen with aging is the most likely cause?

1.
Decreased bowel muscle tone
2.
Increased bile secretion
3.
Increased absorption of calcium
4.
Decreased pancreatic secretions
decreased bowel muscle tone

An 85-year-old patient relates that on the way to an annual physical examination, the patient suffered a sudden loss of vision in the right eye characterized by “a bunch of lights” and a feeling that “a curtain came down.” The most likely diagnosis is:
retinal detachment.

Psoas sign
RLQ pain with passive right leg elevation off table or with right thigh extension side lying = appendicitis

Lachman’s Test
ACL instability of the knee

What is Galaezzi sign
one femur appears shorter when infant is supine

Risk Factors and Cancer:
Obesity and history of PCOS: endometrial cancer
HPV 16 and/or 18: Cervical cancer
BRCA 1 and BRCA2 gene mutation: Ovarian cancer
Unexplained abdominal bloating and constipation: Ovarian cancer
Cancer:
Abnormal vaginal bleeding: Cervical and Endometrial cancer
Often without significant signs or symptoms: All Cervical, Endometrial and Ovarian cancer
Pap test: Cervical cancer
No specific screening tests available or recommended: Ovarian and Endometrial cancer

Treatment for chlamydia?
INSERT HERE: azithromycin. Alternative tx?

Tibial overuse stress syndrome
“inner border” painful on palpation.

DO bone scan or MRI cuz a plain X-ray won’t show a stress fracture.

FOLLOW RICE.

Osgood-Schlatter disease
*Anterior knee pain
*Osteochondritis of the tibial tubercle
*Common during growth spurt
*Pain reproduced with extending the knee against resistance
*Straight leg raise is painless
*RICE and quadricep strengthening

What is Acute Dacryostenosis? How to dx?
redness warmth tenderness and swelling of one lacrimal duct, complication of orbital cellulitis
Dx: Lacrimal sac massage & culture of discharge then treat with atb.

A patient reports to your office, having been struck on the right cheek with a baseball bat. On examination, you notice tenderness and swelling over the cheek. The patient is unable to gaze laterally with his left eye, and he complains of double vision when he attempts to do so. This is most likely associated with:

1.
retinal hemorrhage.
2.
orbital rim fracture.
3.
subarachnoid hemorrhage.
4.
periorbital cellulitis.
orbital rim fractures

A 70-year-old male presents with paresthesia of the lower extremities. On examination, the patient appears pale and shows a decreased vibratory sense. Laboratory tests reveal elevated indirect bilirubin; Hct = 30%; mean corpuscular volume (MCV) = 120 µm3 [normal = 87-103 µm3]. The most likely diagnosis is:
pernicious anemia

Obturator sign
RLQ pain with internal rotation of flexed right thigh = appendicitis

Kerley’s B lines on X-ray indicate
Pulmonary edema

MMR and pregnancy
Don’t get pregnant within the next 4 weeks

COPD patients may prolong their life by the use of?
Oxygen

Rheumatoid Arthritis s/s
Early morning stiffness, sausage joints. Symmetrical involvement. Longer stiffness than OA. Joint space narrowing. Pain, warm, tender, swollen, things.

Hip pain causes?
Legg-Calve-Perthes disease, SCFE, irritable hip

Hydrocele
Collection of serous fluid that causes painless scrotal swelling, easily recognized by transillumination.
Common in older men.

What is Caput Succedaneum
Diffuse edema of scalp due to intrauterine pressure (Cone head)

A 7-year-old child presents with group A streptococcal infection confirmed by throat culture. Past history includes treatment for positive streptococcal infection with erythromycin (EryPed oral suspension) 3 weeks ago. What is the most appropriate next intervention?

1.
Treat with amoxicillin for 10 days.
2.
Obtain culture for all household members.
3.
Treat with azithromycin (Zithromax) for 10 days.
4.
Intramuscular Penicillin.
treat with amoxicillin for 10 days

A 14-year-old patient who fell on an outstretched hand complains of proximal forearm pain. X-ray reveals a positive fat pad sign, and the patient is unable to fully extend the elbow. No definitive bony changes are seen on X-ray. The most likely working diagnosis is:
radial head fracture

Rovsing’s Sign
RLQ pain when pressure is applied to the LLQ = appendicitis

Depo provera can cause
menstrual irregularities and amenorrhea

caude equina syndrome

Varicocele
A palpable “nest of worms” scrotal mass that is only evident in a standing position.

COPD- other points of interest?
INSERT HERE

Rheumatoid Arthritis tx
: NSAIDS, steroids, DMARDS, TNF.

Legg-Calve-Perthes Disease
*Osteonecrosis of the capital femoral epiphysis due to interrupted vascular supply
*Common 3-12 years and more in boys
*Pain in hip or radiation to knee.
*Limp
*Trendelenburg Test – positive pelvic tilit

when rx hormones, consider fam hx of what cancers
uterine and breast

A 16-year-old female adolescent is brought to the nurse practitioner’s office by her mother, who is concerned about her daughter’s recent weight loss. History reveals that the daughter was consistently in the 50th percentile for weight, but is now in the 10th percentile. The mother states that she is concerned that her daughter is purging herself after meals since she often goes to the restroom after eating and remains there for a long period of time. The daughter denies any self-induced vomiting, starvation or excessive activity. She does state that she jogs five miles a day and is in good condition. In addition to a complete blood count with differential, which of the following laboratory tests will be most helpful for further assessment?

1.
Electrolytes, fasting blood sugar
2.
Creatine phosphokinase, follicle-stimulating hormone (FSH)
3.
Electrolytes, blood urea nitrogen (BUN)/creatinine, urinalysis
4.
Electrolytes, FSH, stool for occult blood
electrolytes, blood urea nitrogen (BUN)/creatinine, urinalysis

A 72-year-old female presents with a medical history unremarkable except for the occasional headache. She enjoyed emotional health throughout marriage and childrearing, although her family history is significant for strokes and nervous breakdowns. Since her husband died a year ago, however, she has been waking every morning at 3:00 A.M., is reluctant to go out although she has family members living nearby, and feels a continual urge to sleep. She cannot discuss her late husband without crying. Differential diagnoses would include:
depression, unresolved grief, and hypothyroidism.

Kerley B lines
Horizontal lines on CXR = heart failure

Phalen’s test
The test is performed by pushing the back of the hands together for 1 minute. This compresses the median nerve within the carpal tunnel. Characteristic symptoms (burning; tingling; numbness over the thumb, index, middle, and ring fingers) convey a positive test result.

Hirschsprung’s disease
A condition of the large intestine (colon) that causes difficulty passing stool.in first 48 hours of life, meconium

Testicular torsion
Characterized by scrotal pain and loss of the cremasteric reflex (Stroke inner thigh for contraction of crenatic muscle- testicle raise).
Urologic emergency.

Serous in middle ear may remain in Acute otitis media for weeks after the infection: any treatment needed?
no.

Swan Neck Deformity
deformed position of the finger, in which the joint closest to the fingertip is permanently bent toward the palm while the nearest joint to the palm is bent away from it (DIP flexion with PIP hyperextension). Associated with RA

SCFE
*hip/knee pain
*limp
*common in teens
*Trendelenburg test

Korsakoff Syndrome
chronic thiamine deficiency

An adult female who is homeless presents for an initial obstetric visit at 34 weeks of pregnancy. She is diagnosed with Chlamydia trachomatis infection. What complication will her newborn be most at risk?

1.
Conjunctivitis
2.
Hearing loss
3.
Pneumonitis
4.
Meningitis
conjunctivitis

A 66-year-old patient presents with bilateral otitis media with effusion and white patches in the mouth that do not rub off when wiped with a 4 × 4. The patient should be evaluated for:
HIV infection

Levine’s sign
Clenched fist over chest sign = ischemic chest pain

nuva ring does not affect menstrual cycle

Osteopenia is defined as T-score
-1.0 to -2.5

Phimosis
The foreskin cannot be pulled back to expose the glans. head of penis
Potential urologic emergency.

Acute otitis media tx?
alternative? INSERT HERE

Bouchard’s nodes
Swelling to PIP, in RA there will be Bouchard’s nodes (and not Heberden’s nodes)

Transient Synovitis of the Hip
*”irritable hip”
*limp
*hip pain
*small effusion on US
*h/o URI 1 week prior
*no x-rays
*resolves in 1-2 weeks

implant can help with cramps

A 2-year-old male presents for reevaluation. Two days ago, he had four episodes of vomiting and six diarrheal stools. On physical examination today, his vital signs reveal P = 120 and capillary refill of 3 seconds. The patient’s eyes are sunken and his extremities are mottled and cool to the touch. The treatment plan would include:

1.
start oral electrolyte mixture every hour.
2.
hospitalization for IV fluid replacement.
3.
giving the child loperamide for each diarrheal stool.
4.
catheterizing the child to obtain accurate urine output.
hospitalization for IV fluid replacement

A 17-year-old female has never had her menses. She is at Tanner stage III of sexual development. Her physical examination is completely normal, and her weight is appropriate for her age and height. What is the most likely diagnosis?
Primary amenorrhea

Palm Sign
palm of the hand to the chest = ischemic chest pain

Osteoporosis is defined as T-score
-2.5 and less

Dystonia
involuntary repetitive muscle movements muscle spasms

Paraphimosis
Retracted foreskin that cannot be brought forward to cover the glans.
Potential urologic emergency.

S/s of retinal detachment?
Feels like curtian coming over vision, flashes of light OTHERS? TX?

Osteoarthritis s/s, first line tx
Large weight bearing joints. Early morning stiffness with inactivity. Has both nodes. FIRST LINE Acetaminophen.

Roseola
Human herpesvirus 6. High fever for 2-4 days, then abrupt cessation of fever with appearance of maculopapular rash but not on face

itraconazole – oral

A nurse practitioner places a 76-year-old patient on nifedipine (Procardia) 10 mg t.i.d. for angina. The patient is unable to remember to take the medication at the scheduled times. The practitioner should:

1.
discontinue the issue with the patient’s daughter.
2.
change the dose to extended release 30 mg daily.
3.
reinforce the importance of taking the medication.
4.
increase the dosage to 20 mg b.i.d.
change the dose to extended release 30mg daily

According to the American Diabetes Association, the newest standard for determining the presence of diabetes mellitus based on fasting plasma sugar level is a value equal to or greater than:
126 mg/dL.

Arm Sign
touching the left arm with right arm = ischemic chest pain

Thayer-Martin culture
Used to diagnose gonorrheal pharyngitis or proctitis

Dyskinesia
abnormal involuntary muscle rigidity

Cryptorchidism
Testicle located in inguinal canal or abdomen. Undescended testicle.

S/s of epididimitis?
???? Prinse test

Osteoarthritis recommended exercise
EXERCISE: Isometric exercises for knee OA. Non-weight bearing, like biking, swimming, stationary bike.

Fifth Disease
Parvovirus B19. “Slapped cheek” rash. Lacy. Macular rash.

griseofulvin
tinea capitis

A pediatric male is present with his mother, who states that he is extremely restless at night and constantly scratches his anal area. Which diagnostic procedure would confirm a diagnosis of pinworm infestation?

1.
The patch test
2.
Examination of bed linens
3.
Examination of transparent tape sample from perianal skin
4.
Blood tests for anemia and eosinophilia
examination of transparent tape sample from perianal skin

A 38-year-old male experiences a sudden onset of acute abdominal pain radiating to his groin. He is having difficulty walking and is nauseated. He denies any trauma or previous groin injuries. Examination reveals diffuse swelling of the left testicle and negative cremasteric reflex. The most likely diagnosis is:
testicular torsion.

Pointing Sign
pointing to chest with 1 finger = non-ischemic chest pain

Positive strep allergic to PCN
clarithromycin (biaxin) 250 mg bid x 10 days

Symptoms of initial hearing loss in adults
Loss of high frequency tones & harder to hear consonants than vowel sounds

Current guidelines recommend initiating antiretroviral therapy (ART) when?
in all individuals with HIV-1 infection, including those with early HIV-1 infection and all HIV-infected women, including pregnant women. It should be started regardless of the CD4 T lymphocyte cell count.

Varococele?
Sack of worms

Lachman’s: how to do, what does it test for
flex knee to 30 degrees, slightly externally rotate tibia, stabilize femur, pull up on tibia. If feel laxity (more forward movement) then there is laxity of ACL (instability of knee joint)

Rubella
Rubella virus. Maculopapular rash. Looks like measles rash. Remarkable lymphadenopathy. Macules on soft palate (Forchheimer spots)

miconazole – topical

Which of the following microorganisms are most frequently associated with acute bacterial rhino-sinusitis?

1.
Staphylococcus aureus and Mycoplasma pneumonia
2.
Staphylococcus aureus and Methicillin Resistant Staph aureus
3.
Streptococcus pneumoniae and Haemophilus influenzae
4.
Streptococcus pneumoniae and Pseudomonas aeruginosa
strep pneumo and haemophilus influenzae

Akathesia
intense need to move, restlessness

Babinski’s Sign
big toe remains extended or extends itself and other toes fan out when the sole of the foot is stimulated (normal in children until age 2). If seen child over 2 years or adult = CNS disorder (ALS, brain tumor/injury, meningitis, MS, spinal cord injury/defect, stroke)

A 10-month-old presents with a rash, runny nose, and cough. Examination reveals a cluster of tiny white papules with an erythematous base on the buccal mucosa. What does this suggest?
Measles

Jarisch-Herxheimer reaction
Infections caused by a spirochete such as syphilis or lyme. They dying bacteria release antigens that causes an host reaction. (occurs treating early stage syphilis. Symptoms are headache, myaligas, rigors, sweat, hypotension, and worsening of rash. Resolves on its own.

Cancer of testes?
nodule on teste
OTHERS?

Anterior drawer sign
pulling up on tibia more than expected shows tear/laxity in ACL. Flex hip to 45 degrees, flex knee to 90 degrees – sit on foot.

Rubeola (measles)
Rubeola virus. Maculopapular “brick red” rash. Starts on head and neck, spreads centrifugally to trunk and extremities. reportable disease. 3 C’s (conjunctivitis, coryza, cough). Koplik spots (1-3mm whitish, bluish or gray elevations on the buccal mucosa and hard/soft palate)

What is the goal of antiretroviral therapy (ART)?
suppress plasma viral RNA to undetectable levels and can be important in preventing and managing opportunistic infections.

Akinesia
reduced voluntary muscle

The parent of a 13-year-old diagnosed with seizure disorder calls to report that the child is exhibiting symptoms of a cold, but has no fever. The nurse practitioner should advise the parent that the development of fever may:
lower the seizure threshold.

Battle Sign
bruising around the mastoid process (behind ears) = serious head / brain injury

A child is being treated with methylphenidate (Ritalin) for attention-deficit disorder. The following are all side effects of Ritalin EXCEPT:

1.
muscle cramps
2.
mild irritability.
3.
increased heart rate.
4.
slight increase in growth velocity.
slight increase in growth velocity

Dx: Of BV
Amsel criteria with microscopy: (need 3)
-white thick adherent discharge
-vaginal pH >4.5
-positive whiff-amine test, KOH
-clue cells on saline wet mount (greater than 20%) – epithelium, “bread crumb dip cells”
gram stain, DNA probe. (Epithelial cells dotted with with large numbers of bacteria that obscure cell borders)

S/s of gonorrhea?
Treatment?

Posterior drawer sign
flex knee 90 degrees. sits on toes of the tested extremity to help stabilize it. The examiner grasps the proximal lower leg, approximately at the tibial plateau or joint line, and attempts to translate the lower leg posteriorly. The test is considered positive if there is a lack of end feel or excessive posterior translation

Chickenpox
Herpes virus. Vesicular lesions on erythematous base appearing in crops

16-24 hours sexually active screen for chlamydia is reportable There is also EPT= expedited partner therapy
additional testing= for STI risk reduction
GC/Chlamydia
HIV
syphilis
HAV
HBV
HCV
HSV-2

Agnosia
inability to recognize familiar people or objects

A geriatric female presents for her annual examination. She has been on antihypertensive medications for over 20 years, with good control. Laboratory values are within normal ranges. The nurse practitioner is concerned about the patient’s cardiac health risks, due her to weight and her waist circumference. According to the AHA guidelines, which of the following goals is expected for this patient?

1.
Physical activity for 60 minutes daily, for a minimum of 6 days a week
2.
Physical activity for 30 minutes daily, 7 days a week
3.
Physical activity for 30 minutes daily, for a minimum of 5 days a week
4.
Physical activity for 60 minutes daily, 7 days a week
physical activity for 30 minutes daily, for a minimum of 5 days a week

After a 3-week camping trip, an 11-year-old is seen for a target lesion with central clearing, located in the inguinal area. The patient has had a severe headache, malaise, fatigue, and generalized musculoskeletal pain for several days. Pharmacologic management of this condition includes:
doxycycline (Doryx).

Blue Dot Sign
tender nodule with blue discoloration on the upper pole of the testicle = testicular torsion

vulvovaginitis or STI dx with clinical findings or microscopic examination
1-Bacterial Vaginosis
2-Candid vulvovaginitis
3-Nongonococcap cervicitis/vaginitis

A-Clue Cells with alkaline PH
B- Pseudohyphae
C- Abundant White cells
BV= Clue cells with alkaline ph
Candida Vulvovaginitis= Pseudohyphae
Nongonococcal cervicitis/vaginitis= Abundant white blood cells

Legal scope of practice
defined by the nurse practice act in the state where you practice.

Torsion of testes?
S/S

McMurray’s test – how to do, what does it test for
McMurrays test- “CLICK” medial meniscus (valgus) EXAM. LateRal meniscus (vaRus).

With the patient supine the examiner holds the knee and palpates the joint line with one hand, thumb on one side and fingers on the other, whilst the other hand holds the sole of the foot and acts to support the limb and provide the required movement through range. The examiner then applies a valgus stress to the knee whilst the other hand rotates the leg externally and extends the knee. Pain and/or an audible click while performing this maneuver can indicate a torn medial meniscus. To examine the lateral meniscus the examiner repeats this process from full flexion but applies a varus stress to the knee and medial rotation to the tibia prior to extending the knee once again.

Herpangina
Coxsackie A virus

Astereognosis
inability recognize object placed in palm

An 18-year-old patient presents with complaints of maxillary facial pain and yellow nasal discharge for 14 days. What is the appropriate initial pharmacologic intervention?
Amoxicillin-Clavulanate (Augmentin)

Cremaster Reflex
lightly stroke superior and medial thigh – normal response is elevation of the ipsilateral testicle = negative elevation seen in testicular torsion (not 100%)

The management of COPD in the elderly is best guided by:

1.
spirometry.
2.
arterial blood gases.
3.
radiologic imaging.
4.
symptomatology.
symptomatology

Osteoporosis
Kyphosis causes curvature of the spine that causes rounding of the back, slouching posture. Severe thinning of bones.

Hand, foot and mouth
Coxsackie A virus

An intervention in vulvovaginitis-
1-Candida vulvovaginitis
2-Trichomoniasis
3-BV

A-Clotrimazole Cream
B-Oral Metronidazole
C-Metronidazole gel
D-Clindamycin cream
1-Candida vulvovaginitis
2-Trichomoniasis
3-BV

A-Clotrimazole Cream= Candida
B-Oral Metronidazole= Trich and BV
C-Metronidazole gel= BV
D-Clindamycin cream= BV

Rotator cuff tendonitis
after repetitive overhead activities (throwing, raking), abduction and elevation of shoulder joint worsen symptoms.

disturbed sleep because of pain, arm weakness, dull ache

Pt with pulmonary embolus suspected. After x ray what is next test?
helical CT pulmonary angiogram

Scarlet fever
exotoxin rash secondary to group A strep. Sandpaper like rash that ultimately desquamates

Hook test
bicep tendon rupture, normally with the elbow in flexion and arm supinated, you can hook your finger under the biceps tendon at the level of the AC and it will meet with resistance as you pull your finger against the tendon. A distal bicep tendon tear means that your finger will not feel resistance or be able to hook under the tendon in a hook test. (positive).

Symogi effect v dawn phenomenon:
Symogi: hypoglycemia causes hyperglycemia
Dawn phenomen: increase in insulin due to growth hormones, cortisol etc

Treatments for STI- matching

External genital warts in a 25 yo adult
Gonococcal urethritis in a 22 year old adult
Syphilis in a 45 yo adult
External warts in a 28yo pregnant woman

Ceftriaxone
Injectable PCN
Imiquimod
Trichloroacetic acid TCA
Ceftriaxone= Gonococcal urethritis in a 22 year old adult
Injectable PCN= Syphilis in a 45 yo adult
Imiquimod= External genital warts in a 25 yo adult NEVER in A PREGNANT WOMAN

Trichloroacetic acid TCA= External warts on the 25 yo and thee 28 yo who is pregnant

Ataxia
difficulty cording voluntary movement

Finkelstein’s Test
grasp the thumb and ulnar deviate the hand sharp and quick towards pinky = + De Quervain’s tenosynovitis

A 40-year-old male presents with a 2-week history of rectal pain and itching. He reports a past history of constipation and finding spots of bright red blood on toilet paper several times a week. Rectal examination reveals a tender, swollen, bluish, ovoid mass. The stool guaiac test is negative. Which of the following actions should the nurse practitioner take?
Prescribe bulk-forming agents and hydrocortisone suppositories.

A young, athletic adult presents with shoulder pain and inability to raise the arm above the shoulder. The most likely diagnosis is:

1.
frozen shoulder syndrome.
2.
cervical radiculopathy.
3.
rotator cuff tear.
4.
polymyositis.
rotator cuff tear

Thiazide diuretics
Favorable effect in pts with osteopenia and osteoporosis. They slow down the kidney’s excretion of calcium and increase distal tubule Ca+ reabsorption. Decreases bone demineralization.

Apraxia
difficulty to remember learned motor skill

A 50-year-old male presents with bruises and abrasions on the left leg and forearm, claiming to have had a number of minor accidents at home and at work. The patient reports general feelings of depression, forgetfulness, and, despite the fact that he is regularly employed, a mounting collection of unpaid and overdue bills. In taking the patient’s history, a significant pattern of parental alcohol abuse, stroke, TIA, and MI emerges. During questioning to elicit a more thorough diagnosis, the patient becomes hostile and leaves the clinic without further treatment. The most likely explanation for this behavior is:

1.
Alzheimer’s disease.
2.
chronic alcoholism.
3.
cancer.
4.
carotid artery stenosis.
chronic alcoholism

A nurse practitioner is suturing a simple laceration on an 11-year-old patient. The use of lidocaine with epinephrine is contraindicated in all of the following areas EXCEPT the:
scalp

Homan’s Sign
pain in the calf on forceful and abrupt dorsiflexion of the patient’s foot at the ankle = possible DVT

Which signs and symptoms are associated with-epididymoorchitis
acute bacterial prostatitis
or both

Irritative voiding symptoms
perineal pain (between scrotum and anus)
Prehn’s sign
Usually presents with fever
Scrotal swelling
Irritative voiding symptoms- BOTH
perineal pain (between scrotum and anus)- Prostatiits
Prehn’s sign =Epididymoorchitis
Usually presents with fever=BOTH
Scrotal swelling-Epididymoorchitis

Lachman’s maneuver and sign
Knee joint laxity. Suggestive of ACL damage of the knee.

How to test s1 nerve function?
achillies tendon reflex

bicep tendon rupture – looks like
bicep is curled up into a giant ball

herpangina
painful vesicles on the soft palate and mouth

hand foot and mouth resolves when
2-3 days

blood tests for sickle cell
screening is CBC, diagnosis is HGB electrophoresis

  • HbS and HbF elevated, Reticulocytosis, hemolytic anemia
  • Normocytic/Normochromic
  • shortened life-span of 10-20d (norm 120d)

How to test s2 function? S3, s4? etc?
?

Cancers of the Female Reproductive Tract

  • cervical cancer caused by HPV types=abnormal vag bleeding, vag discomfort during intercourse, malodorous dc and dysuria- symptomatic w 1st abnormal pap
    pap test followed by colposcopy and biopsy
    pelvic ct/MRI or PET
    Tx based on disease progression surgery radiation chemo
  • uterine or endometrial cancer- older age , estrogen tx, nulliparity, obesity, tamoxoifin use, DM family hx-abnormal vag bleeding, Diagnostics – transvag ultrasonography hydroutrsonography endometrial biopsy and D&C
  • ovarian cancer- post menopausal nuliparity 1st preg after 35 fertitlity drugs Braca + has shared etiology with breast ca
    minimal non specific symptoms bloating bladder pressure constipation vag bleeding indigestion, SOB, lethargy weight loss
    Diagnostics- CT w/ contrast of abdomen,
    MRI,US tumor markers 125
    surgery followed by chemo

Drawer sign
A test for knee stability. A diagnostic sign of a torn or ruptured ligament. The positive anterior drawer sign is the test for the anterior cruciate ligament (ACL). The posterior drawer sign is the test for the posterior cruciate ligament (PCL).

Anomia
form of aphasia, unable to recall names of everyday objects

Pel Epstein fever
cyclic fever = rarely seen in Hodgkin’s lymphoma

A 16-year-old female in the first month of taking Ortho-Novum 7/7/7 complains of midcycle spotting. She has not missed any doses and uses no other medication. Which of the following is appropriate?
Providing reassurance

A 28-year-old patient presents with profuse yellow vaginal discharge, odor, and local irritation. She reports completing a 7-day course of oral metronidazole (Flagyl), 500 mg b.i.d. 4 weeks ago. She has had multiple recurrences over the last 18 months. Microscopic examination reveals presence of clue cells. What is the most appropriate treatment intervention?

1.
Oral metronidazole (Flagyl), 500 mg b.i.d., plus metronidazole vaginal gel (MetroGel), for 5 days
2.
Treat both patient and partner(s) with oral metronidazole (Flagyl), 500 mg b.i.d., for 7 days
3.
Metronidazole vaginal gel (MetroGel) twice weekly for 4 to 6 months
4.
Treat patient with metronidazole vaginal gel (MetroGel) b.i.d. and partner(s) with tetracycline, 250 mg q.i.d., for 7 days
Treat both patient and partner(s) with oral metronidazole (Flagyl), 500 mg b.i.d., for 7 days

Aphasia
difficulty verbalizing

An adult female who recently returned from a business trip to Japan presents for a recheck appointment. The only remarkable laboratory result is for thyroid-stimulating hormone (TSH), at 0.3 microunits/mL (normal = 0.4-6 microunits/mL). The patient reports that her neck hurts; examination reveals thyroid tenderness. Which of the following laboratory tests should the nurse practitioner order now?

1.
Triiodothyronine (T3) and free thyroxine (FT4)
2.
Triiodothyronine (T3) only
3.
Triiodothyronine (T3) resin uptake assay
4.
Triiodothyronine (T3) and free triiodothyronine (FT3)
Triiodothyronine (T3) and free thyroxine (FT4)

Research findings have shown that, in order to improve the longevity of a patient who has COPD, the treatment of choice is:
oxygen

vaccination for sickle cell pts
all vax given Preventive strategies that decrease the risk of infection are the routine use of daily antibiotics until five years of age, immunization of children with the 7-valent pneumococcal conjugate vaccine in addition to the 23-valent polysaccharide pneumococcal vaccine, annual influenza vaccination after six months of age, and meningococcal vaccination after two years of age

Finkelstein’s test
identifies deQuervain’s tenosynovitis which is caused by an inflammation of the tendon and its sheath, which is located at the base of the thumb. Test is positive if there is pain and tenderness on the wrist (thumb sign) upon ulnar deviation.

Lidocaine and epinephrine can be used on fingers, earlobes, nose?
False- may lack blood supply due to epinephrine.

match the following cancer types

1-cervical cancer

2-endometrial cancer

3-ovarian cancer

A-Risk factors include obesity and personal history of PCOs (do not shed lining of uterus obesity increases estrogen

B-Long term infection with HPV19 and or-18

C-BRACA1 and BRACA2 gene mutation

D-presentation includes unexplained abdominal bloating and constipation

E-Presentation includes abnormal vaginal bleeding

F-Often without significant clinical signs and symptoms until later disease

G-Screening include PAP

No specific screening tests available or recommended
Risk factors include obesity and personal history of PCOs (do not shed lining of uterus obesity increases estrogen= Endometrial Ca

Long term infection with HPV19 and or-18+Cervical Ca

BRACA1 and BRACA2 gene mutation= Ovarian Ca

presentation includes unexplained abdominal bloating and constipation=Ovarian

Presentation includes abnormal vaginal bleeding= Cervical and Endometrial Ca

Often without significant clinical signs and symptoms until later disease=Cervical, ovarian and endometrial

Screening include PAP=Cervical

No specific screening tests available or recommended =Endo and ovarian ca

otitis media
*Cloudy, bulging TM with impaired mobility
*Effusion – fluid collection w/o infection
*PCV has decreased AOM
*Viral – RSV, influenza
*Bacterial – strep pneumoniae, H. influenzae, M. Catarrhalis

McMurray Test
knee pain and click upon manipulation of the knee, injury to medial meniscus, (KNEE FEELS LIKE LOCKING UP) DX gold standard test for joint damage is the MRI (supine raise knee twisting knee inward/outward while extending in/out)

In treating a pregnant female with migraine headaches, which of the following drugs is in Category X?

1.
Ergotamine tartrate (Ergomar)
2.
Sumitriptan succinate (Imitrex)
3.
Frovatriptan (Frova)
4.
Amitriptyline (Elavil)
ergotamine tartate (ergomar)

Sacrcopenia
loss of muscle mass related to aging

A routine laboratory assessment of a 12-year-old patient with a family history of thalassemia and anemia reveals Tanner stage II presentation and Hct = 35%. In addition to a complete blood count (CBC), the nurse practitioner should order which of the following?
Hemoglobin electrophoresis

Cervical Screening Guidelines
screening should begin at age 21
21-65 it should be every 3 years.
21-29 every 3 yrs.
30-65 – every 5 + HPV.

65 after 3 normal paps – and negative HPV or terminate paps
NO NEED FOR ANNUAL SCREENING FOR AVERAGE RISK WOMEN AT ANY AGE.
WOMEN WITH HYSTERECTOMY with cervix removed – NO SCREENING.

scarlet fever: s/s and treatment and alternative ABT for those who cannot accept 1st line ABT.
???

lead poisoning anemia type, s/s
microcytic hypochromic anemia. s/s n/v, fatigue, loss of appetite, abdomen and joint pain, slowed growth, mental disability.

AOM management

  • <6 months: ABX
  • 6 months to 2 years – ABX ASAP if bilateral or severe illness. Otherwise observe if unilateral or mild
  • >2 years – ABX if severe, observe if not severe or uncertain of diagnosis
  • Amoxicillin.
  • Recent ABX – Augmentin
  • Mild PCN reaction – cefdinir, ceftriaxone, cefuroxime, cefpodoxime
  • Severe PCN allergy – Zpack or clindamycin

Prostatitis- first test to dx?
urine and culture.

Varus stress test (knee)
LCL ligament stability — test should be done at both 0 and 30 degrees.

1) Examiner places fingers of one hand over the lateral joint line assessing for increase in joint line opening while applying a varus stress to the other.
2) Must be done bilaterally for comparison in order to asses laxity/disruption of ligament.

Positive test is that one joint line is opening is larger than the other side. Indicates possible LCL ligament laxity on the side of the larger joint line opening.

Cachexia
underlying illness involving marked weight loss and muscle loss

A common rule to follow when prescribing many medications for the elderly is to:
start at a lower dose than what is commonly prescribed for adults, and increase the dose slowly.

A nurse practitioner is evaluating an infant for possible colic. Which of the following could indicate the need for a more extensive evaluation?

1.
3-oz (85 g) weight gain over the past 2 weeks
2.
Stool negative for occult blood
3.
Moist mucus membranes and flat fontanels
4.
Onset at 4 weeks of age
3-oz (85 g) weight gain over the past 2 weeks

thalassemia alpha – which population
African, Asian

Which of the following would be an appropriate alternative to erythromycin therapy in an 18-year-old patient with Mycoplasma pneumoniae infection?
Azithromycin (Zithromax)

Which of the following is a hallmark of lumbar spinal stenosis?

1.
Incontinence of bowel and/or bladder
2.
Point tenderness of the lumbar spine
3.
Leg, buttock or back pain precipitated by walking.
4.
Bilateral leg pain with sitting.
leg, buttock or back pain precipitated by walking

thalassemia beta – which population
African, Mediterranean, Middle Eastern

Macular Degeneration
loss of center vision, loss of visual acuity, contrast sensitivity but still have peripheral vision, may find Drusen bodies

Valgus Stress Test
application of a medial force to the lateral aspect of a joint in an attempt to create a gap in the medial joint line, thereby testing the stability of the medial aspect of the joint. Medial collateral ligament

Prostitis s/s, tx?
???

thalassemia type of anemia

  • Genetic d/o that bone marrow produces abnormal hemoglobin
  • microcytic, hypochromic, ferritin/iron normal or high, high RDW
  • Gold standard: Hgb electrophoresis: abnormal in beta
  • Blood smear: microcytosis, anisocytosis, poikilocytosis
  • Ferritin and iron normal
  • Provide genetic counseling
  • Minor trait does not require treatment

Acute Bronchitis with purulent discharge with fever tx? (NOT PCN!)
Primary Options:

azithromycin : 500 mg orally as a single dose on day one, followed by 250 mg once daily for 4 days, or 500 mg once daily for 3 days

Secondary Options

erythromycin base : 250-500 mg orally every 6 hours for 7-14 days

clarithromycin : 250 mg orally every 12 hours for 7-14 days

doxycycline : 100 mg orally every 12 hours on day one, followed by 100 mg once daily for 7-14 days

trimethoprim/sulfamethoxazole : 160/800 mg orally every 12 hours for 7-14 days

cephalexin : 250-500 mg orally every 6 hours for 7-14 days

NSAIDS
effect the renal prostaglandins and the effectiveness of the diuretic. NSAIDS inhibit the vasodilatory effect of PG, which predisposes the kidney to ischemia. Decrease blood flow to kidney.

Cataracts
opacity in lens, difficulty with glare, halos around light, blurred vision, gradual onset of decreased night vision, red reflex disappears (Red reflex is now opaque gray instead of orange red glow

A patient with a diagnosis of diverticulosis presents with localized left lower quadrant discomfort, a palpable mass, mild leukocytosis, and T = 100°F (37.8°C). The patient does not appear toxic and can tolerate fluids. An appropriate plan should include:
clear liquids and oral antibiotics.

A 70-year-old patient presents to the clinic complaining of dyspnea, palpitations, and fatigue. The patient reports a 2-week history of blackened stools, which the patient attributes to drinking berry juice. Assessment reveals vital signs of BP = 110/60, P = 100, R = 24; Hgb = 4.5 g/dL; Hct = 16%. What is the most appropriate immediate intervention?

1.
Order serum iron, total iron-binding capacity (TIBC), and ferritin.
2.
Refer to a gastroenterologist.
3.
Send to the emergency room.
4.
Order a complete blood count (CBC) with differential.
Send to emergency room

6 months hx of 39 year old pt who has daily swelling of wrists?
rheumatoid arthritis

treatment of iron deficiency anemia

  • 325 mg ferrous sulfate daily treat 3-6 months, take w Vit C for better absorption
  • Increase fiber and fluid to prevent constipation. May notice black-colored stools.
  • Red meat, beans, green leafy vegetables
  • Do not take w antacids, dairy, FQ, tetracyclines
  • recheck reticulocytes/CBC after 2 weeks after tx started to make sure the problem isn’t in the bone marrow (the reticulocytes should be higher).

The symptoms of benign prostatic hypertrophy are often overlooked because the older male:

1.
understands that surgery is the only cure for this condition.
2.
considers urinary problems to be a normal part of aging.
3.
is fearful of the side effects of additional medication.
4.
is concerned that he will no longer be able to obtain an erection.
considers urinary problems to be normal part of aging

The most common bacteria responsible for pneumonia in older adults residing in the community is:
Streptococcus pneumoniae.

Pancreatic Cancer Dx and Tx?
Dx: AST ALT bilirubin lipase amylase
Refer to GI surgeon for Whipple procedure

Preeclampsia recommendations
Bed rest with bathroon privi, weight and BP monitoring, follow urine protein and serum protein, Cr and plt counts.

Lichen sclerosis
Characterized by thinning of the epidermis and fibrosis (sclerosis) of the dermis; presents as a white patch (leukoplakia) with parchment-like vulvar skin. Small white spots that grow in patches. Itchy, discomfort, bleeding + blisters. In older women.

how to differential b/t B12 and folate deficiency – both are macrocytic
B12 comes with neuro symptoms tingling numbness of hands/feet, starts peripherally migrates centrally, difficulty walking, difficulty fine motor

Multiple myeloma- cancer of the bone marrow, bone pain, weakness… how to dx & tx?
Dx: CBC, FOBT,
Refer to hematologist affects African race

Which class of antihypertensive agents has been associated with acute renal failure and is contraindicated in patients with bilateral renal artery stenosis?
Angiotensin-converting enzyme (ACE) inhibitors

Upon admission to a nursing home, an 85-year-old patient receives a negative result on the purified protein derivative (PPD) test. To determine TB status in the frail elderly, a second PPD should be performed in:

1.
72 hours.
2.
1 week.
3.
3 months.
4.
6 months.
1 week

Rheumatoid arthritis dx?

Polymyalgia Rheumatica
inflammatory disorder causing muscle and joint pain and stiffness (shoulders/hips), aching stiffness about the upper arms, neck, lower back and thighs, symptoms develop quickly, and worse in the morning

An 18-month-old child presents with a bulging, immobile tympanic membrane; T = 103°F (39.4°C). Assessment also reveals a grade II/VI systolic murmur at the left sternal border. After initiation of treatment for otitis media, the most appropriate intervention is to:
reevaluate the patient in 10 days.

Which of the following is NOT used for therapy of an acute attack of gout?

1.
Colchicine (Colcrys)
2.
Indomethacin (Indocin)
3.
Methylprednisolone (Solumedrol)
4.
Allopurinol (Zyloprim)
allopurinol (zyloprim)

Tanner stage 1 girls
-Preadolescent, no pubic hair, other than fine hair similar to abdomen
-No breast dev.

Reiters syndrome?

pernicious anemia cause/tx
destruction of parietal cells in stomach leads to decreased intrinsic factor leads to B12 not being absorbed.

causes of b12 deficiency

  • Macrocytic Normochromic
    no intake of foods of animal origin, bariatric surgery, antacids, h2 blockers, proton pump inhibitors, metformin

Gouty arthritis?

Tanner stage 2 girls
-sparse growth of long slightly pigmented, downy, straight
-small area of glandular tissue w/ surrounding areola that begins to widen around age 10-11. Breast Bud

Polymyalgia Rheumatica dx & tx?
DX elevated sed rate, Creactive protein, (Low hemoglobin)
line TX oral prednisone

The management of choice for polymyalgia rheumatica includes:
prednisone.

A 26-year-old female presents with abdominal distention, bloating, and intermittent crampy abdominal pain relieved by defecation. She has four to six loose stools a day when stressed. Between bouts of diarrhea, she often has constipation. Physical examination results are normal. The CBC was normal. The most likely diagnosis is:

1.
diverticulitis.
2.
viral gastroenteritis.
3.
inflammatory bowel disease.
4.
irritable bowel syndrome
irritable bowel syndrome

how does fever affect seizure threshold
fever lowers seizure threshold

A mother has just noticed that her 15-month-old, 15-kg child has recently ingested an unknown quantity of iron tablets. The mother has ipecac syrup and activated charcoal on hand. The mother should be advised to:

1.
take the child to the primary care provider.
2.
give ipecac per recommendation on the bottle label.
3.
take the child to the emergency room.
4.
give the child 30 g of activated charcoal diluted with water
take child to emergency room

Club foot (Other name for?)
talipes equinovarus,

Which of the following pharmacotherapeutics would be most important to administer to a patient who has a corneal abrasion?
Gentamicin ophthalmic (Genoptic)

s/s of brain tumor:
gets worse when lying down and neurological focal deficits

Tanner stage 3 girls
Breast enlargement without separate nipple contour. One mound.
Pubic hair fills out but is straight, starts to curl

A 13-year-old is concerned because she has not yet begun to menstruate. Physical examination indicates that the patient is at Tanner stage IV and is of average height and weight. Which of the following would be the most appropriate response to this patient?

1.
“We’ll need to refer you to an endocrinologist for a complete workup.”
2.
“We’ll need to do some tests to find out why you are not having periods.”
3.
“I’ll give you some pills that will make your periods start.”
4.
“Your development is exactly as expected for your age; you’ll probably begin to have periods within a year.”
“Your development is exactly as expected for your age; you’ll probably begin to have periods within a year.”

ABSENCE SEIZURE other name, define
PETIT MAL SEIZURE: SUDDEN BRIEF LAPSES OF IN ATTENTION. SEEN ON

Migraine s/s? Tx? differ from tension, cluster, stroke?
?

Tanner stage 4 girls
Areola and breast projects as secondary mound
Pubic hair is more thicker, darker, coarse – starts to curl

Bouchards nodes can be found in?
RA & OA

A patient with type 1 diabetes mellitus who is on NPH and regular insulin split-dosing presents with complaints of early morning rise in fingerstick blood glucose. A review of an at-home glucose test reveals increased morning levels. After an increase in the evening insulin dose, the problem worsens. This is most likely an example of:
the Somogyi effect.

A 14-year-old male with bronchitis is being treated with fluids and expectorants. He returns to the clinic with a fever of 103°F (39.4°C), right pleuritic chest pain, and green sputum. Which of the following examination results would be expected?
Right lower lobe crackles

mini mental status exam MMSE – what is the cut off for dementia
MMSE- <24 dementia. The lower your score the worse off you are

Most common cause of death cancer Women? Men?
Lungs!

Tanner 1 Boys
Prepuberty/none

Heberden’s nodes can be found in?
OA

The most common bacterial cause of cellulitis in patients with diabetes mellitus is:

1.
Pseudomonas aeruginosa.
2.
group B beta-hemolytic Streptococcus.
3.
Staphylococcus aureus.
4.
Staphylococcus saprophyticus.
Staphylococcus aureus

subdural hematoma
skateboarding concussion. Diagnosed with CT.

List of cancers incidence?
skin: melatonin cancer most deadly?
Basal cell cancer: most incidence?

Tanner 2 Boys
testes enlarge/few straight fine hairs

Tubercle
small round projection on bone or surface

The follow-up examination of a 12-month-old treated 2 weeks ago for bilateral otitis media reveals a normal left tympanic membrane and a right tympanic membrane with visible serous fluid levels and decreased mobility. An appropriate plan should include a:
follow-up evaluation of the ears in 2 months.

A 46-year-old female found a lump in her breast this morning. History includes: no prior breast disease; G2P2 (first birth, age 22); maternal aunt diagnosed with breast cancer at age 72; last menstrual period was 2.5 weeks ago. On examination, a nurse practitioner palpates a 2-cm round, soft, mobile, tender lesion in the upper outer quadrant of the right breast. Mammogram is negative. Which of the following actions is most appropriate?

1.
Reassure the patient that no further action is necessary.
2.
Repeat unilateral right mammogram in 3 months.
3.
Schedule a right-breast ultrasound.
4.
Refer for genetic testing if the lump increases in size.
schedule a right breast ultrasound

fibromyalgia
11/18 points. Widespread pain for at least three months

Cancer death lists?

Tanner 3 (boys)
Penis lengthens, darker coarse hair starts to curl

Colles fx
fracture of distal radius of the forearm, DINNER FORK fracture, most common type wrist fracture

A 50-year-old male presents with a chief complaint of malaise. Further questioning reveals that his primary concern is delayed ejaculation. He is currently taking the following medications: atenolol (Tenormin), 50 mg daily; paroxetine (Paxil), 20 mg daily; loratadine (Claritin-D), 1 tablet daily; and hydrochlorothiazide (HCTZ), 25 mg daily. The most likely cause of the patient’s concern would be:
paroxetine (Paxil).

Risk factors for genital herpes include:

1.
multiple sexual partners and years of sexual activity.
2.
genetic transmission.
3.
infection with other sexually transmitted diseases before the age of 12.
4.
multiple sexual partners and frequent use of public spas.
multiple sexual partners and years of sexual activity

A 68-year-old female presents with a new onset of left-sided throbbing headache. She has noticed some spots in her visual fields that come and go. She is being treated with an NSAID for arthritis. Sedimentation rate is elevated, but all other laboratory values are within normal limits. The headache is most likely due to:
temporal arteritis.

On physically assessing a newborn’s head, you note an enlarged, fluid-filled area that is subperiosteal and does not cross suture lines. This finding indicates the presence of:

1.
macrocephaly.
2.
subgaleal hemorrhage.
3.
congenital hypothyroidism.
4.
cephalohematoma.
cephalohematoma

BPPV
Dix Hallpike maneuver-

Diverticulitis
small pouch like herniations, acute onset of fever, LLQ pain, anorexia, N/V,
Dx CBC, leukocytosis, shift bands to left,
TX fluoroquinolone, flagyl, , if severe refer to ER

Tanner 4 boys
penis widens, thicker, darker, coarse, curly hair

Sexual organs cancer deaths?
Ovaries in women
What about men? Prostate?

cluster headaches: tx, abortive and prophylaxis
tx High dose O2 via Mask 12L 100%, imitrex (abort) and a CCB for prophylaxis. VERAPAMIL

s/s of skin cancer:

tanner 5 boys/girls
Adult pattern

Pancreatitis symptom
midepigastric pain radiates to midback

Which of the following is the most serious outcome of Barrett’s esophagus?
Esophageal adenocarcinoma

A 65-year-old patient being treated with digoxin (Lanoxin) for chronic heart failure (HF) presents with complaints of palpitations. Three days ago, the patient was placed on clarithromycin (Biaxin) by another provider for community-acquired pneumonia. The appropriate course of action at this point would be to:

1.
increase the digoxin dosage by 25%.
2.
obtain a digoxin level and change the antibiotic to azithromycin (Zithromax).
3.
obtain a chest X-ray.
4.
obtain a serum potassium level and perform a 12-lead EKG.
obtain a digoxin level and change the antibiotic to azithromycin (zithromax)

A 39-year-old patient was diagnosed with acute bronchitis in the emergency department and treated with acetaminophen, dextromethorphan, and metaproterenol (Alupent). The patient’s history reveals a smoking habit of 1 pack per day. The patient now presents to a nurse practitioner’s office with a fever of 101.2°F (39.4°C) and a cough productive of thick, yellow-green, foul-smelling sputum. The nurse practitioner should encourage smoking cessation and prescribe:
a macrolide antibiotic.

A 54-year-old male who is 30 pounds overweight states that he awakens at night with heartburn and the taste of hot acid in his mouth. Stress makes his condition worse, yet baking soda seems to provide some relief. Findings on examination are normal and the stool for occult blood is negative. The preliminary diagnosis(es) should be:

1.
esophagitis.
2.
esophageal spasm.
3.
helicobacter pylori.
4.
gastroesophageal reflux disease.
GERD

migraine tx: abortive and prophylaxis
Triptains for abortive.
Prophylaxis:

  • (propranolol)
    -TCA (Amitriptyline)
  • Anticonvulsants (gabapentin, topiramate).

Lichen Sclerosis
whitish patches on genitals, lesions chronic affect vulva/anal in elderly patients, risk for cancer

eGFR
test used to monitor kidney function and evaluate chronic kidney disease

Anemia: pernicious anemia:
neurological deficits

Bells Palsy, which nerve
CN VII- Facial BELLS

Other anemia: s/s, tx, differentiation

Signs of depression
anhedonia, weight loss or gain, fatigue, change in appetite, insomnia or hypersomnia, feeling of guilt or suicide.

Thrichomonas
yellow-greenish discharge with dyspareunia, pelvic exam reveals strawberry patches on cervix

A 15-year-old patient returns for contraceptive services 2 weeks after a diagnosis of trichomonas vaginitis and treatment with 2 g of metronidazole (Flagyl). She reports that discharge and itching are gone, but she is urinating frequently, accompanied with a lot of burning. The patient has not resumed sexual activity and has menstruated since her last visit. Examination reveals mild suprapubic tenderness, no leukorrhea, and a normal wet mount. Gonococcal culture and chlamydia tests are negative. Which diagnostic test should be performed immediately?
Microscopic examination of urine

A late adolescent patient presents with complaints of acute-onset unilateral right eye pain, visual changes, seeing halos around lights, abdominal pain, and intermittent nausea. Physical findings include an erythematous right eye without discharge, a cloudy cornea, and a moderately dilated right pupil that is nonreactive to light. The nurse practitioner should immediately:

1.
apply steroid drops to the right eye.
2.
apply mydriatic drops to the eye.
3.
refer to the emergency department.
4.
make an appointment for him to see an ophthalmologist.
refer to emergency department

A nurse practitioner orders pulmonary rehabilitation for a 75-year-old with COPD. Expected outcomes of this program include all of the following EXCEPT:
increased lung capacity.

In a 6-year-old child, which of the following is the preferred drug for stage 1 Lyme disease?

1.
Trimethoprim-sulfamethoxazole (Bactrim)
2.
Azithromycin (Zithromax)
3.
Doxycycline (Doryx)
4.
Cefuroxime (Ceftin)
cefuroxime (ceftin)

Anemia of chronic disease

Trichimonas
dx is wet mount/prep, flagella, partner needs treated too TX flagyl PO for 7 days

ectopic pregnancy
Hx of amenorrhea and new onset of bloody vaginal spotting, left adnexal tenderness and cervical motion tenderness

Which CN control eye MOVEMENT
EOM- CN III, IV, VI. (LR6SO4) 3-pupil/accommodation `

A 4-year-old girl with a history of recurrent urinary tract infections returns for follow-up after diagnosis of the most recent infection. During the physical examination, the 4-year-old tells the nurse practitioner that her brother (age 9) lies on top of her and rubs his penis near her pubic area. After completing the physical examination, the nurse practitioner must:
report suspicions to the appropriate child protective services agency.

An 86-year-old patient is experiencing radiculopathy-associated lower back pain that has not improved over the past 4 weeks. The medical history includes a lumbar discectomy. The best type of imaging study for evaluating this patient would be a/an:

1.
MRI.
2.
spinal X-ray.
3.
myelography.
4.
nuclear bone scan.
MRI

Fitz-Hugh-Curtis syndrome
Perihepatic infection which results in liver capsule inflammation from pelvic infections such as gonorrhea & chlamydia. PID infections. Sharp pain on the right upper quadrant, with PID symptoms.

PID
DX NAAT TX ceftriaxone, doxy, flagyl

Folic deficiency anemia

UTI urine culture positive when….
KNOW THAT UTI IS 100,000 (10 x 5) CFU’S TO BE DIAGNOSED in non preg patient and 10 x 3 in preg patient

3+ protein in urine, how do you follow up ?
do 24 hour urine for protein and creat clear

Beta anemia

Chlamydia Symptoms, Dx, & Tx
Fitz-Hugh syndrome, asymptomatic, mucopurulent, vaginal discharge, bleeding
Dx: gold standard, nucleic acid amplification test NAAT, gen probe, antigen detection (swab), urinalysis, wet prep >20 WBC
Azithro 1 dose or doxycycline for 7 days annual screenings

A 2-year-old child is diagnosed with radial head subluxation (“nursemaid’s elbow”). After closed manipulation, the best indication of successful treatment is that:
the child quickly begins to use the affected arm.

A 30-month-old with a history of frequent ear infections was seen in September after a symptom-free summer. Examination revealed bilateral bulging eardrums. After two separate courses of antibiotics, fluid is noted in both ears. The parent reports speech difficulties. The most likely diagnosis is:

1.
otitis media with effusion.
2.
myringitis.
3.
recurrent acute otitis.
4.
Esutachian tube dysfunction.
otiti media with effusion

12 weeks
uterine fundus first rises above the symphysis pubis.

Any baby 2-24 months with UTI and fever. how do you follow up?

  • do renal and bladder ultrasound for first febrile UTI

cycle cell anemia

Wernicke’s aphasia
difficulty with comprehension but none with speech

A 67-year-old patient with type 2 diabetes mellitus, congestive heart failure (CHF), and mild coronary artery disease is currently taking digoxin, 0.25 mg daily; hydrochlorothiazide, 25 mg daily; metformin, 500 mg daily; glipizide, 10 mg daily; and atorvastatin (Lipitor), 20 mg h.s. Which of the following is an accurate statement regarding this regimen?
The hydrochlorothiazide will predispose the patient to digoxin toxicity.

A 19-year-old patient who has used oral contraceptives for 3 years plans to discontinue the pill at the end of her current cycle to become pregnant. Which of the following daily supplements would receive the highest priority?

1.
Calcium carbonate
2.
Folic acid
3.
Ferrous sulfate
4.
Multivitamin
folic acid

16 weeks
UF is between symphysis pubis and the umbilicus

RBC casts in urine, what is possible diagosis
glomerulonephritis

iron deficient anemia

20 weeks
UF at level of umbilicus

Broca’s aphasia
comprehends speech but unable to verbalize speech

During development of a treatment plan for an 84-year-old patient with hypothyroidism, the nurse practitioner must keep in mind the possibility of the worsening of:
osteoporosis.

A frail elderly widow presents to the senior clinic with a 3- to 4-day history of confusion that became worse the evening before but currently seems a little better, although her speech is rambling and difficult to follow. During the examination, the patient is anxious and agitated. She is on cardiac medications, an antidepressant, and insulin. The most likely diagnosis is:

1.
mild stroke.
2.
polypharmacy.
3.
electrolyte imbalance.
4.
agitated depression.
polypharmacy

First choice Atb for UTI
Nitrofurantoin (Macrobid)

Hpylori negative ulcer , tx
s: h2 first bedtime, may combine them with PPI. Do 6-8 weeks. PUNT after.

pregnancy anemia

20 to 35 weeks
Measure the distance between upper edge of pubic symphysis and the top of the uterine fundus using a tape measure. + or – 2 cm. 32 30-34 weeks

Initial therapy for a 3-week-old infant with uncomplicated symptoms of gastroesophageal reflux disease includes which of the following positions and change in diet?
Smaller, more frequent feedings while holding infant

A 46-year-old female presents with a complaint of amenorrhea for 3 months. The first-line diagnostic workup is:

1.
urine pregnancy test.
2.
progesterone challenge.
3.
serum prolactin level.
4.
thyroid-stimulating hormone (TSH).
urine pregnancy test

HBsAg means what
patient current HAS HbSAg (the test has the word “HAS” in it)

Define fremitis:
When is it increased?

  1. Fremitus is enhanced by consolidation.
  2. Fremitus is decreased by pleural fluid.
  3. Fremitus is decreased by pneumothorax.
  4. TRUE. Fremitus is enhanced by consolidation.
  5. TRUE. Fremitus is markedly decreased by pleural fluid.
  6. TRUE. Fremitus is decreased by pneumothorax.

CHF
S1, S2 and S3

Temporal arteritis
The cause of the blood vessel inflammation is unknown. In some cases, the swelling affects just part of an artery with sections of normal vessel in between. Symptoms include acute onset of headaches located on one temple, skin over artery is indurated, jaw pain, vision loss, fever, and weight loss, anorexia, fatigue.

Anti-HBs positive
immunity either from previous disease or vaccine

Bronchitis:
s/s, Tx,

RF for suicide
elderly white man after death of spouse, past Hx of suicide, family Hx of suicide, gender males have a higher success rate, hx of depression or bipolar

Temporal arteritis dx & tx
Dx usually requires biopsy of the temporal artery. Elevated sed rate and c reactive protein, high risk with polymyalgia rheumatica, the condition needs Prompt treatment with steroid medications to prevent permanent vision loss

Zollinger Ellison syndrome
tumors causes stomach to produce too much acid resulting in PUD

HBeAg positive
E- EEEEEEEnfectious! It is an indicator of active viral replication; this means the person infected with Hepatitis B can likely transmit the virus on to another person

Alpha fetoprotein
produced by fetal liver, high for twins, neural tube defect. Low AFP RF down syndrome. For either order the triple screen

Barrets esophagus:
Increases risk of cancer of esophagus

Somogyi
too much insulin in the blood at night causes a rebound increasing blood sugar in the AM

Anti-HBc
c is for caught it. either has or HAD hep B. The presence of
anti-HBc indicates previous
or ongoing infection with
hepatitis B virus in an
undefi ned time frame.

HIV
Varicella contrainindicated

Sedimentation rate is elevated and patient has visual spots and left sided-headache:
temporal arteritis

Anti-HCV positive. What is next step?
order HCV RNA. Biopsy of liver to check stage.

Psoriasis diagnosis s/s and tx:
sharply defined plaques with silvery scales: use auspitz sign when scales are pealed away and capillary bleeding. Sun helps pt. OTHER INFO?
Chronic inflammatory skin disease characterized by extensive erythematous, circumscribed scaly papules, and plaques.

Runs a variable course and seldom completely subsides. Severity is aggravated by genetic, infectious, emotional, and environmental factors.

Lesions are red, inflamed, silvery-white scaly, and circumscribed papules and plaques on elbows, knees, extensor limbs, and scalp. Psoriatic nails have a pitted surface and/or hypertrophic (subungual) changes.

Diagnosis is usually clinical.

Mild or limited psoriasis is treated with topical corticosteroids and/or vitamin D analogs.

Moderate to severe and/or extensive psoriasis may require phototherapy, oral retinoids, methotrexate, biological agents, or cyclosporine.

Inevitable abortion
vaginal bleeding with pain and cervical dilation and/or effacement.

Dawn Phenomenon
early morning increase in blood sugar between 2-8am

threatened abortion
vaginal bleeding without dilation of the cervix or passage of tissue. minimal pain

Pulses paradoxes
is an abnormally large decrease in stroke volume, systolic blood pressure and pulse wave amplitude during inspiration

s/s of preeclapsia:
headache, visual disturbances, edema.
Tests:
Treatments: metoprolol

Pencil like stool
think colon cancer: Tenesmus or the feeling of having to defecate without having stools, pain upon defecation, or sciatica can be symptoms of rectal cancer

S/s Intussusception and age it ususally occurs
Intussecption- sausage shaped mass in upper right quad. Currant jelly stool. Ur bowels prolapse into another part of ur intestine. Barium or air enema can help to reduce this. Previously healthy then they get sick. Usually before 2 years

incomplete abortion
moderate to diffuse bleeding with the passage of tissue and painful uterine cramping

Contraindications for Thiazides
Serious sulfa allergies, hypotension, gout, renal failure due to loss of sodium, hypokalemia, may worsen DM, lead to metabolic alkalosis, by action of the renin angiotensin system, decreased volume and arterial pressure. May cause hyperglycemia due to insulin resistance and decreased insulin release

How to palpate for rotator cuff injury:
??

Age of pyloric stenosis
Pyloric stenosis- 3-12 weeks

How to assess for knee injury:
Lachman
Drawer test:
Miniscus versus cruciable ligament tear

Presumptive signs of pregnancy
signs felt by women, amenorrhea, nausea, breast tenderness, N/V, urinary frequency,

Thiazides are beneficial for:
osteoporosis, reduces calcium excretion by kidneys, and stimulates osteoblasts which help with bone growth

Glimeperide
Don’t use this DM med with insulin

medications that can increase GERD
BB, CCB, HTN meds increase GERD.

when to refer ankle injury?
Grade of strain?

Probable signs of pregnancy
examiner suspects a women is pregnant. such as enlarged uterus, hegar’s sign(softening and compressibility of the lower uterus), chadwick’s sign (deepened violet-bluish color of the cervix and vaginal mucosa), goodell’s sign(softening of the cervical tip), ballottement, braxton hicks, positive pregnancy test, fetal outline felt by examiner.

what time of day should H2 blockers be taken
before bed

primary amenorrhea:
Primary amenorrhea: lack of menses by age 15 years in a patient with appropriate development of secondary sexual characteristics, or absent menses by age 13 years and no other pubertal maturation.

Secondary amenorrhea: lack of menses in a non-pregnant female for at least 3 cycles of her previous interval, or lack of menses for 6 months in a patient who was previously menstruating.

A1C result for Dx of DM
6.5 or greater and confirmed on a different day

Positive signs of pregnancy
audible fetal heartbeat, fetal movement felt by examiner, ultrasound visualization of fetus, cardiac activity on U/S.

intermittent claudication
Initial evaulation check the ankle and brachial blood pressure before and after exercise.

A1C results for Dx of Pre-DM
5.7-6.4

radial head fracture:
caused by fall onto wrist: x rays may show positive flat pad sign also known as sail sign.

epigastric pain that radiates to the back
pancreatitis

Random glucose that confirms DM
200 or higher

different between amylase and lipase
lipase more specific to alcoholic pancreatitis, starts to rise 4-8 hours after symptoms begin

amylase starts to rise 2-12 hours after onset of symptoms

Theophylline
contraindicated drugs are erythromycin, phenytoin and cimetidine.

Nursemaid elbow:
after reduction the child quickly resumes activity

treatment for high triglycerides.
High Triglycerides- causes pancreatitis >500.
If >500 treat with Niacin or Fibrate or Niaspan.
If your patient is already on NIACIN you can add a fibrate like (LOPID/TRICOR).
Apparently an insulin infusion works also

Webber test:
used to test for both conductive and neural hearing loss.

Eye tests
The cover/uncover test screens for strabismus. Visual acuity is the snellen. Cataracts screened by using the red reflex. (positive by white relaxation.

Which medications increase the risk of developing Type 2 DM?
Glucocorticosteroids
HCTZ
Atypical antipsychotics
HMG Co-A reductase inhibitors

Mitral Regurge murmur:
How to diagnose: Mitral regurgitation (MR) may present with dyspnea, usually on exertion, palpitations, and/or decreased exercise tolerance.

Typically, presents as a holosystolic blowing murmur at the apex, radiating to axilla.

1st Tests To Order

transthoracic echo
ECG

Koilonychia
spoon shaped nails associated with IDA.

CCBs (Verapamil)
Med that can make GERD worse?

Fructosamine test
checks sugar for past 2-4 weeks, high results = high blood sugar

hyperparathyroidism – labs and causes
Elevated PTH with primary- hypercalcemia or secondary-hypocalcemia
Primary hyperparathyroidism- benign adenoma (most common) or familial;
Secondary- is a response to hypocalcemia (renal failure, drugs)

s/s of hemorrhoids and tx:
Bulk forming agents with hydrocortisone tx

s3
opening snap, CHF, normal variant in healthy young adults and athelets

Negligence
When a practitioner fails to exercise the care that a reasonable person would exercise (Injury does not have to happen)

Duodenal ulcer
Type of ulcer where symptoms occur 2-5 hours after eating. (Symptoms can be relieved by antacids).

Herberden’s nodes
Hard nodules or enlargements
of the distal interphalangeal
joints of the fingers. Found in degenerative joint disease

s/s hyperparathyroidism
Fragile bones that easily fracture (osteoporosis)
Kidney stones
Excessive urination
Abdominal pain
Tiring easily or weakness
Depression or forgetfulness
Bone and joint pain

s/s of colon cancer:

herbal remedies commonly used:
Fever few: migraines, migraines during menstration.

DTAP, Tdap, Td
Dtap is the pediatric version you receive from 2 mo to 6 yrs. Adults begin with Tdap. Td as booster every 10 yes.

Gastric ulcer
Type of ulcer where symptoms occur within minutes after eating (less relief from antacids)

treatment for hyperparathyroidism
Primary: surgical parathyroidectomy if symptomatic or biphosphonates and monitoring (BP, renal, bones) if asymptomatic
Secondary: Vitamin D and calcium & treat cause

Addison’s :electrolytes, symptoms, treatment
Addison’s- adrenal deficiency – usually autoimmune cause. Deficient in cortisol (low blood sugar cuz cortisol stimulates gluconeogenesis in the liver so low cortisol = hypoglycemia in times of stress)… hyperpigmentation (Kennedy’s perpetual tan) and low aldosterone (leads to low Na in blood — SALT CRAVINGS! — high K) , You must give cortisol. (Diagnosis Plasma Cortisol <5 mcg/dl @ 0800.) *You have ADD cortisol because cortisol is low in ADDisons

Mini-mental status exam:
Also called folstein mini-mental status exam- used for memory or- attention and calculation, recall, language, ability to follow simple commands and orientation.

Menses
The onset of menses occurs between Tanner stages III and IV.

Hypertrophied left ventricle
A common early finding in pt with chronic aortic regurgitation.

Tzank, PCR, or DFA
Tests that can be used to dx shingles

Delerium
reversible acute confusion, hours to days, short attention span, memory loss and disorientated.

Causes:prescription meds, substance abuse, dru-drug interactions, withdrawl, infections, sepsis, electrolyte inbalances

Alzheihmer does not cause it

Cushing syndrome
Central obesity, moon face, purple striae, hairy, hypertension, elevated plasma CORTISOL in AM. “INC BS, SODIUM” Dec K. You must draw cortisol levels in the morning.

Fever:
decreases the threshold of seizures.

when to start basal insulin
If you are already on TWO oral drugs for diabetes and A1c is 9 or higher, start BASAL insulin. If you cannot tolerate metformin and your A1c is 9 or higher start BASAL insulin.

Beta thal anemia:
electrophoresis

Implanon
progestin-only method and provides three yrs of pregnancy protection.

Diarrhea, abdominal pain, N/V
Classic symptoms of Cdiff

parathyroid gland and PTH – role in the body

  • PTH is responsible for calcium loss or gain from bones, kidneys, and GI tract.

Neutral Protamine Hagedorn (NPH)
intermediate acting insulin

E Coli
The Most common pathogen with Pyleonephritis

HIV monitor progress of disease?
CD4 and viral load tests

3-4 weeks ago
A patient with a primary case of scabies was probably infected how long ago?

Legg-calve-perthens
Referral for aggressive treatment if they are 7 yrs of age

best test to differentiate iron deficiency from other anemias?
ferritin

Hypothyroidism- lab findings, diseases associated with, tx
High TSH LOW Free T4/T3, However, Free T4 is much more specific to this disease. Hashimotos (autoimmune) think of everything in ur body is slowing down. Synthroid.

Hypokalemia
A pt taking HCTZ and complaining of muscle cramps probably has?

Primary amenorrhea
-no menarche by age 16
-usually due to anatomic or genetic causes

trigeminal neuralgia:
Facial pain syndrome in the distribution of ≥1 divisions of the trigeminal nerve.

Diagnosis is clinical, with a history of paroxysms of sharp, stabbing, intense pain lasting up to 2 minutes.

First-line therapy is medical, to which the majority of patients are partially responsive.

Primary Options

carbamazepine : 200 mg/day orally initially given in 1-2 divided doses, usual maintenance dose is 400-1200 mg/day given in 2 divided doses

oxcarbazepine : 300 mg/day orally initially, usual maintenance dose is 600-1200 mg/day, maximum 1200 mg/day given in 2 divided doses

Which tx for hyperthyroidism is preferred in pregnancy
PTU PREFERED IN PREGNANCY

HTN
AV Nicking is seen in what disease?

Dubowitz and Ballard method
tests the elasticity of cartilage in the ears only.

Acute bacterial conjunctivitis in infants:
Primary Options

azithromycin ophthalmic : (1%) 1 drop into the affected eye(s) twice for one day, then once daily for 4 days

erythromycin ophthalmic : (0.5%) apply to the affected eye(s) four times daily

polymyxin B/trimethoprim ophthalmic : (10,000 units/mg) 1 drop into the affected eye(s) four times daily

hyperthyroidism
Graves disease-autoimmune. Lid lag, exophthalmos, everything is hyper (body wise). Treatment: PTU/Tapazole.

chorionic villus sampling (CVS)
CVS 10- 12 weeks
Amniocentesis 15-18 weeks

Complications of Cellulitis with a diabetic patient
OSTEOMYELITIS

Beta Blockers, CCB, & TCAs
Medications used for migraine prophylaxis

HCTZ interacts with digoxin:
Digoxin toxicity

Hashimotos, TSH and Free T3/t4 , plus test to connfirm
first order TSH and get high TSH. Then order thyroid panel and get low t3. The diagnosis is confirmed by the presence of thyroid autoantibodies in the blood, usually at high levels. Thyroid peroxidase antibodies (TPO) are more common that thyroglobulin antibodies

most common pathogens for community acquired pneumonia in older adults:
H.Influenza and S. Pneumoniae
Pneumocystis jiroveci can cause pneumonia in people whose immune system is not working well.
Streptococcus pneumoniae, or Pneumococcus, continues to be responsible for 20 to 50 percent of CAP, and is the single most common pathogen causing CAP.

Mycoplasma, Chlamydophila, and Legionella, collectively known as the “atypical” CAP pathogens, cause 20 – 30% of CAP. Legionella is commonly implicated in severe CAP, while Mycoplasma and Chlamydophila more often result in mild clinical disease.

Influenza is the most common respiratory virus resulting in CAP. Influenza infection also predisposes to subsequent development of bacterial CAP, referred to as “post-influenza pneumonia”.

Hemophilus and Moraxella are common pathogens, particularly in patients with COPD or intrinsic lung disease.

Mammograms
UPSTK end at 75
ACOG up to 74

Hyyphae
A skin lesion fluoresces under a Wood’s lamp. What microscopic finding is consistent with this?

Latent TB treated with
Latent TB usually treated with INH

Most common cause of acute bacterial sinusitis:
H.Influenza and S. Pneumoniae

Elderly PPD
Neg PPD, a second test PPD should be performed in 1 week.

Calcium
A supplement known to cause constipation?

s/s and testing for PCOS:
Key historic findings
female of reproductive age
irregular menstruation
infertility
hirsutism

Other Factors

scalp hair loss
oily skin or excessive sweating
acne
overweight or obesity
hypertension
acanthosis nigricans

1st Tests To Order

serum total and free testosterone
serum dehydroepiandrosterone sulfate (DHEAS)
serum 17-hydroxyprogesterone
serum prolactin
serum TSH
oral glucose tolerance test
fasting lipid panel

Other Tests to Consider

serum androstenedione
pelvic ultrasound
basal body temperature monitoring
luteal phase progesterone measurement
serum LH and FSH
Tx:
Acute
with infertility and desiring fertility

weight loss
metformin
clomiphene
metformin
dexamethasone
gonadotropins
metformin
in vitro fertilization
metformin
laparoscopic ovarian drilling

Ongoing
not desiring current fertility

with hyperandrogenic features alone
oral contraceptive pill
metformin
mechanical hair removal or topical therapy
antiandrogen
antiandrogen plus oral contraceptive pill
long-acting GnRH analog plus oral contraceptive pill
with oligoamenorrhea alone
weight loss
oral contraceptive pill
metformin
cyclic progestin
with hyperandrogenic features plus oligoamenorrhea
weight loss
oral contraceptive pill
metformin
mechanical hair removal or topical therapy
antiandrogen plus oral contraceptive pill
long-acting GnRH analog plus oral contraceptive pill

Diagnosing TB, which is gold standard for dx of TB
ACTIVE TB order, NAAT, C&S, AFB. The AFB is not diagnostic. SPUTUM FOR C & S is gold standard.

Ribavirin se
hemolytic amemia

Improve Urine flow
An elderly patient will take Saw Palmetto because he thinks it will help with what?

symptomatolgy
management of COPD for the elderly

60 yo
CDC recommends Zoster vaccine in what age of immunocompetent pt?

Mantoux test results, what size induration is considered positive.

5mm-think immunocompromised or person in close contacts.
10 think Immigrants, working status (nurses), drug users, home life.
15 Think no risks

28 week old pregnant patient has single episode of vaginal bleeding without other s/s: What to do?
Ultrasound

What will you hear with percussion of emphysema lungs, what will you see on CXR, what will you notice about their breathing
Emphysema Lungs- Percussion-HYPERENNOSANCE tactile frem + egophony- dec. CXR- flattened diaphragms with hyperinflation. Inc. AP diameter, accessory muscles, pursed-lip breathing, weight loss.

TIA V. Stroke:
Tia has absence of residual s/s.

Biaxin and digoxin
Biaxin raises digoxin level

Alcohol
If a pt is taking metronidazole, they should avoid what?

Terbinafine, atorvastatin, simvastatin
Which 3 medications should have liver function test prior to starting?

McBurneys point: rebound tenderness-
sign for acute appendicitis

MANTRELS score [41]

Score is based on clinical characteristics of the patients. The higher the score out of a possible total of 10, the greater the chance of having acute appendicitis.

M: Migration of pain to right lower quadrant = 1 point

A: Anorexia = 1 point

N: Nausea and vomiting = 1 point

T: Tenderness in right lower quadrant = 2 points

R: Rebound tenderness = 1 point

E: Elevated temperature = 1 point

L: Leukocytosis = 2 points

S: Shift of WBC count to left = 1 point

Or use apache 2 calculator.

Omezaprole
se headache, abdominal pain and gas

asthma patient on low-dose ICS still has symptoms, what is next step
IF LOW DOSE ICS, NEXT IS MEDIUM DOSE

intermittent asthma, day time symptom frequency, night time awakenings, tx
<2 / week day, <2 month – night, SABA

Polymyalgia rheumatica s/s, tests and tx:
Basics:
Typically a history of neck, shoulder girdle, and/or hip girdle stiffness and pain, occurring in patients age 50 years or older.

Patients complain of difficulty rising from seated or prone positions, varying degrees of muscle tenderness, shoulder/hip bursitis, and/or oligoarthritis.

More common in women.

About 15% to 20% of patients with polymyalgia rheumatica (PMR) have giant cell arteritis (GCA); 40% to 60% of GCA patients have PMR.

Diagnosis is made via history and with supportive laboratory tests indicating an elevated ESR or CRP.

Rapid improvement often occurs within 24 to 72 hours with low-dose prednisone.

s/s:
Key Factors

shoulder/hip girdle stiffness
shoulder/hip girdle pain
rapid response to corticosteroids

Other Factors

acute onset
low-grade fever
anorexia
weight loss
malaise
depression
asthenia
oligoarticular arthritis

dx:

Criteria: must have any 3 factors, or just 1 and a temporal artery biopsy positive for giant cell arteritis

Age over 65 years

Bilateral shoulder girdle pain

More than 1 hour morning stiffness

Symptom onset <2 weeks

ESR >40 mm/hour

Depression/weight loss

Upper arm tenderness, bilateral.

Tests: 1st Tests To Order

ESR
C-reactive protein (CRP)
CBC
ultrasound

Other Tests to Consider

TSH
MRI
serum protein electrophoresis
serum creatine phosphokinase
TX:
Acute
initial presentation

corticosteroid: prednisone : 10-20 mg orally once or twice daily
calcium + vitamin D + bisphosphonate
nonsteroidal anti-inflammatory drug (NSAID)
methotrexate
folic acid
tocilizumab

Ongoing
treatment-resistant or relapse or disease exacerbation

corticosteroid (increased dose)
calcium + vitamin D + bisphosphonate
methotrexate plus folic acid
tocilizumab or leflunomide

Nummular Eczema
pruritic inflammatory dz – young, old
fall/winter, coin-shaped plaques – common on lower leg extremities

tx: moisturizers or topical steroids

Mitral Valve porlapse
Which mitral disorder results fro redundancy of the mitral valve’s leaflets?

Mild persistent, daytime symptoms, night awakenings, tx

2 but not daily symptoms, >2 per month, but not weekly, SABA + low dose ICS

interductal papilloma
is a small, benign tumor that forms in a milk duct in the breast. These tumors are made of gland and fibrous tissue as well as blood vessels. They most commonly occur in women between ages 35 and 55. Unilateral spontaneous serous or serosanguineous discharge

s/s of polymyalgia rheumatica:
pelvic girdle pain for 6 months, unintentional weight loss, pain with ROM, unintentional weight loss, low hemoglobin and elevated ESR

LDL
The lipid particle with the greatest atherogenic effect is

Mod Persistent, daytime symptom frequency, night awakenings, tx
daily daytime symptoms, night awakenings >1 per week, SABA+ICS+LABA or SABA+medium dose ICS

Fasting blood glucose of 126:
Diagnose Diabetes

Alzheimer’s disease
starts with short term memory loss

Check TSH
If a pt’s lipids are elevated, what would the NP do next?

s/s of hypoglycemia if patient is on a betablocker and insulin:
sweating. Not tachycardia

ACE inhibitors
Which class of medication is frequently used to improve long-term outcomes in pts with systolic dysfunction?

Cellulitis
Staph aureus most common bacteria with people DM

Severe persistent, daytime, night awakenings, tx
symptoms throughout the day, nightly asthma symptoms- SABA, Med ICS plus LABA.

pulsus paradoxus
Pulsus paradox Apical pulse can still be heard even though the radial pulse is no longer palpable. Certain issues cause impairment with diastolic filling, 10 or greater drop in the SYSTOLIC pressure. I think her patient had asthma and their pressure dropped by 10 etc.

Tx for trichomonas vaginitis:
metronidazole 2 grams (flagyl)
s/s: Key Factors

vaginal discharge
dysuria
discharge adherent to vaginal mucosa

Other Factors

prior episodes
fever
pruritus
vulvodynia
vaginal dryness
vaginal bleeding
abdominal pain
dyspareunia
erythema
pale epithelium
shiny epithelium
decreased elasticity
friable epithelium
strawberry cervix
testing:
1st Tests To Order

vaginal pH
amine “whiff” test of vaginal secretions
wet mount microscopy of vaginal secretions
Gram stain of vaginal secretions
HIV test
nucleic acid amplification test (NAAT)
VDRL
serum rapid plasma regain (RPR) test
Criteria:

Bacterial vaginosis:

Vaginal pH >4.5

Whiff test

Clue cells (vaginal epithelial cells with distinctive stippled appearance in saline wet mount by being covered with bacteria)

Adherent white vaginal discharge.

cepalohematoma
enlarged fluid filled area that is subperiosteal and does not cross the suture line

Iron Def. Anemia
Which hypochromic, microcytic anemia has elevated RDW?

What to check before starting statins
Must check LFT before starting Statin

normal changes with aging?
less gastric acid production.

Forced Vital Capacity (FVC)
The volume of air expired during a forced maximal expiration after a forced maximal inspiration.

Clinical Inertia
Providers who do not increase therapy even if test results indicate the need.

T wave inversion suggests:
acute coronary ischemia

Potassium supplements
A patient taking an ARB should avoid?

Gastic ulcer v peptic ulcer disease:
Peptic ulcer:
Usually presents as chronic, upper abdominal pain related to eating a meal (dyspepsia).

Use of nonsteroidal anti-inflammatory drugs (NSAIDs) and Helicobacter pylori infection are the most common causes.

There may be some epigastric tenderness, but often there are no other signs on physical examination.

Endoscopy is diagnostic and may show an ulcer in the stomach or proximal duodenum. H pylori infection should be sought.

In the absence of “alarm” (red flag) symptoms or signs, testing for and treating H pylori and/or empiric acid inhibition therapy is appropriate.

Most common complications are gastroduodenal bleeding and perforation, either of which may be the presenting symptom, particularly in patients taking NSAIDs.

Gastritis:
A histologic term for inflammation of the gastric mucosa.

Helicobacter pylori infection and use of nonsteroidal anti-inflammatory drugs (NSAIDs) or alcohol are the most common causes. Other causes include stress (secondary to mucosal ischemia) and autoimmune gastritis. Rare forms include phlegmonous gastritis (a rare bacterial infection).

Diagnosis is based on clinical history and characteristic histological findings. A variety of methods may be used to diagnose H pylori infection.

Presence of suspicious features suggestive of upper GI malignancy requires urgent endoscopy. These include GI bleeding, anemia, early satiety, unexplained weight loss (>10% body weight), progressive dysphagia, odynophagia, or persistent vomiting.

Treatment depends on the etiology. Options include H pylori-eradication therapy, reduction of NSAIDs or alcohol exposure, and symptomatic therapy with histamine-2 antagonists and/or proton-pump inhibitors.

If untreated, progression to peptic ulcer disease may occur. Other complications include gastric carcinoma and gastric lymphoma.

Who gets statins

  1. Patients with any form of clinical ASCVD
  2. Patients with primary LDL-C levels of 190 mg per dL or greater
  3. Patients with diabetes mellitus, 40 to 75 years of age, with LDL-C levels of 70 to 189 mg per dL
  4. Patients without diabetes, 40 to 75 years of age, with an estimated 10-year ASCVD risk ≥ 7.5%

Age
The primary risk factor for development of breast cancer in women of average risk is?

Who gets moderate intensity statin

  1. Age > 75 years + ASCVD
  2. DM + 70-189 LDL but low <7.5 ASCVD risk
  3. healthy, 70-189 LDL, very low 5-<7.5 ASCVD risk (4 healthy, 70-189 LDL, low >7.5 ASCVD risk, may choose moderate or high intensity)

To assess abstract thought:
ask meaning of common proverb.

Lyme disease
child 6 yrs old amoxicillin or cefuroxime

14 days or longer
How long should a pt be treated with atb if he has prostatits secondary to an STD?

Latent TB
INH for 9 months isoniazid

Thiazide diuretic c/i and side effects
no sulfa allergies,
MoNKey low: hypo Mg, Na, K
GLUT – hyperGlycermia, hyperLipidemia, hyperUricemia, hyperTriglyseridemia,

No satelite lesions with diaper rash:
contact dermatitis
Satelite lesions- candida infection

Chronic Venous Insufficiency
Impaired venous return. Achy legs relieved by elevation, edema after prolonged standing, night cramps, brownish discoloration, cold, ulcers. Etc. do support stockings.

pulmonary rehab goals include:
increase exercise capacity, decreased hospitalization, enhanced quality of life.

NOT increased lung capacity.

H and H testing infant
6, 9 and 12

Cetirizine
Which long-acting antihistamine is sedating?

peripheral artery disease (same as PVascularD)
Nocturnal pain relieved by lowering legs, poor pulses, dependent rubor, intermittent claudication, atrophy, shiny, hairless, cold feet. Initial do a pulse check, ABI 0.9 or less is PAD. Ateriography is the most DEFINITIVE test. Try to develop collateral circulation. Otherwise- Trental, Pletal ( a xanthine derivative used as a drug to treat muscle pain in people with peripheral artery disease)

devlopment warning signs
-visual delay
-does not pick up toy by 6 mo
-no reaction to noise
-no laugh
does not sit up
-does not raise head by 3 mo while on tummy

Gynecological & GI bleeding
What are the 2 most common causes of IDA in adults?

Tx for isolated systolic HTN
ccb

ADHD (Attention-Deficit Hyperactivity Disorder)
a psychological disorder marked by extreme inattention and/or hyperactivity and impulsivity
-s/s present before the age of 7 as late as 12
-s/s for at least 6 mo
-must occur in home, school and play

-Tx ritalin, concerta,metadate,focalin, adderall, vyanse

Costovertebral angle tenderness (CVA)
A pt with pyelonephritis will likely have what symptom?

S4 when
S4-LVH stiffening, Tennesse, late diastole. “Atrial kick/gallop”

menarche
breast dev stage 3 to 4, predominatly 4

No significant systemic effects. They are predictable.
Class effect of nasal steroids?

s3 when heard
S3- HF, Kentucky, early diastole. Abn >35. Bell

Spermarch
genital stage 3

Hyperopic
Farsighted (objects nearby are blurry)

VSD
thrill, felt at LLSB

Presbyopic
Age-related farsightedness

MVP how it sounds, when to treat and how to treat
MVP- S2 click, followed by systolic murmur. Asymptomatic. MVP with palpitations is treated with BB.

HTN meds that can cause heartburn
BB, CCB, alpha agonists. (HTN meds). EXAM

Myopic
Nearsighted

Tx sinusitis
AUGMENTIN. if pen allergic – doxy or levofloxacin (adults). kids get clinda+3rd gen cephalosporin (IDSA) adults get treated 5-7 days, kids get 10-14 days

In what condition are the turbinates dark red and swollen?
Acute rhinitis

test for mononucleosis
monospot is a heterophile antibody test

Gyrate lesions
Scabies

Otitis Externa (swimmers ear)- bacterial cause and tx
Pseudomonas aeruginosa. (other- S. aureus). External ear pain- d/c itching, hearing loss, tragus, green d/c. TREATMENT: Corticosporin, Cipro
Topical antimicrobials or antibiotics such as acetic acid, aminoglycosides, polymyxin B, and quinolones are the treatment of choice in uncomplicated cases

Most likely diagnosis for rash that starts on belly and spreads to the rest of the body including the face. Causes severe itching.
Varicella

Weber lateralization to which ear
Weber’s test is performed by softly striking a 512-Hz tuning fork and placing it midline on the patient’s scalp, or on the forehead, nasal bones, or teeth.
If the hearing loss is conductive, the sound will be heard best in the affected ear.
If the loss is sensorineural, the sound will be heard best in the normal ear.
The sound remains midline in patients with normal hearing

Pt: NP, I can’t hear in my left ear.
NP (wonders to herself): I wonder if it is conductive hearing loss.
There are three options:
Does Weber test and pt can hear best in left ear (affected ear) == “Sir, you have conductive hearing loss.”
Does Weber test and patient can hear best in RIGHT ear (normal ear) == “Sir, you have sensorineural hearing loss”
Hears both equally == “Sir, you do not have hearing loss.”

What skin condition is a raised reddened area with a dimpled look?
Basal cell carcinoma

Rinne test results and interpretation
AC>BC is normal. BC>AC is conductive hearing loss

Skin condition that can manifest when stressed. Itchy and dries up like cream colored sand paper before healing
Eczema

Incretins do what?
Signal the pancreas to increase insulin secretion & signals the liver to stop producing glucagon

AOM tx
Amox, If your patient is only PCN allergic do azithromycin or clarithromycin.

AOM bacterial cause
. Usually S. pneumo. (others: h influ, mor catarrhalis).

Lactic acidosis is a potential adverse side effect most commonly seen with which medication?
Metformin (Glucophage)

Koplik spots are a diagnostic indicator of
lusters sm. Size red papules w/ white centers in the buccal mucosa by lower molars -rubeola measles

What is a bubo?
a swollen, inflamed lymph node in the armpit or groin.

diabetic retinopathy
Diabetic Retinopathy-Cotton wool spots (moderate retinopathy), micro-aneurysms. ALSO RETINAL HEMORRHAGES ON CENTER OF EYE APPEAR ORANGE RED

Where is the site for most breast tumors in women?
Upper outer quadrant

hypertensive retinopathy
Hypertensive Retinopathy- Copper/silver wire arterioles. AV nicking(mild retinopathy). Retinal “flame” Hemorrhages

What is a positive prehn’s sign?
Lifting of the testes

papilledema appearance and indicates what
Papilledema- optic disc swollen w/ blurred edges due to increased ICP

Which cranial nerve is most responsible for papillary constriction?
Oculomotor

S/S of retinal detachment
Retinal Detachment- Floaters, curtain, flashes of light. Painless.

Which type of ulcer feels better right after a patient eats?
Duodenal ulcer

age-related macular degeneration: how affects sight, how test
Age-Related Macular Degeneration – Painless loss of “central vision” reports straight lines appear curved. Periphery is preserved. Give amsler grid.

The most critical assessment finding with acute abdomen with peritonitis with a bowel perforation is?
Rigidity

Which ECG changes would the provider see in a pt with classic angina?
ST depression

Cataracts, how affect elderly eye sight
in elderly night vision issues. Opaque

acute angle closure glaucoma: what patient will see, findings on physical exam, disposition
Acute Angle-closure glaucoma- acute/severe halos, cupping optic nerve, cloudy cornea, mid-dilated oval pupil. ER STAT

Dependent rubor is found in what disease?
Ateriosclerotic occlusvie disease

Primary open angle glaucoma, how affects sight
Primary Open Angle Glaucoma- CN2 gradual changes in peripheral vision LOST FIRST, then second central vision

What to test for with rash on hands and feet?
rash hands soles/feet think to test for secondary syphilis RPR then VDRL are screening, then dx FTA-ABS.

pityriasis rosea
pityriasis rosea itchy, herald patch, xmas tree pattern,

Wickham’s striae
fine, lace-like network of white lines in lichen planus

lichen planus tx
topical steroids

Describe scarlet fever rash
Scarlet fever (Scarlantina)- “sandpaper textured-pink rash with sore throat” strawberry tongue, rash starts on head and neck, spreads to trunk. The skin THEN desquamates.

Impetigo appearance
Impetigo-Gram positive. Itchy pink-red lesions, evolve into vesiculopustules that rupture

Treatment for impetigo, plus for allergies
. If bullous-large blisters. Severe- Keflex, dicloxacillin. PCN Allergic-Azithro, clinda.

acne rosacea: appearance and treatment
Acne Rosacea- chronic small acne like papules/pustules around nose mouth chin. TREATMENT- Metrogel, Azelex. Low dose tetracycline.

treatment mild acne
Acne Vulgaris (common acne)- ON EXAM
mild (topicals only) *open/closed comedone w/ or w/o sm. papules. Retin-A, acne worsens 4-6 weeks if no improvement in 8-12 weeks increase dose or add erythromycin, benzoyl peroxide.

treatment moderate acne
Moderate (topicals plus antibiotics)- papules, pustules w/ comedones. Continue with topicals combined with topical antibiotics. Then add ORAL antibiotics tetra, mino, doxy

treatment of severe acne
Severe- with painful indurated nodule, cysts, abscesses, pustules. Accutane- check LFTs, 2 forms of contraceptives, monthly prego testing, only prescribe 1 month supply.

varicella zoster
“contagious 48 h. before, until all lesions crusted over” low grade fever, generalized lymphadenopathy, intense itching, erythematous macules, papules develop over macules, then vesicles erupt. “initially on trunk, then scalp and face” TREATMENT supportive, antihistamines, acyclovir 20mg/kg 5xd. If given first 24 hours works best.

MRSA tx
MRSA TREATMENT: Bactrim, doxy, mino, clinda. If sulfa allergy do not use Bactrim.

actinic keratosis: s/s, gold standard dx, tx
Actinic Keratosis- Precursor to squamous cell carcinoma. “numerous dry round and pink to red lesions” with a rough and scaly texture. Does not heal. Slow growing in sun exposed areas. Diagnosis: BIOPSY Golden Standard. Treatment: Sm. (cryotherapy), Lrg. (5-FU cream)- which causes ur skin to ooze, crust, scab, redness.

Tinea Corporis
Tinea Corporis- “ring like itchy rash, slowly enlarge central clearing”-Treatment: most respond to topical antifungals, if severe do oral Lamisil.

Eczema, s/s, mediated by which immunoglobulin, tx
Atopic Dermatitis (eczema)- Inherited. Extremely itchy. On flexural folds, neck, hands. Inc. IgE. “small vesicles that rupture leaving painful, bright-red, weepy lesions” they become lichenified from itching. First line: Topical steroids. Avoid hot water/soaps. PO antihistamines.

Koebner phenomenon
(Koebner phenomenon- new psoriatic plaques form over skin trauma)

Psoriasis tx
TREATMENT: Topical steroids, Tar preps (mild). For (severe) do anti-TNF, or immunologic.

Melanoma: how it looks
Melanoma- Dark Moles, uneven texture, different colors, irregular, >6mm, could be itchy. EXAM

Lyme: name of rash, what is the rash like
Erythema Migraines- (stage 1 Lyme) Target bulls-eye, usually appears in 7-14 days POST being bitten by a deer tick. Rash is hot to touch with rough texture, flu like symptoms.

Lyme: how dx
DX: B. Burgdorferi via ELISA, then confirm with western blot. Increased ESR.

Lyme: tx
TREATMENT: Less than 7 Amoxicillin or cefuroxime axetil. Older than 7 Doxycycline.

Rocky mountain spotted fever: s/s
RMSF- Inc. fever, chills, N/v, photophobia, myalgia, arthralgias THEN 2-5 days later you develop a petechial rash on forearms, ankles, wrists, that spreads towards trunk and becomes generalized. Think rocky NC/OK/AK/TN/MO.

Rocky mountain spotted fever – dx
DX: PCR essay with Rickessetti Antigen

Rocky mountain spotted fever – tx
TREATMENT- doxycycline.

acute sinusitis
CP?
Diagnostic?
(DDx)
Pharm, NonPharm?
f/u
Recognize normal course? Refer?

  • S Pneu. H. Flu. M. Cat. Viral
  • Facial or upper molar pain. Nasal congestion >10d. Purulent nasal d/c or PND- awaken at night w cough. Fever (mostly w children not adults). Can be due to allergy flare-up, then boggy swollen nasal turbinate. Tender sinuses.
  • H&P, Positive transillumination.
  • If mild uncomplicated- topical decongestants (Afrin), Steroid and saline nasal spray, Mucolytic
  • If severe, 1st: Augmentin
    Alt: Levaquin, Doxy, Cefdinir, Cefin

Arcus Senilis
normal finding in elderly
white, grey, or blue opaque ring in the corneal margin (peripheral corneal opacity), or white ring in front of the periphery of the iris. It is present at birth but then fades; however, it is quite commonly present in the elderly.

Epistaxis
anterior -Kiesselbach plexus
risk- nose picking

posterior – Woodruff –HTN, atherosclerosis- can lead to hemorrhage- refer to ED

tx: Anterior — pressure sitting position leading forward -lessen swallowing of blood

Epiglottis
severe, life-threatening infection of the epiglottis and supraglottic structures that occurs most commonly in children between 2 and 12 years of age

Periorbital cellulitis
Strep pneumo, Strep pyogenes, H flu, Staph aureus, Staph epi, oral anaerobes

acute onset of erythematous swollen eyelid w proptosis (bulging of the eyeball) and eye pain. Bad EOM exam w pain. h/o recent rhinosinusitis or URI.
Caused by bacterial infection of the orbital contents (fat and ocular muscles).
More common in children than adults.
Serious complications. Refer to ED.

Strep throat pharyngitis
Grp A Strep

Fever, Sore throat, Tonsillar exudate, absence of cough

PCN V, Amoxicillin
Alt: Azithromycin
Ibuprofen/Tylenol

Complications:
Scarlet fever- Sandpaper texture maculopapular rash that desquamates. Strawberry tongue. Sore throat that can lead to Acute Rheumatic Fever (affects the heart and valves, joints, brain)
Poststrep glomerulonephritis- abrupt onset of proteinuria, hematuria, dark-colored urine, RBC casts a/b HTN and edema

Mono

AOM

OME

OE

macular degeneration (MD)

Retinopathy
HTN
DM

Herpetic whitlow
A VIRAL skin infection of the finger(s) that is caused by HERPES SIMPLEX (type 1 (ORAL) or type 2 (GENTIAL) virus infection, from DIRECT CONTACT with either a cold sore or genital herpes lesion.

Self-limited infection: ANALGESICS or nonsteroidal anti-inflammatory drugs (NSAIDs) for pain PRN.
SEVERE infections: Treat with ACYCLOVIR (Zovirax).

A 65-year-old woman presents for a follow-up examination after a new patient visit. She has not seen a healthcare provider for several years. She is a smoker and her hypertension is now adequately controlled with medication. Her mother died at age 40 from a heart attack. The fasting lipid profile shows cholesterol = 240 mg/dL, HDL = 30, and LDL = 200. In addition to starting Therapeutic Lifestyle Changes, the nurse practitioner should start the patient on:

1.bile acid sequestrant.

  1. a statin drug.
  2. a cholesterol absorption inhibitor.
  3. low-dose aspirin.
    A statin drug

The most commonly prescribed medication for mild systemic lupus erythematosus (SLE) is:

  1. azathioprine (AZA).
  2. belimumab (Benlysta).
  3. ibuprofen (Advil).
  4. cyclophosphamide (Cytoxan).
    ibuprofen (advil)

The most common sign of cervical cancer is:

  1. postcoital bleeding.
  2. strong odor from vaginal discharge.
  3. itching in the vaginal area.
  4. molluscum contagiosum.
    postcoital bleeding

The nurse practitioner prescribes amitriptyline (Elavil) for a patient with neuropathic pain secondary to diabetes mellitus. On follow-up, the patient complains of urine retention and dry mouth. The practitioner would:

  1. discontinue amitriptyline and begin ibuprofen (Motrin).
  2. refer to physical therapy.
  3. start methocarbamol (Robaxin).
  4. discontinue amitriptyline and begin gabapentin (Neurontin).
    discontinue amitriptyline and begin gabapentin (neurontin)

A 17-year-old male with rheumatoid arthritis is being treated with an NSAID and omeprazole (Prilosec). The patient complains of headache, abdominal pain, and gas. These symptoms are most likely:

  1. associated with the omeprazole.
  2. related to the underlying condition.
  3. the result of the NSAID.
  4. caused by viral gastroenteritis.
    associated with the omeprazole

The medication of choice for the initial treatment of juvenile rheumatoid arthritis is:

  1. acetaminophen.
  2. prednisone.
  3. aspirin.
  4. ibuprofen.
    ibuprofen

A 12-year-old with sickle cell anemia has recently experienced a sickle cell crisis and presents for a follow-up examination after a recent hospitalization. It is most important to continue monitoring growth, development, and:

  1. white blood cell levels.
  2. fecal occult blood test.
  3. hemoglobin levels.
  4. urine dipsticks.
    hemoglobin levels

A 90-year-old female is brought to the clinic by her neighbor. She states that everything is fine, but the nurse practitioner notes that she has poor hygiene and bruises on her trunk. The neighbor is concerned that the patient often has no money to buy food, despite income from social security and a coal miner’s pension. The nurse practitioner suspects abuse. Which of the following is the nurse practitioner obligated to do next?

  1. Report the case to the proper authorities.
  2. Tell the neighbor to check on the woman daily and report back.
  3. Document the data and report the information to risk management.
  4. Call the patient’s family and inquire about the concerns.
    report the case to proper authorities

In most cases, the first manifestation of Alzheimer’s disease is:

  1. impaired judgment.
  2. decrease in short-term memory.
  3. disorientation in time and place.
  4. decrease in long-term memory.
    decrease in short-term memory

The optimal treatment for latent tuberculosis is:

  1. rifampin (Rifadin) for 5 months.
  2. isoniazid (Nydrazid) for 9 months.
  3. pyrazinamide for 6 months.
  4. ethambutol for 6 months.
    osioniazid (nydrazid) for 9 months

Unilateral spontaneous serous or serosanguineous discharge from a single duct of a breast is most often caused by:

  1. intraductal papilloma.
  2. mucinous breast lesions.
  3. Paget’s disease.
  4. ductal carcinoma in situ.
    intraductal papilloma

A young child with asthma presents for follow-up evaluation. After numerous changes in medications and doses, the parents report that the child continues to have difficulty with coughing, especially during the night. Which of the following conditions would be the most likely cause of the continued asthma symptoms?

  1. Vocal cord dysfunction
  2. Cystic fibrosis
  3. Gastroesophageal reflux
  4. Allergic rhinitis
    gastroesophageal reflux

A pregnant woman with known HIV infection can reduce the risk of perinatal transmission through zidovudine (Retrovir) therapy. Based on current research, optimal therapy is to start daily dosing:

  1. post amniocentesis.
  2. after 14 weeks of gestation.
  3. if premature rupture of membranes occurs.
  4. if maternal viral loads are greater than 10,000.
    after 14 weeks gestation

T-wave inversion with a normal ST segment on a 12-lead EKG may represent:

  1. acute coronary ischemia.
  2. right ventricular hypertrophy.
  3. atrial hypertrophy.
  4. hyperkalemia.
    acute coronary ischemia

The 16-year-old mother of a 2-month-old presents the infant, reporting that the child is very irritable and does not feed well. During physical examination, the child’s head drops back and the child exhibits sudden flexing of the extremities. As the flexing stops, the child cries uncontrollably. Funduscopic examination reveals retinal hemorrhages. Which of the following diagnostic tests should be ordered?

  1. Skull X-rays
  2. MRI
  3. CT scan
  4. Pet scan
    CT scan

The most important diagnostic factor in evaluating angina pectoris is the patient’s:

  1. King of Heart’s monitor.
  2. physical examination.
  3. history.
  4. echocardiogram.
    history

There are three reasons for latex allergies. Which of the following does NOT cause the problem?

  1. Immediate hypersensitivity
  2. Irritant contact of dermatitis
  3. Cytotoxic hypersensitivity
  4. Immune complex reaction
    cytotoxic hypersensitivity

A patient on warfarin (Coumadin) therapy for recurrent deep vein thrombosis (DVT) is about to have lumbar spinal fusion surgery. The patient’s warfarin is put on hold starting 5 days prior to the surgery and subcutaneous Lovenox has been ordered for DVT prophylaxis until the resumption of the warfarin. The nurse practitioner knows that the patient’s postoperative warfarin dose should be restarted based on the:

  1. value of her morning Prothrombin time.
  2. loading dose of 10 mg, plus the previous warfarin dose.
  3. baseline PT and INR values.
  4. target INR of 2.
    caseline PT and INR values

An 87-year-old patient presents with round, pruritic plaques and small vesicles on the lower legs. The most likely diagnosis is:

  1. allergic contact dermatitis.
  2. plaque psoriasis.
  3. cutaneous T-cell lymphoma.
  4. nummular eczema
    nummular eczema

A 3-year-old presents with a 2-day history of acute diarrhea with a total of 8 watery stools without blood or mucus, and 2 episodes of vomiting in the past 48 hours. Assessment reveals no current antibiotic therapy, dehydration <5%, soft abdomen with hyperactive bowel sounds, no masses or organomegaly; other physical findings are normal. Which of the following is the most appropriate management plan?

  1. Stool culture; Immodium AD liquid.
  2. No laboratory workup; instruct parents on signs and symptoms of dehydration; diet of clear liquids, advancing to bananas, rice cereal, Jell-O, and soup.
  3. Start diet of clear liquids, advancing to bananas, rice cereal, Jell-O, and soup over next 72 hours; Imodium AD liquid, 1 mg/sml
  4. Stool for ova and parasites; advise parents of likely sources of Giardia; prescribe furazolidone (Furoxone)
    No laboratory workup; instruct parents on signs and symptoms of dehydration; diet of clear liquids, advancing to bananas, rice cereal, Jell-O, and soup.

A mother presents her 12-month-old child with concern because the child does not yet say “mama” or “dada.” The mother reports that in the first months of life, the child loved listening to music and being talked to. The past medical history is negative for ear infections. The nurse practitioner should:

  1. evaluate for hearing loss.
  2. refer to speech therapy.
  3. re-evaluate in 3 months.
  4. check for cerumen impaction.
    evaluate for hearing loss

The most common side effect of the oral ribavirin used in the treatment of hepatitis C is:

  1. hemolytic anemia.
  2. weight loss.
  3. depression.
  4. hypothyroidism.
    hemolytic anemia

An 86-year-old patient presents with pinguecula, a yellow triangular degenerative tissue thickening of the bulbar conjunctiva. This condition is:

  1. cured by eye drops.
  2. a genetic process.
  3. a malignant growth.
  4. a solar-induced lesion
    a solar-induced lesion

During a well-child examination of an 18-month-old, premature tooth decay and inflamed gums, particularly of the maxillary incisors, are noted. The approach to this problem includes:

  1. referring the child to a dentist, assessing for bottle feeding and fluoride in local water.
  2. teaching the mother to brush the child’s teeth, encouraging vitamin supplements.
  3. recommending fluoride mouthwash and toothpaste, suggesting celery sticks for snacks.
  4. providing fluoride treatment, teaching dental hygiene, and eating foods high in calcium.
    referring the child to a dentist, assessing for bottle feeding and fluoride in local water

Which of the following is typically observed in the GI system upon stimulation of the parasympathetic nervous system?

  1. No overall effect
  2. Increased sphincter tone
  3. Increased peristalsis
  4. Decreased secretions
    increased peristalsis

Type 2 diabetes mellitus is characterized by which of the following?

  1. Decreased production of exogenous glucagon-like peptide
  2. Autoimmune beta-cell destruction
  3. Relative deficiency of dipeptidyl peptidase-4
  4. Decreased reabsorption of sodium-glucose transporters
    decreased reabsorption of sodium-glucose transporters

In pulmonary function testing, forced vital capacity represents the:

  1. maximum volume of air that can be forcefully exhaled after maximum inspiration.
  2. total volume of air that is exhaled after normal inspiration.
  3. total volume of air that the lungs can hold, minus the expiratory reserve volume.
  4. volume of air that can be forcefully exhaled in 1 second.
    maximum volume of air that can be forcefully exhaled after maximum inspiration

An adult male presents as first patient of the day after awaking about 5:00 AM with indigestion and chest pressure. On checking his pulse, he found no change from previous measurements. He states that, while waiting for the nurse practitioner to arrive, he began to feel a little clammy. A 12-lead EKG records the following patterns. On the basis of these EKG readings, the diagnosis is:

  1. anterior-septal acute myocardial infarction.
  2. lateral acute myocardial infarction.
  3. inferior acute myocardial infarction.
  4. anterior acute myocardial infarction.
    inferior acute myocardial infarction

During the past 24 hours, a 62-year-old has experienced abdominal pain that radiates to the back. The patient also reports several episodes of nausea and vomiting, a low-grade temperature, and a history of excessive drinking. Physical examination reveals a distended abdomen. Laboratory serum values indicate elevated alkaline phosphatase, amylase, and serum lipase. The most likely diagnosis is:

  1. alcoholic liver disease.
  2. acute mesenteric ischemia.
  3. viral hepatitis.
  4. acute pancreatitis.
    acute pancreatitis

An otherwise healthy adult female presents complaining of vaginal burning and itching during urination. She has a social history of two sexual partners. She denies fever or vaginal discharge, but has noted a lesion on one labium. The nurse practitioner should order all of the following EXCEPT:

  1. wet mount and potassium hydroxide.
  2. dark-field microscopy of fluid from the lesion.
  3. nucleic acid amplification testing for gonorrhea.
  4. serology for Haemophilus ducreyi.
    dark-field microscopy of fluid from lesion

A 3-year-old female has been diagnosed with bacterial meningitis. She attends preschool daily at a local church day care program. The day before her diagnosis, she ate lunch with her mother at a local restaurant. For which contact(s) would chemoprophylaxis with rifampin be recommended?

  1. Preschool contacts in the past 7 days
  2. A friend of the patient’s sister who visited after school yesterday
  3. The checkout employee at the grocery store yesterday
  4. Diners at the restaurant during lunch the day the patient ate there
    preschool contacts in past 7 days

Congenital dysplasia of the hip:

  1. more commonly affects both hips.
  2. occurs more commonly in females.
  3. is correctable at any age.
  4. typically self-corrects by 12 weeks of age.
    more commonly affects both hips

Anticholinergic agents such as ipratropium (Atrovent) and tiotropium (Spiriva) are used in COPD primarily to:

  1. induce bronchodilation.
  2. decrease airway inflammation.
  3. expand the lung fields.
  4. treat hypoxemia.
    induce bronchodilation

The clinical presentation of placenta previa that develops during the third trimester includes:

  1. intermittent dark-red spotting.
  2. occult bleeding with abdominal or back pain.
  3. sudden onset of painless and profuse bleeding.
  4. painless occult bleeding that becomes visible upon onset of labor.
    sudden onset of painless and profuse bleeding

A 27-year-old male patient, who works as a janitor, presents with a 6-month history of an intermittent rash on his hands. History reveals itching and occasional burning. Examination reveals irregularly-distributed scaly maculopapular erythematous patches extending from the dorsum of the hand several inches up the forearms, and dry palms with no nail involvement. The most likely diagnosis is:

  1. contact dermatitis.
  2. scabies infestation.
  3. psoriasis.
  4. eczema.
    contact dermatitis

Anorexia nervosa occurs most commonly in which of the following?

  1. Painters
  2. Only children
  3. High-level athletes
  4. Individuals from large families
    high level athletes

A frail elderly patient presents with constipation. Which of the following normal physiologic changes seen with aging is the most likely cause?

  1. Decreased bowel muscle tone
  2. Increased bile secretion
  3. Increased absorption of calcium
  4. Decreased pancreatic secretions
    decreased bowel muscle tone

A patient reports to your office, having been struck on the right cheek with a baseball bat. On examination, you notice tenderness and swelling over the cheek. The patient is unable to gaze laterally with his left eye, and he complains of double vision when he attempts to do so. This is most likely associated with:

  1. retinal hemorrhage.
  2. orbital rim fracture.
  3. subarachnoid hemorrhage.
  4. periorbital cellulitis.
    orbital rim fractures

A 7-year-old child presents with group A streptococcal infection confirmed by throat culture. Past history includes treatment for positive streptococcal infection with erythromycin (EryPed oral suspension) 3 weeks ago. What is the most appropriate next intervention?

  1. Treat with amoxicillin for 10 days.
  2. Obtain culture for all household members.
  3. Treat with azithromycin (Zithromax) for 10 days.
  4. Intramuscular Penicillin.
    treat with amoxicillin for 10 days

A 16-year-old female adolescent is brought to the nurse practitioner’s office by her mother, who is concerned about her daughter’s recent weight loss. History reveals that the daughter was consistently in the 50th percentile for weight, but is now in the 10th percentile. The mother states that she is concerned that her daughter is purging herself after meals since she often goes to the restroom after eating and remains there for a long period of time. The daughter denies any self-induced vomiting, starvation or excessive activity. She does state that she jogs five miles a day and is in good condition. In addition to a complete blood count with differential, which of the following laboratory tests will be most helpful for further assessment?

  1. Electrolytes, fasting blood sugar
  2. Creatine phosphokinase, follicle-stimulating hormone (FSH)
  3. Electrolytes, blood urea nitrogen (BUN)/creatinine, urinalysis
  4. Electrolytes, FSH, stool for occult blood
    electrolytes, blood urea nitrogen (BUN)/creatinine, urinalysis

An adult female who is homeless presents for an initial obstetric visit at 34 weeks of pregnancy. She is diagnosed with Chlamydia trachomatis infection. What complication will her newborn be most at risk?

  1. Conjunctivitis
  2. Hearing loss
  3. Pneumonitis
  4. Meningitis
    conjunctivitis

A 2-year-old male presents for reevaluation. Two days ago, he had four episodes of vomiting and six diarrheal stools. On physical examination today, his vital signs reveal P = 120 and capillary refill of 3 seconds. The patient’s eyes are sunken and his extremities are mottled and cool to the touch. The treatment plan would include:

  1. start oral electrolyte mixture every hour.
  2. hospitalization for IV fluid replacement.
  3. giving the child loperamide for each diarrheal stool.
  4. catheterizing the child to obtain accurate urine output.
    hospitalization for IV fluid replacement

A nurse practitioner places a 76-year-old patient on nifedipine (Procardia) 10 mg t.i.d. for angina. The patient is unable to remember to take the medication at the scheduled times. The practitioner should:

  1. discontinue the issue with the patient’s daughter.
  2. change the dose to extended release 30 mg daily.
  3. reinforce the importance of taking the medication.
  4. increase the dosage to 20 mg b.i.d.
    change the dose to extended release 30mg daily

A pediatric male is present with his mother, who states that he is extremely restless at night and constantly scratches his anal area. Which diagnostic procedure would confirm a diagnosis of pinworm infestation?

  1. The patch test
  2. Examination of bed linens
  3. Examination of transparent tape sample from perianal skin
  4. Blood tests for anemia and eosinophilia
    examination of transparent tape sample from perianal skin

Which of the following microorganisms are most frequently associated with acute bacterial rhino-sinusitis?

  1. Staphylococcus aureus and Mycoplasma pneumonia
  2. Staphylococcus aureus and Methicillin Resistant Staph aureus
  3. Streptococcus pneumoniae and Haemophilus influenzae
  4. Streptococcus pneumoniae and Pseudomonas aeruginosa
    strep pneumo and haemophilus influenzae

A child is being treated with methylphenidate (Ritalin) for attention-deficit disorder. The following are all side effects of Ritalin EXCEPT:

  1. muscle cramps
  2. mild irritability.
  3. increased heart rate.
  4. slight increase in growth velocity.
    slight increase in growth velocity

A geriatric female presents for her annual examination. She has been on antihypertensive medications for over 20 years, with good control. Laboratory values are within normal ranges. The nurse practitioner is concerned about the patient’s cardiac health risks, due her to weight and her waist circumference. According to the AHA guidelines, which of the following goals is expected for this patient?

  1. Physical activity for 60 minutes daily, for a minimum of 6 days a week
  2. Physical activity for 30 minutes daily, 7 days a week
  3. Physical activity for 30 minutes daily, for a minimum of 5 days a week
  4. Physical activity for 60 minutes daily, 7 days a week
    physical activity for 30 minutes daily, for a minimum of 5 days a week

The management of COPD in the elderly is best guided by:

  1. spirometry.
  2. arterial blood gases.
  3. radiologic imaging.
  4. symptomatology.
    symptomatology

A young, athletic adult presents with shoulder pain and inability to raise the arm above the shoulder. The most likely diagnosis is:

  1. frozen shoulder syndrome.
  2. cervical radiculopathy.
  3. rotator cuff tear.
  4. polymyositis.
    rotator cuff tear

A 50-year-old male presents with bruises and abrasions on the left leg and forearm, claiming to have had a number of minor accidents at home and at work. The patient reports general feelings of depression, forgetfulness, and, despite the fact that he is regularly employed, a mounting collection of unpaid and overdue bills. In taking the patient’s history, a significant pattern of parental alcohol abuse, stroke, TIA, and MI emerges. During questioning to elicit a more thorough diagnosis, the patient becomes hostile and leaves the clinic without further treatment. The most likely explanation for this behavior is:

  1. Alzheimer’s disease.
  2. chronic alcoholism.
  3. cancer.
  4. carotid artery stenosis.
    chronic alcoholism

A 28-year-old patient presents with profuse yellow vaginal discharge, odor, and local irritation. She reports completing a 7-day course of oral metronidazole (Flagyl), 500 mg b.i.d. 4 weeks ago. She has had multiple recurrences over the last 18 months. Microscopic examination reveals presence of clue cells. What is the most appropriate treatment intervention?

  1. Oral metronidazole (Flagyl), 500 mg b.i.d., plus metronidazole vaginal gel (MetroGel), for 5 days
  2. Treat both patient and partner(s) with oral metronidazole (Flagyl), 500 mg b.i.d., for 7 days
  3. Metronidazole vaginal gel (MetroGel) twice weekly for 4 to 6 months
  4. Treat patient with metronidazole vaginal gel (MetroGel) b.i.d. and partner(s) with tetracycline, 250 mg q.i.d., for 7 days
    Treat both patient and partner(s) with oral metronidazole (Flagyl), 500 mg b.i.d., for 7 days

An adult female who recently returned from a business trip to Japan presents for a recheck appointment. The only remarkable laboratory result is for thyroid-stimulating hormone (TSH), at 0.3 microunits/mL (normal = 0.4-6 microunits/mL). The patient reports that her neck hurts; examination reveals thyroid tenderness. Which of the following laboratory tests should the nurse practitioner order now?

  1. Triiodothyronine (T3) and free thyroxine (FT4)
  2. Triiodothyronine (T3) only
  3. Triiodothyronine (T3) resin uptake assay
  4. Triiodothyronine (T3) and free triiodothyronine (FT3)
    Triiodothyronine (T3) and free thyroxine (FT4)

In treating a pregnant female with migraine headaches, which of the following drugs is in Category X?

  1. Ergotamine tartrate (Ergomar)
  2. Sumitriptan succinate (Imitrex)
  3. Frovatriptan (Frova)
  4. Amitriptyline (Elavil)
    ergotamine tartate (ergomar)

A nurse practitioner is evaluating an infant for possible colic. Which of the following could indicate the need for a more extensive evaluation?

  1. 3-oz (85 g) weight gain over the past 2 weeks
  2. Stool negative for occult blood
  3. Moist mucus membranes and flat fontanels
  4. Onset at 4 weeks of age
    3-oz (85 g) weight gain over the past 2 weeks

Which of the following is a hallmark of lumbar spinal stenosis?

  1. Incontinence of bowel and/or bladder
  2. Point tenderness of the lumbar spine
  3. Leg, buttock or back pain precipitated by walking.
  4. Bilateral leg pain with sitting.
    leg, buttock or back pain precipitated by walking

A 70-year-old patient presents to the clinic complaining of dyspnea, palpitations, and fatigue. The patient reports a 2-week history of blackened stools, which the patient attributes to drinking berry juice. Assessment reveals vital signs of BP = 110/60, P = 100, R = 24; Hgb = 4.5 g/dL; Hct = 16%. What is the most appropriate immediate intervention?

  1. Order serum iron, total iron-binding capacity (TIBC), and ferritin.
  2. Refer to a gastroenterologist.
  3. Send to the emergency room.
  4. Order a complete blood count (CBC) with differential.
    Send to emergency room

The symptoms of benign prostatic hypertrophy are often overlooked because the older male:

  1. understands that surgery is the only cure for this condition.
  2. considers urinary problems to be a normal part of aging.
  3. is fearful of the side effects of additional medication.
  4. is concerned that he will no longer be able to obtain an erection.
    considers urinary problems to be normal part of aging

Upon admission to a nursing home, an 85-year-old patient receives a negative result on the purified protein derivative (PPD) test. To determine TB status in the frail elderly, a second PPD should be performed in:

  1. 72 hours.
  2. 1 week.
  3. 3 months.
  4. 6 months.
    1 week

Which of the following is NOT used for therapy of an acute attack of gout?

  1. Colchicine (Colcrys)
  2. Indomethacin (Indocin)
  3. Methylprednisolone (Solumedrol)
  4. Allopurinol (Zyloprim)
    allopurinol (zyloprim)

A 26-year-old female presents with abdominal distention, bloating, and intermittent crampy abdominal pain relieved by defecation. She has four to six loose stools a day when stressed. Between bouts of diarrhea, she often has constipation. Physical examination results are normal. The CBC was normal. The most likely diagnosis is:

  1. diverticulitis.
  2. viral gastroenteritis.
  3. inflammatory bowel disease.
  4. irritable bowel syndrome
    irritable bowel syndrome

A mother has just noticed that her 15-month-old, 15-kg child has recently ingested an unknown quantity of iron tablets. The mother has ipecac syrup and activated charcoal on hand. The mother should be advised to:

  1. take the child to the primary care provider.
  2. give ipecac per recommendation on the bottle label.
  3. take the child to the emergency room.
  4. give the child 30 g of activated charcoal diluted with water
    take child to emergency room

A 13-year-old is concerned because she has not yet begun to menstruate. Physical examination indicates that the patient is at Tanner stage IV and is of average height and weight. Which of the following would be the most appropriate response to this patient?

  1. “We’ll need to refer you to an endocrinologist for a complete workup.”
  2. “We’ll need to do some tests to find out why you are not having periods.”
  3. “I’ll give you some pills that will make your periods start.”
  4. “Your development is exactly as expected for your age; you’ll probably begin to have periods within a year.”
    “Your development is exactly as expected for your age; you’ll probably begin to have periods within a year.”

The most common bacterial cause of cellulitis in patients with diabetes mellitus is:

  1. Pseudomonas aeruginosa.
  2. group B beta-hemolytic Streptococcus.
  3. Staphylococcus aureus.
  4. Staphylococcus saprophyticus.
    Staphylococcus aureus

A 46-year-old female found a lump in her breast this morning. History includes: no prior breast disease; G2P2 (first birth, age 22); maternal aunt diagnosed with breast cancer at age 72; last menstrual period was 2.5 weeks ago. On examination, a nurse practitioner palpates a 2-cm round, soft, mobile, tender lesion in the upper outer quadrant of the right breast. Mammogram is negative. Which of the following actions is most appropriate?

  1. Reassure the patient that no further action is necessary.
  2. Repeat unilateral right mammogram in 3 months.
  3. Schedule a right-breast ultrasound.
  4. Refer for genetic testing if the lump increases in size.
    schedule a right breast ultrasound

Risk factors for genital herpes include:

  1. multiple sexual partners and years of sexual activity.
  2. genetic transmission.
  3. infection with other sexually transmitted diseases before the age of 12.
  4. multiple sexual partners and frequent use of public spas.
    multiple sexual partners and years of sexual activity

On physically assessing a newborn’s head, you note an enlarged, fluid-filled area that is subperiosteal and does not cross suture lines. This finding indicates the presence of:

  1. macrocephaly.
  2. subgaleal hemorrhage.
  3. congenital hypothyroidism.
  4. cephalohematoma.
    cephalohematoma

A 65-year-old patient being treated with digoxin (Lanoxin) for chronic heart failure (HF) presents with complaints of palpitations. Three days ago, the patient was placed on clarithromycin (Biaxin) by another provider for community-acquired pneumonia. The appropriate course of action at this point would be to:

  1. increase the digoxin dosage by 25%.
  2. obtain a digoxin level and change the antibiotic to azithromycin (Zithromax).
  3. obtain a chest X-ray.
  4. obtain a serum potassium level and perform a 12-lead EKG.
    obtain a digoxin level and change the antibiotic to azithromycin (zithromax)

A 54-year-old male who is 30 pounds overweight states that he awakens at night with heartburn and the taste of hot acid in his mouth. Stress makes his condition worse, yet baking soda seems to provide some relief. Findings on examination are normal and the stool for occult blood is negative. The preliminary diagnosis(es) should be:

  1. esophagitis.
  2. esophageal spasm.
  3. helicobacter pylori.
  4. gastroesophageal reflux disease.
    GERD

A late adolescent patient presents with complaints of acute-onset unilateral right eye pain, visual changes, seeing halos around lights, abdominal pain, and intermittent nausea. Physical findings include an erythematous right eye without discharge, a cloudy cornea, and a moderately dilated right pupil that is nonreactive to light. The nurse practitioner should immediately:

  1. apply steroid drops to the right eye.
  2. apply mydriatic drops to the eye.
  3. refer to the emergency department.
  4. make an appointment for him to see an ophthalmologist.
    refer to emergency department

In a 6-year-old child, which of the following is the preferred drug for stage 1 Lyme disease?

  1. Trimethoprim-sulfamethoxazole (Bactrim)
  2. Azithromycin (Zithromax)
  3. Doxycycline (Doryx)
  4. Cefuroxime (Ceftin)
    cefuroxime (ceftin)

An 86-year-old patient is experiencing radiculopathy-associated lower back pain that has not improved over the past 4 weeks. The medical history includes a lumbar discectomy. The best type of imaging study for evaluating this patient would be a/an:

  1. MRI.
  2. spinal X-ray.
  3. myelography.
  4. nuclear bone scan.
    MRI

A 30-month-old with a history of frequent ear infections was seen in September after a symptom-free summer. Examination revealed bilateral bulging eardrums. After two separate courses of antibiotics, fluid is noted in both ears. The parent reports speech difficulties. The most likely diagnosis is:

  1. otitis media with effusion.
  2. myringitis.
  3. recurrent acute otitis.
  4. Esutachian tube dysfunction.
    otiti media with effusion

A 19-year-old patient who has used oral contraceptives for 3 years plans to discontinue the pill at the end of her current cycle to become pregnant. Which of the following daily supplements would receive the highest priority?

  1. Calcium carbonate
  2. Folic acid
  3. Ferrous sulfate
  4. Multivitamin
    folic acid

A frail elderly widow presents to the senior clinic with a 3- to 4-day history of confusion that became worse the evening before but currently seems a little better, although her speech is rambling and difficult to follow. During the examination, the patient is anxious and agitated. She is on cardiac medications, an antidepressant, and insulin. The most likely diagnosis is:

  1. mild stroke.
  2. polypharmacy.
  3. electrolyte imbalance.
  4. agitated depression.
    polypharmacy

A 46-year-old female presents with a complaint of amenorrhea for 3 months. The first-line diagnostic workup is:

  1. urine pregnancy test.
  2. progesterone challenge.
  3. serum prolactin level.
  4. thyroid-stimulating hormone (TSH).
    urine pregnancy test

Leave a Comment

Scroll to Top